Sunteți pe pagina 1din 182

RCC &

STEEL STRUCTURES

For
CIVIL ENGINEERING
RCC & STEEL STRUCTURES
SYLLABUS
RCC: Working stress, Limit state and Ultimate load design concepts; Design of beams, slabs,
columns; Bond and development length; Prestressed concrete; Analysis of beam sections at
transfer and service loads.

Steel Structures: Working stress and Limit state design concepts; Design of tension and
compression members, beams and beam- columns, column bases; Connections - simple and
eccentric, beam- column connections, plate girders and trusses; Plastic analysis of beams
and frames.

ANALYSIS OF GATE PAPERS


Exam Year 1 Mark Ques. 2 Mark Ques. Total
2003 3 5 13
2004 2 5 12
2005 2 6 14
2006 1 5 11
2007 - 6 12
2008 2 5 12
2009 2 2 6
2010 1 2 5
2011 4 2 8
2012 2 3 8
2013 4 1 6
2014 Set-1 2 3 8
2014 Set-2 5 - 5
2015 Set-1 4 2 8
2015 Set-2 1 3 7
2016 Set-1 1 2 5
2016 Set-2 1 1 3
2017 1 2 5

© Copyright Reserved by Gateflix.in No part of this material should be copied or reproduced without permission
CONTENTS (RCC)
Topics Page No
1. BASICS

1.1 Plain and Reinforced Concrete 01


1.2 Compressive Strength of Concrete in Structure 02
1.3 Flexural Strength of Concrete (Modules of Rupture) 02
1.4 Tensile Strength of Concrete 02
1.5 Stress-Strain Curve of Concrete 03
1.6 Concrete Strain at Ultimate Strength 04
1.7 Design Stress-Strain Curve 04
1.8 Shrinkage and Creep in Concrete 04
1.9 Design Stress Strain Curve for Steel 06

2. DESIGN OF RCC STRUCTERES

2.1 Introduction 07
2.2 What is Limit State Method 07
2.3 Characteristic Strength of Method 07
2.4 Characteristic Load 08
2.5 Partial Safety of Factor 08
2.6 Assumption Limit State of Collapse: Flexure 09
2.7 Working Stress Method 12
2.8 Deficiency in Working Stress Method 12
2.9 Permissible Stresses 13
2.10 Permissible Stresses in Concrete 13
2.11 Permissible Stress in Steel Reinforcement 13

3. CONCRETE TECHNOLOGY

3.1 Introduction 15
3.2 Chemical Composition of Raw Materials 16
3.3 Type of Cements 16
3.4 Admixture 18
3.5 Different Type of Admixture 19
3.6 Aggregates 20
3.7 Permissible Limits for Impurities in Water 21
3.8 Concrete 21
3.9 Methods of Proportional Concrete 21
3.10 Water Cement Ratio 22

4. REINFORCED SECTION

4.1 Balanced, Under-Reinforced And Over Reinforced Section 23

© Copyright Reserved by Gateflix.in No part of this material should be copied or reproduced without permission
4.2 Moment of Resistance Calculation 24
4.3 Doubly Reinforced Section 26
4.4 Analysis Doubly of Reinforced Section 26
4.5 Moment of Resistance Doubly of Reinforced Section 27
4.6 Design of Doubly of Reinforced Section 27
4.7 Flanged Beam 28
4.8 Effective Flange Width 29
4.9 IS Code Specification 32
4.10 Design of Flanged Section 33
4.11 Limit State of Collapse in Shear 34
4.12 Diagonal Tension and Diagonal Compression 34
4.13 Mechanism of Shear Resistance 36
4.14 Shear Stress 36
4.15 Design Shear Strength of Concrete in Beams 37
4.16 Shear Reinforcement in Beam 39
4.17 Minimum Shear Reinforcement 42
4.18 Design Steps 42
4.19 Bond and Anchorage 43
4.20 Torsion 45
4.21 Effect of Torsional Moment 46

5. BEAM COLUMN

5.1 Design of Beam and Slab 48


5.2 I.S 456 Provisions 48
5.3 Control of Deflection 49
5.4 One-way Slab 50
5.5 Introduction 50
5.6 One-way and Two-way Slabs 51
5.7 Code Requirements on Reinforcement and Detailing 51
5.8 Column Subjected to Axial Compression and Uniaxial bending
(clause 39.5 ) 55
5.9 Column Subjected to Axial Compression and Biaxial bending
(clause 39.6) 56
5.10 Code Procedure for Design of biaxilly Loaded Columns 56
5.11 Depth of Foundation 58
5.12 Design Consideration 58

6. BASIC ELEMENT OF PRESTRESS CONCRETE

6.1 Losses in Priestess 62


6.2 Loss of Prestress Due to Friction 63
6.3 Loss of Prestress Due Anchorage Slip 63
6.4 Loss of Prestress due to Creep of Concrete 64
6.5 Loss due to Shrinkage of Concrete 65
6.6 Loss of Prestress due to Relaxation of Steel 65
6.7 Deflection of Restressed Beam 66

© Copyright Reserved by Gateflix.in No part of this material should be copied or reproduced without permission
6.8 Prestressed Concrete 68
6.9 Requirement of High Strength Steel & Concrete in Prestressing 69

7. GATE QUESTIONS 71

CONTENTS
(STEEL STRUCTURES)
Topics Page No
1. GENERAL DESIGN SPECIFICATION

1.1 Design Objective 98


1.2 Methods of Design 98
1.3 Loads and Forces 98
1.4 Load Combinations 98
1.5 Geometrical Properties 98
1.6 Classification of Cross Sections 98
1.7 Limit State Designs of Steel Structures 99
1.8 Connections: Riveted Connection 99
1.9 Terminology in Bolted Connection 101
1.10 IS 800-2007 Specification for Spacing and Edge Distances
of Bolt Holes 101
1.11 Types of Joints 102
1.12 Design Strength of Plates in a Joint 103
1.13 Block Shear Strength 104
1.14 Design Strength of Bearing Blots 104
1.15 Bearing Capacity or Bolts �Vdpb � 104
1.16 Tensile Capacity of Bolts 105
1.17 Welded Connection 105
1.18 Types of Welded Joints 105
1.19 Specification for Welding 106
1.20 Fillet Weld 107
1.21 Reduction in Design Stresses for Long Joints 108
1.22 Eccentric Connection - Plane of Moment and The Plane of
Welds is the same 108

2. DESIGN OF TENSION MEMBERS


2.1 Tension Member 110

© Copyright Reserved by Gateflix.in No part of this material should be copied or reproduced without permission
2.2 Design Strength Due to Yielding of Gross Section 110
2.3 Design Strength Due to Rapture of Critical Section 110
2.4 Design Strength Due to Block Shear 112
2.5 Lug Angles 112

3. DESIGN OF COMPRESSION MEMBERS

3.1 Introduction 114


3.2 Slenderness Ration 114
3.3 Design Compressive Stress and Strength 115
3.4 How to Select the Shapes of Compression Members 115
3.5 Steps for Design of Compression Members 116
3.6 Battens 116
3.7 Design of Battened Columns 116
3.8 Design of Slab Base 117

4. DESIGN OF BEAMS

4.1 Introduction 119


4.2 Bending Strength of Laterally Supported Beam 119
4.3 Design Procedure 120
4.4 Shear Strength of Laterally Supported Beam 120
4.5 Web Buckling Strength 121
4.6 Web Crippling 121

5. Plastic Analysis 122


5.1 Introduction 122
5.2 Fully Plastic Moment of a Section 122
5.3 Bending of Beams Symmetrical About Both Axes 123
5.4 Shape Factor 124
5.5 Load Factor 124
5.6 Fundamental Conditions for Plastic Analysis 124
5.7 Mechanism 124
5.8 Basic Theorems of Plastic Analysis 125

6. GATE QUESTIONS 126

7. ASSIGNMENT QUESTIONS (RCC & STEEL STRUCTURE) 151

© Copyright Reserved by Gateflix.in No part of this material should be copied or reproduced without permission
RCC

© Copyright Reserved by Gateflix.in No part of this material should be copied or reproduced without permission
1 BASICS

1.1 PLAIN & REINFORCED CONCRETE


Plain Concrete:
• Compressive strength of concrete is
It is a mixture of sand, gravel, cement, and
the most important property of
water which results in a solid mass.
concrete. Because other properties like
Concrete is strong in compression but weak
in tension. Its tensile strength is approx,
one tenth of compressive strength. Plain
concrete is mostly used in mass concrete
work. (As in dams) tensile strength, shear strength, bond
1.1.1 REINFORCED CONCRETE strength, density, impermeability,
durability etc. can be inferred from the
1) It is a concrete with reinforcement compressive strength using established
embedded in it. The embedded correlations.
reinforcement makes it capable of
resisting tension also.
2) Steel bars embedded in the tension
zone of concrete, relieves concrete of
any tension and takes all tension
without separating from concrete.
3) The bond between steel and
surrounding concrete ensures strain • Compressive strength can be
compatibility i.e., the strain at any point measured by standard test on concrete
in the steel is equal to that in the cube. (or cylinder) specimen.
adjoining concrete.
4) Reinforcing steel imparts ductility to
concrete which is otherwise brittle
material.
(5) Here ductility means large deflection
owing to yielding of steel, thereby
giving ample warning of impending
• Strength of concrete in uniaxial
collapse.
compression is determined by loading
6) Tensile stress in concrete arises on
standard test cube (150 mm size) to
account of direct tension, flexural
failure in compression testing machine.
tension, diagonal tension (due to shear),
temperature and shrinkage effect, • The test specimens are generally tested
28 days after casting (and continuous
restraint to deformation.
curing)
7) Under these conditions, reinforcements
must be provided across potential • Cube is always tested on sides i.e., face
tensile crack. in touch with mould.
• Strength of cube is expressed to be
1.1.2 GRADE OF CONCRETE nearest of 0.5N/mm2.
• As per IS 456: 2000, three test
• specimens of a sample are taken.

© Copyright Reserved by Gateflix.in No part of this material should be copied or reproduced without permission
• Additional samples may be required for fcr = 0.4W N/mm2 [W is in kN for onset of
various purposes such as to determine cracking.]
the strength of concrete at 7 days or for However, stress strain variation is not
finding out the strength for striking the linear hence as per IS code
formwork, or to determine the duration fcr = 0.7 f ck N/mm2
of curing, or to check the testing error.
Flexural strength is used to determine the
Additional specimen may also be
onset of cracking or the loading at which
required for testing samples cured by
cracking starts in a structure.
accelerated methods.
• To report, strength of cube, we take 1.4 TENSILE STRENGTH OF CONCRETE
average of the strength of three
specimen of a sample. Tensile strength of plain concrete is
• Individual variation should not be more obtained by the splitting test.
than ±15% of average if variation is 2P
more, test results of the sample are Splitting tensile strength f ct =
πdL
invalid.
• fct = splitting tensile strength = 0.66 fcr,
1.2 COMPRESSIVE STRENGTH OF where fcr is Modulus of rupture
CONCRETE IN STRUCTURE • Direct tensile strength = [0.5 – 0.625]
fcr.
Strength of concrete is found to decrease
with increase in the size of specimen.
However, beyond 450mm size, there is no
decrease in the compressive strength of
concrete.
Thus, compressive strength of concrete in
structure is taken as 0.67 fck.

1.3 FLEXURAL STRENGTH OF


CONCRETE (MODULES OF RUPTURE)

Tensile strength of concrete in flexure is 1.5 STRESS-STRAIN CURVE OF CONCRETE


called flexural strength.

M W×100
= ×103 N/mm2 [if W is in • It is found by testing cylinder under
Z 6×100× (100 ) 2

compression. The max. Strength


6 obtained is the cylinder strength.
kN] [Assuming linear stress strain curve Cylinder strength = 0.8 × cube strength
and contribution of steel area to be (150 mm φ , 300 long)
negligible]

© Copyright Reserved by Gateflix.in No part of this material should be copied or reproduced without permission
• Cylinder is tested to obtain stress strain (Crushing strain is 0.3%-0.5%)
curve, because curve, we have to obtain 7) Curves are generally linear up to a
condition for uniaxial stress condition. stress of 0.6 times the peak stress.
In case of cubes, due to friction between 8) Modulus of elasticity of concrete for
the all practical purpose is taken as
concrete surface and the steel plate of secant modulus at a stress of around
testing machine, lateral restraints 0.33 f ck . This Ec is generally found
occurs. acceptable in representing an
• The effect of this lateral restraint is to average value of Ec under service
compressive strength in longitudinal load condition (static loading).
direction. This effect dies down with
increasing distance from the friction
surface [called platen restraint]. Thus as
the distance from the friction surface
increases (i.e., as height/width ratio
increases), compressive strength
decreases.

• Modulus of elasticity is primarily


influenced by the elastic properties of
aggregate and to a lesser extent by the
conditions of curving, age of concrete,
mix proportion and type of cement.
• As per IS code:
E c = 5000 f ck ( E c and f ck are in
N/mm2)
Short term modulus of elasticity of
concrete
• Long term modulus of elasticity
From the above stress-strain curve,
E
following points must be noted. including creep E ce = c
1) Max compressive stress occurs at a 1+θ
strain value of 0.002. i.e., 0.2%. The Where, θ = creep coefficient
value of stress at 0.002 strain is E c = short term modulus of elasticity
called compressive strength of θ = creep coefficient
concrete. ultimate creep strain
2) Lower strength concrete has great =
elastic strain at the age of loading
deformability i.e., ductility than high Age at loading Creep coefficient
strength concrete.
7 days 2.2
3) Descending part of high strength
28 days 1.6
concrete is sleeper.
1 year 1.1
4) High strength concrete gets crushed
at smaller strain.
1.6 CONCRETESTRAIN AT ULTIMATE
5) The point where curve ends is called
STRENGTH
crushing strain.
6) High strength concrete is more • If a concrete cylinder is axially loaded
brittle as compared to low strength the ultimate strength is at 0.2% strain.
concrete.

© Copyright Reserved by Gateflix.in No part of this material should be copied or reproduced without permission
• For flexure, crushing is assumed to
occur at 0.35% strain.
Note:- Actually it is seen that if the stress
strain distribution is

• Creep is thought to occur due to:


1) Internal movement of adsorbed
a) Rectangular, strain = 0.2% water
b) Triangular, strain = 0.35% 2) Viscous flow or sliding between gel
c) Trapezoidal, strain = 0.2 – 0.35% particles
3) Moisture loss
1.7 DESIGN STRESS-STRAIN CURVE 4) Growth in micro cracks

• Ascending part is taken as 2nd degree • Effect of creep are


parabola. 1) Increase in deflection of beams and
• 0.67 fck = Strength of concrete in slabs
structure 2) Increased deflection of slender
• 𝛾𝛾m = partial safety factor for material column that may lead to buckling
strength. 3) Gradual transfer of load from
𝛾𝛾m = 1.5 for limit state of collapse concrete to reinforcing steel in
𝛾𝛾m = 1.0 for limit state of serviceability comp. members
4) Loss of prestress in prestressed
concrete

• Beneficial effects of creep are:


1) Reduction in stress induced by non-
uniform load or restrained
shrinkage
2) In indeterminate structure, stress
1.8 SHRINKAGE & CREEP IN CONCRETE induced due to settlement of
support is reduced due to creep.
1.8.1 Creep
• Factors influencing creep
• When concrete is subjected to sustained Creep increase when:-
loading, its deformation keeps on (a) Cement content is high
increasing with time, even though the (b) W/c ratio is high
stress level is not altered. (c) Aggregate content is low
• Time dependent component of total (d) Air entrainment is high
strain is called creep. (e) Relative humidity is low
(f) Temperature (causing moisture loss)
is high
(g) Size/thickness of member is small
(h) Loading occurs at early age

© Copyright Reserved by Gateflix.in No part of this material should be copied or reproduced without permission
(i) Loading sustained over a long period Steam curing under pressure reduces
Creep coefficient for design drying shrinkage and moisture movement

As long as stress in concrete does not 1.8.2 SHRINKAGE


exceed one-third of its characteristic
strength, creep may be assumed to be The shortening in length of a member or
proportional to the applied stress. contraction of the concrete per unit length
Thus under service load condition, due to drying when concrete sets are
creep will be proportional to stress. known as shrinkage.
This concept can be used to compute Shrinkage can be classified as
total deflection (initial + creep) by usual a) Plastic shrinkage
linear elastic analysis with reduced b) Drying shrinkage
modulus of elasticity. The reduced
modulus of elasticity 1.8.2.1Plastic shrinkage
Ece = Ec/(1+θ)
Where Ece = reduced modulus of This type of shrinkage manifests itself soon
elasticity taking into account long term after the concrete is placed in the form
effect of creep while the concrete is still in plastic state.
E c = short term modulus of elasticity Loss of water by evaporation from the
θ = creep coefficient surface of concrete or by absorption by
aggregate or sub grade is believed to be the
reason for plastic-shrinkage.
As aggregate & steel restrain this effect,
cracks appear at the surface or internally
around aggregate or reinforcement. Thus
shrinkage is prevented by using aluminium
powder and expansive cement or shrinkage
compensating cement.
Age at loading θ
7 days 2.2
1.8.2.2Drying shrinkage
28 days 1.6
1 Year 1.1
• It is an everlasting process and occurs
mainly due to loss of water held in the
• Intermediate value of creep coefficient
gel pores when concrete is kept in
may be interpolated by assuming that
drying condition.
the creep coefficient decreases linearly
• The finer the gel, the more is the
with the log of time in days.
shrinkage.
Thus, creep coefficient for age of
• Harder aggregate leads to lower
loading at 15 days in
shrinkage but higher shrinkage stress.
0.6 [ log10 28-log1015]
θ15 = 1.6 + Reverse is true for softer aggregate.
[log10 28-log10 7] • Shrinkage decreases with increase in
i,e., θ = C - θ 0 log t size of member.
• Effect of creep can be reduced by • Concrete made with smaller size
1) Using high strength concrete aggregate shrinkage more than
2) Delaying the application of finishes, concrete made with larger size
partitions walls etc. aggregate.
3) Adding reinforcement • Shrinkage produces tensile cracks in
4) Steam curing under pressure. any member which is restrained.
Note:

© Copyright Reserved by Gateflix.in No part of this material should be copied or reproduced without permission
• For a given environment, the total
shrinkage depends more on the total
amount of water present in concrete at
the time of mixing and to a lesser extent
on the cement content.
• In the absence of test data, an apex
value of total shrinkage strain for
design may be taken as 0.0003
shrinkage decreases with increase in
relative humidity.
• ½ of total shrinkage assumed in 1st
month and 3/4th in 1st 6 months.

1.9 DESIGN STRESS STRAIN CURVE FOR


STEEL

γ m = Factor of safety
γ m = 1 for limit state of serviceability
= 1.15 for limit state of collapse
• Increase of stress in strain hardening
region is neglected
• In RCC steel never reaches its ultimate
strength because strain at which
ultimate strength is reaches will never
be reached.
• Concrete will get crushed before steel
reaches its ultimate strength.

Note:-Failure of RCC beam always occurs


due to crushing of concrete. It never occurs
due to failure of steel. (i.e., shaping of steel)

• We may use same curve for tension and


compression for both grades of steel
(Hot rolled or cold worked.) This may
lead to insignificant error. (in case of
compression)

© Copyright Reserved by Gateflix.in No part of this material should be copied or reproduced without permission
2 DESIGN OF RCC STRUCTURE

2.1 INTRODUCTION

The philosophy of the limit state method of


design represents a definite advancement
over the traditional design philosophies.
Unlike working stress method, which is
based on calculations at service load
conditions, alone, and unlike ultimate load
method, which is based on calculations on
ultimate load conditions alone, limit state
method aims for a comprehensive and
rational solution to the design problem, by
considering safety at ultimate load and
serviceability at working loads.
The limit state philosophy uses a partial
safety factor format which attempts to
provide adequate safety at ultimate loads 2.3 CHARACTERISTIC STRENGTH OF
as well as adequate serviceability at service MATERIAL
loads by considering all possible ‘limit state’.
The strength of material below which not
2.2 WHAT IS LIMIT STATE METHOD? more than 5% of the test results are
expected to fall is known as the
• The acceptable limit for the safety characteristic strength of the material and
serviceability requirement of a denoted by f ck .
structure or structural element before
fck= fm-1.65δ
failure occurs is called limit state. 2
• In this structure is so designed that it  x-x 
δ= ∑  
carry the loads with sufficient degree of  n-1 
safety and serviceability and structure
f m = Mean of strength
will not become unfit for which it is to
be designed. δ = Standard deviation
fck = Characteristics strength of material
x = Concrete cube test result
x = mean of concrete cube test result
N = No. of samples.

© Copyright Reserved by Gateflix.in No part of this material should be copied or reproduced without permission
(Fm-1.65δ) and (Fm+1.65δ) are two
important limit within which ‘probability of
lying test result’ is maximum. These limit
called confidence limit.

Table1.1: Specified characteristic


compressive strength of concrete at 28
days. Definition Curve For Characteristic Load

Grade Specified characteristic 2.5 PARTIAL SAFETY OF FACTOR


Designation compressive strength at 28
days N/mm2 In limit state method of design two factors
M10 10 of ms safety are used one to account for
M15 15 uncertainty in material property and other
M20 20
M25 25 for uncertainty in loading. Hence the
M30 30 factors are called partial factor of safety
M35 35 γ = Partial safety factor for material
M40 40 strength.
Note:- γmc for concrete = 1.5 (because of greater
1. In the designation of a concrete mix, variation is excepted in the strength of
letter M refers to the mix and the concrete than that in the steel
number to the specified characteristic reinforcement) γ ms for steel = 1.15
compressive strength of 15-cm cube at
28 days, expressed in N/mm2.
2. M5 and M7.5 grades of concrete may be 2.5.1 Materials
used for lean concrete bases and simple
foundation for masonry walls. These The value of partial safety factor for
mixes need not be designed. material strength should account for the
3. Grade of concrete lower than M20 shall following parameters.
not be used in reinforced concrete. (As a) Possibility of deviation of the strength
per IS 456 : 2000) of material.
4. For sea water grade of concrete lower b) Deviation of the sectional dimensions.
thanM30 shall not be used in reinforced c) Accuracy of the calculation procedure,
concrete. (As per IS 456: 2000) and
d) Risk to life and economic consequences.
2.4 CHARACTERISTIC LOAD
2.5.2 Loads:-
Value of load which has a 95% probability
of not being exceeded during the life of the The value of partial safety factor for load
structure is known as characteristic load should account for the following
and is denoted by F parameters:-
a) Unusual increase in loads beyond that
=
F Fm + 1.65δ
used for deriving characteristic values,
Fm = mean value of load b) Unforeseen stress redistribution,

© Copyright Reserved by Gateflix.in No part of this material should be copied or reproduced without permission
c) Inaccurate assessment of the effects of
loading and 2.6 ASSUMPTION LIMIT STATE OF
d) Importance of the limit state considered COLLAPSE: FLEXURE

Table 1.2: Value partial safety for loads 1. Plane Section before Bending
under various load combination Remains Plane even after the
Bending
No Description Collapse Service This assumption mean that strain at any
ability point on the cross-section is directly
1 D.L + L.L 1.5 1
proportional to its distance from its
2 D.L + (W.L) or
(E.L) combination neutral axis, it mean strain diagram is
linear.
(i) for normal case 2. The maximum strain in concrete at
D.L. + W.L (or E.L) 1.5 1 the outermost compression fiber is
taken as 0.0035 in bending.
(ii) For checking 0.9 1
stability against The maximum strain in concrete in
overturning/stres compression will be fiber AB, (1.1) i.e.,
s reversal D.L + in the Topmost fiber, and according to
W.L (or E.L) this assumption its value will be limited
3 (D.L) + L.L + W.L to 0.0035. Concrete has very low
(orE.L)
combination ductility and so it gets crushed in
D.L 1.2 1 compression at such low strain only.
L.L 1.2 0.8
W.L (or E.L) 1.2 0.8

Design values are obtained when partial


safety factor are applied to characteristic
load and materials.
3. Relationship between compressive
1. Materials:-The design strength of the stress distribution in concrete and
materials fd is given by strain in concrete may be assumed to
fd = fck/γmc be rectangular, Trapezoidal,
f ck = characteristic strength of the parabolic or any other shape which
material results in prediction of strength in
γmc = partial safety of factor for material substantial agreement with the
result of test.
2. Loads:- The design load Fd is given by An acceptable strain-stress curve is as
Fd = F γf shown below note that Characteristic
F = Characteristic load strength of concrete fck is reduced to
γ f = Partial safety of factor for load 2/3fck = 0.67 fck because of size effect.
Design compressive stress in concrete
may be taken as fd; where
Table1.3: Indian standard specifications
0.67f ck 0.67f ck
for loads. fd = = =0.45f ck
Characteristic Loads Indian Standard γ ms 1.5
Specification
Dead loads IS : 875 (Part 1) : 1987
Imposed loads IS : 875 (Part 2) : 1987
Wind loads IS : 875 (Part 2) : 1987
Seismic loads IS : 1893 : 1984

© Copyright Reserved by Gateflix.in No part of this material should be copied or reproduced without permission
From strain diagram (similar triangle)
0.0035 0.002
=
AC AB
0.002 2
AB =  AC = xu
0.0035 3.5
4
AB = x u
7
4
• For design purpose, the compressive BC = AC – AB = x u - x u
7
strength of concrete shall be assumed to
3
be0.67 times the characteristic strength BC = xu
and partial factor of safety γmc = 1.5 7
shall be applied in addition to this. Compression force of rectangular
• It may be noted that for the design of portion = Area of stress diagram ×
flexural members the characteristic width of the section
strength of concrete is taken as 0.67 f ck 3
C1 = 0.45f ck . x u b
instead of f ck . This is on account for the 7
fact that in actual structure size of C1 = 0.193 f ck x u b
concrete member may be more that the This force will act at a distance y1 =
cube size tested in laboratory. Larger 3
x u from the top.
size leads to more variability in 14
strength. Hence, strength of the Compression force of parabolic portion
concrete member will be lesser than = Area of stress diagram × Width of the
that in cube. However beyond 450 mm section
size cube the variation of strength is not 2
much. = base × Height × Width of the section
3
• The variation of strain-stress curve 2 4
shall be parabolic upto 0.002 strain and, ×0.45f ck . x u b
3 7
thereafter, the stress remains constant
up to the maximum permissible strain
C2 = 0.171f ck .x u .b
of0.0035. At limit state, this is an This will act at a distance
idealized curve for concrete in 3 3 4  3 12
y2 =  x u + × x u  = x u + x u =
compression and is valid for all grades 7 8 7  7 56
of concrete irrespective of percentage 24x u +12x u 36
of tensile reinforcement. = xu
56 56
9
y2= x u from top
14
This combined compressive force will
act at a y distance from top.
C.G. location of total force from top (y)
C y +C y
= 1 1 2 2
C1 +C2

• Design stress block parameter for the


above curve (Fig. 1.2) is calculated as
follows.

© Copyright Reserved by Gateflix.in No part of this material should be copied or reproduced without permission
3 9 • The strain-stress relationship for
0.193f ck .x u .b× x u +0.171f ck .x u . x u
14 14 steel in tension and in compression
0.193f ck .x u .b+0.171f ck .x u .b is assumed to be the same.
Original cross section area is defined
y = 0.42x u
as the area of cross section of steel
∴ Lever arm distance between centroid at zero stress.
of compressive force to centroid of Note:-
tensile force (z) is given by z= d − y 1. Design stress for steel=
z = d – 0.42x u Permessible stress f
= y = 0.87 fy
Total compressive force Partial safety factor 1.15
C = C1 + C2 = 0.193f ck .x u .b + 0.171f ck .x u .b For Fe 250 = 0.87 × 250 = 257.0
C = 0.36f ck .x u .b N/mm2.

2. Total strain at yield (∈) for mild steel


4. The tensile strength of concrete is
ignored. fy 250
will be = = 0.00125; whereas
The cracked concrete does not E s 2 ×105
contribute towards enhancement of total strain at design yield stress,
moment of fy 250
Resistance of the section, except ∈yd = = = 0.00109.
γ ms E s 1.15 × 2 ×105
providing bond between concrete and
steel for developing tensile strain and • In a similar manner total strains ∈ '
thus stress in steel. and ε’d’(design strain) at any stress,
5. For design purpose partial factor of say 0.9 fy and 0.9 fyd’ cold-worked
safety for steel γ ms = 1.15 and the deformed bars of grade Fe 415 can
be evaluated as under(from the fig
stress in steel are derived from
1.3b) :
stress-strain curve.
0.9f y
• A nominal strain-stress curve for =∈' + 0.0003
mild steel bars from test may be Es
obtained a shown in Fig 1.6(a). The 0.9×415
= +0.0003.=0.0022
strain is obtained by dividing 2×105
change in length by actual length; Where 0.0003 is the inelastic strain
whereas stress is determined as at 0.9 fy (figure 1.3b) and
load divided by original cross 0.9f y 0.9 × 415
sectional area*. ∈ 'd= +0.0003.=
γ ms E s 1.15 × 2 ×105
• This strain-stress curve is simplified
to a curve as shown in Fig. 1.5(a). +0.0003 = 0.00192
• Similarly the strain-stress curve for • In a flexural member, where
cold worked deformed bars of Grade compression reinforcement is
Fe 415 and Fe 500 which are, provided, the strain at the c.g. of
generally, in use in India is as shown compression reinforcement is
in Fig. 1.6(b). calculated from strain diagram and
This is idealized to a curve of Fig. the corresponding stress is
1.5(b). determined as explained above.
• For all type of steel, modulus of • For ready reference, typical points
elasticity, E = 2 × 105 N/mm2. on design strain-stress curves for all
types of steel bars are given in Table
1.4.

© Copyright Reserved by Gateflix.in No part of this material should be copied or reproduced without permission
Table 1.4:- Typical points on the design (Fe 250). Thus a strain of 0.0031
strain-stress curves. will ensure sufficient ductility in
mild steel reinforcement besides
Stress For fy = 250 For fy = 415 MPa For fy = 500 MPa causing the stress equal to 0.87
level MPa
Strain Stress Strain Stress Strain Stress fy.
0.80fyd 0.00087 173.9 0.00144 288.7 0.00174 347.8 This assumption thus,
0.85fyd 0.00093 184.8 0.00163 306.7 0.00195 369.6
0.90fyd 0.00098 195.7 0.00192 324.8 0.00226 391.3 restricts the depth of neutral
0.95fyd 0.00104 206.5 0.00241 342.8 0.00277 413.0 axis.
0.975fyd 0.00106 212.0 0.00276 351.8 0.00312 423.9
1.0 fyd 0.00109 217.4 0.00380 360.9 0.00417 434.8
2.7 WORKING STRESS METHOD
6. Maximum strain in Tension
reinforcement in the section at • This was the traditional method of
failure shall not design.
be less than • Both concrete and steel are assumed to
fy 0.87f y behave in linearly elastic manner.
∈st = +0.002= +0.0020 • Stresses within the materials are not
1.15E s Es allowed to exceed the permissible
The above value of strains for different stresses.
grades of steel as follows: • Working stress method of design used
i) For Fe250, to be the basis of design for all RCC
εst = [250/(1.15⨉2⨉105)]+0.002 or structures in the past. But these days it
εst = 0.0031 finds application in calculating
ii) For Fe 415 serviceability requirement like
εst = [415/(1.15⨉2⨉105)]+0.002 deflection and crack width under
or εst = 0.0038 and service load condition.
iii) For Fe 500, • It is also used in the design of few
εst = [500/ (1.15⨉2⨉105)] +0.002 structures like liquid retaining
or εst = 0.0042 structures and highway bridges and
chimney.

2.8 DEFICIENCY IN WORKING STRESS


METHOD

• It may not be possible to keep the stress


with in permissible stress. This is
because of
iv) Minimum strains at collapse a) Term effect of shrinkage and creep
ensures that the design stress in b) Effect of stress concentration and
tensile steel at collapse will other secondary effect.
always be fyd. (0.87fy) All such effects result in significant local
v) Note that for Fe 250, the strain is increase in stresses into inelastic range
0.0031 is much larger than that and redistribution of the calculated
required for development of stress.
stress equal to 0.87fy. In mild • In working stress method actual margin
steel the strain at which stress is of safety is not equal to the factor of
0.87f y safety used in WSM because the stress
0.87 fy is = 0.00109 but
Es strain curve is not linear upto collapse.
the code does not give separate Actual margin of safety here is given in
recommendation for mild steel

© Copyright Reserved by Gateflix.in No part of this material should be copied or reproduced without permission
variability regarding its strength and
Grade of Direct Bending Direct Permissible bond properties than steel which is produced
concrete tension compression compression
stress in Tbd for under well controlled conditions.
σ td σ abc σ cc plain bars in • The factor of safety for flexural
(N/mm2) (N/mm2) (N/mm2) tension (N/mm2) compressive in concrete is = 3. Thus,
M 20 2.8 7.0 5.0 0.8
M 25 3.2 8.5 6.0 0.9 the permissible compressive stress in
M 30 3.6 10.0 8.0 1.0 concrete in flexural compression is
N 35 4.0 11.5 9.0 1.1
M 40 4.4 13.0 10.0 1.2 σ abc =0.333f ck .
collapse load
term of factor lie the F.O.S.
working load
characterstic stress 2.10 PERMISSIBLE STRESSES IN
on the other hand is CONCRETE
permissible stress
• WSM fails to discriminate between
• The permissible stress of concrete in
different types of loads that act
direct tension is denoted by σ td . The
simultaneously, but have different
degrees of uncertainty. This may values of σ td for member in direct
sometimes lead to significantly tension for different grades of concrete
conservative design particularly when are given in the table below.
two different loads have counteracting • It may be worth nothing that the factor
effect. Example, if dead load and wind of safety of concrete in direct tension is
load produce counteracting stress but if from 8.5 to 9.5.
they are simply added, the design load • The permissible stresses of concrete in
would be much larger. bending compression σ cbc' in direct
compression σ cc' and the permissible
2.9 PERMISSIBLE STRESSES
stress in bond for plain bars in tension
• In working stress method, the stresses Tbd are given in table 21 of IS 456 for
in materials are not exceeded beyond different grades of concrete which is
their permissible values. The presented in table below.
permissible stress in a material is given • For plain bars in compression, the
by values of bond stress are obtained by
Permissible stress =
Limiting strengh increasing the respective value in
Factor of safety tension by 25 percent, as given in the
• In case of steel reinforcement, the table below and
limiting strength is either the yield • For deformed bars in tension the
stress or 0.2% proof stress, as the case values bond stress given in table are to
may be. For concrete, the limiting be increased by 60%.
strength is the crushing strength in
compression. 2.11 PERMISSIBLE STRESS IN STEEL
• The factor of safety in the case of tensile REINFORCEMENT
steel reinforcement is approximately =
1.82. • Permissible stresses in steel
Hence, the permissible tensile stress in reinforcement for different grades of
steel is σst =0.55f y . steel, diameters of bars and the types of
stress in steel reinforcement are given
• For concrete, the factor of safety is in table IS 456.
higher than for steel. This is so because
• Selective values of permissible stress of
concrete suffers from higher degree of
steel of grade Fe 250 (mild steel) and Fe

© Copyright Reserved by Gateflix.in No part of this material should be copied or reproduced without permission
415. (high yield) strength deformed In case of steel reinforcement of small
bars) in tension ( σst or σss ) and diameter, the stress will be uniform for
direct stress as well as for bending stress.
compression in column ( σsc ) are
Therefore in steel bars, the permissible
furnished in Table 1.6 as a ready stresses in bending and direct stresses re
reference. the same for lower dia bars up to 20mm φ .
For more than20 mm, permissible tensile
Table:- 1.6:- Permissible stress in steel stress is usually reduced.
reinforcement
Type of stress in steel Mild steel High yield
reinforcement bars, Fe 250, strength Note:-
(N/mm2) deformed bars
Fe 415, (N/mm2) The value of σst is given at the centroid of
Tension σ st or σ ss 140 230 tension reinforcement subjected to the
(a) Up to and including 20 130 230 condition that when more than one layer of
mm diameter tensile reinforcement is provided, stress at
(b) Over 20 mm diameter
Compression in column 130 190 the centroid of outer most layer shall not
bars σsc exceed by more than10% that given in the
above table.
Note:-
Effect Wind and Earthquake on the
permissible stress:-

• When the effect of wind or earthquake


is taken into account, the permissible
values for stresses in steel and concrete
1
are increased by 33 %
3

It can be observed that for a given grade of


concrete σ abc <σ cbc i.e. a greater F.O.S. is
adopted for direct stress than for a
bending stress. This is because when a c/s
is subjected to bending stress, the stress
induced on it is variable being maximum at
extreme fibre and zero at N.A.
When the maximum stress exceeds the
permissible value, the extreme fibre will
not fail actually, but will transfer the
additional force to the inner fibre which
has a lower stress. However, when the
section is subjected to a direct stress, all
points of the section have uniform stress
having no scope for such a transfer of the
force.
It is for this reason larger F.O.S. is adopted
for direct stress than bending stress.

Note:-

© Copyright Reserved by Gateflix.in No part of this material should be copied or reproduced without permission
3 CONCRETE TECHNOLOGY

3.1 INTRODUCTION
Concrete a composite man-made material,
is the most widely used building material in
the construction industry. It consists of a
rationally chosen mixture of binding
material such as lime or cement, well grade
fine and coarse aggregates, water and
admixtures (to produce concrete with
special properties.)
• The good concrete are which fulfills two
criteria :-
a) The concrete has to be satisfactory
in its hardened state, and
b) Also in its fresh state while being
transported from the mixer and
placed in the formwork. In this chapter we will discuss the
• The requirements in the fresh state are constituent material of concrete as
that the consistency of the mix be such discussed in above flow diagram of
that it can be compacted by the means concrete one by one.
desired without excessive effort, and 3.1.1 Cement
also that the mix be cohesive enough for
the methods of transporting and placing Cement is a material which has cohesive
used so as not produce segregation with and adhesive properties in the presence of
a consequent lack of homogeneity of the water.
finished product.
3.1.2 Manufacturing Process:
• The primary requirements of good
concrete in its hardened state are a 1. Portland cement is manufactured by
satisfactory compressive strength and grinding together 1 part of calcareous
an adequate durability. (CaCO3) and one part of argillaceous
The flow of this chapter is as per the flow (clay and shales). (Dry process)
diagram described below 2. The mixture is burnt in a kiln at a
temperature of about 1300°C to 1500°C
where material use and form a small
Clinkers of 3 mm to 20 mm in size.
3. Then the clinker is cooled down using
moderate cooling condition (It is
observed that slow or quick cooling
condition reduced the strength of the
cement.)
4. In moderate cooling condition
temperature of clinker is brought down

© Copyright Reserved by Gateflix.in No part of this material should be copied or reproduced without permission
to 500°C in 15 min and in further Fe2O5 Gives colour and helps in 0.5 – 6
10 minute the temperature is brought, fusion of different
ingredients
down to atmospheric.
MgO Imparts colour and 0.5 – 4
5. Cooled, clinker is then mix with hardness. If in excess, it
gypsum. causes cracks in mortar
6. The mixture is ground to required and concrete and
fineness in ball mills to get the final unsoundness.
Na2O + These are residues, and 0.5 – 1.3
product known as cement.
K2O if in excess cause 0.1 – 0.4
7. Gypsum is required to retard the setting TiO2 efflorescence and 0.1 - 0.2
time. P2O5 cracking
SO3 Makes cement sound 1– 2
3.2 CHEMICAL COMPOSITION OF RAW
MATERIALS Notes:-
1. The rate of setting of cement paste is
• The three constituents of hydraulic controlled by regulating the ratio
cements are lime, silica and alumina.
SiO 2 / ( Al2 O3 + Fe 2 O3 ) .
• In addition, most cements contain small
proportions of iron oxide, magnesia, 2. Where development of much heat of
sulphur trioxide and alkalis. hydration is undesirable, the silica
• There has been a change in the content is increased to about 21
composition of Portland cement over percent, and the alumina and iron oxide
the years, mainly reflected in the contents are limited to 6 percent each.
increase in lime cement and in a slight 3. Resistance to the action of sulphate
decrease in silica content. waters is increased by raising further
• An increase in lime content beyond a the silica content to 24 percent and
certain value makes it difficult to reducing the alumina and iron contents
combine completely with other in 4 percent each.
compounds. 4. The variation in composition depends
• Consequently, free lime will exist in the largely on the ration of CaO to SiO2 in
clinker and will result in an unsound the raw materials.
cement. An increase in silica content at
the expense of alumina and ferric oxide 3.3 TYPES OF CEMENTS
makes the cement difficult to fuse and
form clinker. 1. Ordinary Portland Cements:-
The approximate limits of chemical • It is widely used cement for most of
composition in cement are given in the work. It is suitable for general
table. construction works when there is no
exposure to sulphates in the soil or
Constituents of Portland Cement (Raw in the ground water.
Material) • Three different grade of ordinary
Oxide Function Composition
Portland cements to produce high
(%) strength cement is
CaO Controls strength and 60-65 33 grade of O.P.C.
soundness. Its deficiency 43 grade of O.P.C.
reduces strength and 53 grade of O.P.C.
setting time.
SiO2 Gives strength. Excess of 17 – 25
it causes slow setting. 2. Rapid hardening Cements:-
Al2O5 Responsible for quick 3–8 • (Name itself shows that it hardened
setting, if In excess, it and attains its strength earlier than
lowers the strength ordinary Portland cement).

© Copyright Reserved by Gateflix.in No part of this material should be copied or reproduced without permission
• Rapid hardening cement is finer • It has low heat of hydration and is
than ordinary Portland cement and better resistant to soil and water
it contains more C3S and less C2S containing excessive amounts of
than the OPC sulphates, alkalies.
• 3 days strength of RHC is same of 7 • Because of its low heat evolution it
days strength of OPC can be used in mass concrete
• The main advantage of rapid structure such as dams &
hardening cement is that shuttering foundations.
may be removed much earlier, thus
saving considerable time and 5. Pozzolana Cements:-
expenses. • This is manufactured by grinding
• Rapid hardening cement is also used Portland cement clinker with
for road work where it is necessary Pozzolana and required quantity of
to open the road traffic with the gypsum. (The pozzolana are
minimum delay. materials which at ordinary
temperature react with lime in
3. Extra Rapid Hardening Cement:- presence of water, resulting in
• In this cement 2% Cacl2 is mixed cementing materials.) Fly ash, burnt
with rapid hardening cements. clay are used as Pozzolana.
• This Cacl2 imparts quick setting • The proportion of Pozzolana may be
properties. 10-25% by weight of Pozzolana
• While using this cement maximum cement.
time of 20 minute is available for • This cement has higher resistance to
mixing, transporting and placing the chemical agencies and to sea water
concrete and also this cement is because of absence of lime.
used within one month. • Advantage of this cement are
• Extra rapid hardening cement used reduction in cost, increased
in cold weather because of large impermeability.
heat of evolution.
6. High Alumina Cement:-
4. Portland Blast Furnace Cement:- • It is non-Portland cement, it is
• It is manufactured by mixing manufacture by Melting mixture of
Portland cement clinker with aluminous and Calcareous Materials
granulated blast furnace slag (This in Suitable proportion and grinding
slag is waste product obtained from the resulting clinker to fine powder
blast furnace which contains oxide (which is black in color).
of lime, silica and alumina) and • The raw materials used for its
gypsum in suitable proportions and manufacturing are chalk and bauxite
grinding the mixture to the required which is special clay of extremely
fineness. high alumina content.
• The proportion of the slag should • It is used where early removal of
not less than 25% and not more form work is required.
than 65% of the total mass of the • Its rapid hardening properties arise
mixture. from the presence of Calcium
• It contains approximately 45% CaO aluminate like, Calcium silicates in
and 35% silica. Portland cement.
• It is similar and cheaper than O.P.C • It’s one day strength is equal to 28
(and it can be replace it.) days strength of ordinary Portland
cement.

© Copyright Reserved by Gateflix.in No part of this material should be copied or reproduced without permission
• It is recommended for sea and Sulpho-aluminates which have
under water work. expansive properties and so causes
disintegration of concrete (Due to
Table 10.3 Composition of typical this nature it expand and this larger
high alumina cement volume will create pressure on
concrete which will result in cracks
Composition (%) and finally disintegration of
Al2O3, TiO2 43.5 concrete.)
Fe2O3, FeO, Fe3O4 13.1 • It is strongly recommended for
CaO 37.5
structure in sea water, coastal areas
SiO2 3.8
MgO 0.3 and Marshy lands.
SO3 0.4 • This is used in canal.
Insoluble material 1.2
Loss on ignition 0.2 9. Super Sulphated Cement:-
• It is manufactured by grinding
7. Low Heat Portland Cement:- together a mixture of 80 – 85% of
• It is manufactured by reducing the granulated blast furnace slag and 10
% of C3S and C3A of ordinary – 15% of calcium sulphate and
Portland cement, because of this about 5% of Portland cement
cement gets the strength at a slower clinker.
rate and the heat of hydration is • The cement is highly resistant to
less. This will require long time sulphate attack.
curing and keeping forms for a long • Because of low heat of evolution it is
time. useful for mass concrete works. It is
• It is used in Mass Concrete Works, also useful for foundation works
such as dams and retaining walls. where aggressive chemical
(For mass construction work when conditions exist.
OPC is used temperature may rises
at its highest level when time passes 10. Quick Setting Portland Cement:-
outer layer will cool and contract • The quantity of gypsum is reduced
while the inner mass is still a higher and small percentage of aluminium
temp. This may leads to serious sulphate is added. It is ground much
cracking). finer that ordinary Portland cement.
• Heat generated in OPC at the end of Properties :-
3 days –80cal/gm. While in low heat Initial setting time – 5 minutes
cement it is about 50 cal/gm of Final setting time = 30 minutes
cement. Use :-
It is used when concrete is to be laid
8. Sulphate resisting cement:- under water or in running water.
• It is similar to ordinary Portland
cement except that it contains more 3.4 ADMIXTURE
silicates and less quantity of
aluminates. Suitable materials known as admixtures
• It is used for under water structure which may be added to concrete mix, Just
particularly exposed to alkali before or during the mixing to modify one
actions. or more properties of the concrete in the
• Soluble sulphates like MgSO4, CaSO4 plastic or hardened states as desired.
and Na2SO4 when present in ground The objective of admixture are :-
reacts with cement and form

© Copyright Reserved by Gateflix.in No part of this material should be copied or reproduced without permission
• To increase the rate of strength • A small quantity of sugar about 0.05
development at early ages. percent of the mass of cement will
• To retard the initial setting times. act as an acceptable retarder: the
• To increase the workability without delay in setting of concrete is about
changing the water content. 4 hours.
• To increase the resistance to freezing • A large quantity of sugar, say 0.2 to
and thawing, vinsol resins and air 1 percent of the mass of cement, will
entertainment admixture which is used virtually prevent the setting of
for this purpose which cause air to be cement.
incorporated in the form of minute tiny • When sugar is used as a controlled
bubbles in concrete during mixing to set retarder, the early strength of
increase the workability and resistance concrete is severely reduced but
to freezing and thawing. beyond about 7 days, there is an
increase in strength of several
3.5 DIFFERENT TYPE OF ADMIXTURE percent compared with a non-
retarded mix. This is probably due
1. Mineral Admixtures to the fact that delayed setting
a) Fly ash produces a denser hydrated cement
• This increase impermeability gel.
because it is finer than the cement
particles. Note:-
• This is by product of wooden Retarders tend to increase the plastic
thermal power plant and is shrinkage because the duration of the
produced daily in large quantities. plastic stage is extended, but drying
• These require very less water to wet shrinkage is not affected.
their surface.
• Same work which is obtained by c) Water Reducing Admixture:
OPC is achieved by lower A substance which either increases
water/cement ratio. workability of freshly mixed mortar or
concrete without increasing water-
2. Chemical Admixtures cement ratio or maintains workability
a) Accelerating Admixture:- with reduced water-cement ratio. These
A substance which increases the rate of are:
strength development or reduces the • Lignosulphonic acid and its salts.
setting time. CaCl2, Sodium silicate, • Formaldehyde derivatives
Sodium chloride, Calcium nitrite and • Hydroxylated carboxylic acids
calcium nitrate are used as accelerator. • Calcium lignosulphonate
• Barium chloride acts as an
accelerator only under warm d) Air-entraining Admixture :-
condition A substance which causes air to be
• Most frequently used accelerator is entrapped in the form of tiny bubbles in
calcium chloride (CaCl2) motor or concrete, during mixing to
increasing its workability and
b) Retarding Admixture:- resistance to freezing and thawing.
A substance which delays the setting Example: Vinsol resin, using of
time of cement paste. Gypsum, Aluminum powder.
Tartaric acid, sugar are used as
retarding admixture. e) Super Plasticizing Admixture :-

© Copyright Reserved by Gateflix.in No part of this material should be copied or reproduced without permission
• A substance which imports very high • The Rounded particle produce
workability with a large decrease in smoother mix for a given water/cement
water content (at least 20%) for a given ratio.
workability. • On the other hand, angular or flaky
The resulting improvement in particles reduce the workability and
workability can be exploited in two demand more cement and water to give
ways. By the specified strength of concrete mix.
a) By producing concrete with a very
high workability or concrete with a • Not more than 10 – 15% of flaky
very high strength. particles should be used in concrete.
b) Concrete of normal workability but
with an extremely high strength 3.6.1 Bulking of Sand
owing to a very substantial
reduction in the water/cement ratio. Due to the presence of moisture content,
• A high range water reducing admixture aggregates bulk in volume. The moisture
is also referred to as a super plasticizer particles form a thin film around the
and these are sulphonated melamine aggregates and exert surface tension. This
formaldehyde condensates; sulfonated keeps the particles away from each other
naphthalene-formaldehyde and thus aggregates bulk in volume.
condensates; modified lignosulfonates; • This bulking in volume generally
and other such as sulfonic-acid esters negligible in the case of course
and carbohydrate esters. aggregates. It has great importance in
• The role of super plasticizer is to case of fine aggregates or sand.
disperse, the particles remove air • For sand the volume goes on increasing
bubbles, example; Sulphonated melanin until the moisture content is about 8%
formaldehyde and to retard setting. by the mass of sand.
• Bulking increases with fineness of the
3.6 AGGREGATES aggregates.
• The bulking of sand may be as large as
The aggregates consist of about 75% of 30 – 40%
volume of concrete and they greatly • With further addition of moisture
influence the properties of concrete. content the thin film of water coated
• Aggregate give body to the concrete round the sand start disappearing and
reduce the shrinkage effect of cement the volume of sand begins to decrease
and make the concrete durable. till finally at 25 – 30% of moisture
• Generally rounded shape aggregate is content. The volume of sand returns to
used because shape of aggregate affects its original volume when it is dry.
the workability of concrete.
• To achieve the best possible strength,
concrete should be as dense as possible
i.e. it should contain minimum void.
Voids are greatly influenced by the
shape of aggregate.
• The rounded particles can be packed to
produce a concrete with 33% void
means 67% of the volume of concrete is
occupied by the aggregate.

© Copyright Reserved by Gateflix.in No part of this material should be copied or reproduced without permission
3.6.2 Fineness Modulus mg/l for reinforced
concrete work
In order to ensure the presence of all sizes Suspended matter 2000 mg/l
of particles, the property of aggregate
called fineness modulus is defined. The 3.8 CONCRETE
fineness modulus of an aggregate is an
index number which is roughly Concrete is a carefully proportioned
proportional to the average size of the mixture of cement, fine aggregate, coarse
particles in the aggregate. aggregate and water. Sometimes to modify
the physical properties of concrete a
3.6.3 Fineness Modulus for Different variety of admixture may be added. The
Type of Sand preparation of concrete consists of the
following operations:-
Fine sand = 2.2 – 2.6 1. Proportioning of ingredients
Medium sand = 2.6 – 2.9 2. Measurement of materials
Coarse sand = 2.9 – 3.2 3. Mixing and placing of concrete
4. Compaction
Sand having fineness modulus more than 5. Curing.
3.2 will be unsuitable for making
satisfactory concrete. 1) Proportioning of Ingredient
The fineness modulus is the sum divided by Proportioning of ingredients means
100 of the cumulative percentage mass determining the relative amounts of
which is retained on each of the ten sieves ingredients (cement, FA, CA to get the
specified by I.S. code. required strength of concrete.)
Sieves are 150mμ, 300m µ , 600m µ , This can be done by two way:
1.18mm, 2.36 mm, 4.75 mm, 10 mm, 20 i) Design Mix.
mm, 40 mm, 80 mm and larger is required ii) Nominal Mix.
increasing in the ration of 2 : 1
i) Design Mix:-in design mix the
Note:- Fineness modules of 4.00 can be proportion of ingredients of
interpreted to mean that the fourth sieve, concrete to obtain a desired
i.e. the average size of particle is 1.18 mm. strength can be found out by
laboratory method. Concrete grades
Water > M 20 are design mix concrete.

Water is required for proper chemical ii) Nominal Mix:- In this method
action and amount of water required is proportion of ingredients of
about (25 – 35%) of the weight of cement concrete can be choose on the
used. Water/cement = 0.4 arbitrary method M 5, M7.5 M 10
and M 20 are the nominal mix
3.7 PERMISSIBLEL LIMITS FOR
IMPURITIES IN WATER 3.9 METHODS OF PROPORTIONAL
CONCRETE
Impurity Permissible Limits
Organic 200 mg/l 1) Maximum Density Method
Inorganic 3000 mg/l It is used as a theoretical approach to
Sulphates 400 mg/l determine the grading of aggregate of
Chlorides 2000mg/l for plain obtained maximum density.
concrete work 500 It is given by Fuller’s

© Copyright Reserved by Gateflix.in No part of this material should be copied or reproduced without permission
d agents may be mixed to increase the
Pd =100 workability of the mix.
D
• For such concrete, therefore, water-
d = Maximum size of fine aggregate.
cement ratio is reduced; resulting is an
D = Maximum size of Coarse aggregate.
increase in the strength.
Pd = % by weight of particle size finer
than d in total mixture.

3.10 WATER CEMENT RATIO

• Water cement ratio is the ratio of


volume of water mixed in concrete to
volume of cement used.
• The strength and workability of
concrete depends to a great extent on
the amount of water used.
• For a given proportion of the materials,
there is an amount of water which gives
the greatest strength.
• Amount of water less than this
optimum water decreases the strength
and about 10percent less be insufficient
to ensure complete setting of cement.
Similarly, more than the optimum water
increases the workability but decreases
the strength.
• The use of an excessive amount of
water not only produces low strength
but increases shrinking and decreases
density and durability.
• According to Abram’s water cement
ratio law, for any given conditions of
test, the strength of a workable
concrete mix is dependent only on the
water-cement ratio.
• Lesser the water-cement ratio in a
workable mix, greater will be its
strength.
• From Abram’s law, it follows that
provided the concrete is fully
compacted, the strength is not affected
by aggregate shape, type or surface
texture, or the aggregate grading, the
workability and the richness of the mix.
• Concrete vibrated by efficient
mechanical vibrators requires less
water-cement ratio, and hence have
more strength. Sometimes, plasticising

© Copyright Reserved by Gateflix.in No part of this material should be copied or reproduced without permission
4 REINFORCED SECTION

4.1 BALANCED, UNDER-REINFORCED 4.1.3 When the section is under


AND OVER-REINFORCED SECTION reinforced

4.1.1 Balanced Section a) x < x cr


b) Tension steel reaches the max.
It is a section in which both steel and Permissible value prior to concrete.
concrete reaches their maximum c) Area of tension steel is less than that in
permissible value simultaneously. i.e., the balanced section
when f st = σ st (permissible tensile stress in
steel), f cbc = σ cbc (permissible compression
stress in concrete)
Under this condition
1 1
N=n 0 = n0 =
σst 3σ
1+ 1+ st
mσcbc 280
Note
Neutral axis depth coefficient under d) In under reinforced section failure is
balanced condition depends only upon tensile failure. Under failure condition,
stress in tension steel. in under reinforced section, neutral axis
If Ast bal = Area of tension steel under is above balanced section N.A. Hence
balanced condition strain in steel increase leading to wider
A σ and deeper tension cracks and
Then p 0 = stbal = cbc η0
bd 2σst increased beam curvature and
n deflections. The process continues until
J 0 =1- 0 the max. strain in concrete in comp. side
3
reaches the ultimate compressive strain
of concrete resulting in crushing of
4.1.2 Under reinforced and over concrete. This type of failure is called
reinforce section tension failure. Note that moment of
resistance beyond moment of
resistance corresponding to yielding of
tension steel (i.e., Point A) does not
increase significantly, but curvature
increase significantly.
• Under reinforced condition, section
suffers large deflection and cracking
prior to failure. Thus it is a ductile
failure.
• Ductile failure gives sufficient
warning before failure hence
advisable.

© Copyright Reserved by Gateflix.in No part of this material should be copied or reproduced without permission
4.1.4 When section is over reinforced  x
M R w = ( σst .A st )  d- 
 3
a) x>x cr
1  x 
b) Concrete reaches its max. permissible M R W =  f cbc ×x.b   d- 
2  3 
value prior to steel.
c) A st >A st bal  σ  x 
Where, f cbc =  st   
d) Over reinforced section fails to utilize  m   d-x 
the full strength of costlier material i.e., x can be calculated from bx2/2
steel. In the over reinforced section, = mAst (d – x)
N.A. depth is large hence strain across
the section remains low. Consequently, For balanced section:-
the curvature, deflection and crack fst = σst ; fcbc=σcbc
width also remain low. σcbc  x  σcbc  x 0 
M RWO = x 0 b  d- 0  =  
Thus failure is sudden (without any sign 2  3  2  d 
of warning). This type of failure is called  x0  2
brittle failure. 1 −  .bd
 3d 
• As there is no significant warning
1
before failure. IS code not permit over- M rwo = n 0 J 0 σ cbc .bd 2
reinforced section design. 2
M RWO =R w bd 2 ,
Note:- Other types of failure besides
tension and brittle failure are 1
where R w = n 0 J 0 σ cbc
2
• Failure due to snapping of tension For over reinforced section:
reinforcement which occurs when there [f cbc =σcbc ,fs t <σst ]
is extremely low amount of 1  x 
reinforcement provided. But this M RW =M RW =  σ cbc bx   d- 
situation is rather rare, we safeguard 2  3 
against this by providing minimum  x f  d-x 
M RW = ( f st A st )  d-  Where st =   .σ cbc
tension reinforcement.  3 m  x 
• Failure due to dynamic loading
• Minimum tensile steel in beams is given The following table must be
by remembered as a ready reference
A st min 0.85
= Grade of Steel Fe 250 Fe 415
bd fy
σst N/mm2 [for φ < 140 230
• Maximum tensile steel in beams is given 130* 230*
20 mm]
by
[for φ > 20 mm]*
A st max
=0.04 n0 0.4 0.2887
bD 0.4179* 0.2887*
J0 0.867 0.904
4.2 MOMENT OF RESISTANCE 0.860* 0.904*
CALCULATION σcbc η0 0.143 0.063
=
P0 × σcbc σcbc
2 σst
For under reinforcement Percentage tensile 0.160 0.063
[fst =σst ,f cbc <σcbc ]: reinforcement in σcbc * σcbc *
fractions

© Copyright Reserved by Gateflix.in No part of this material should be copied or reproduced without permission
1 0.1734 0.130 σcbc M
=
Rw n 0 J 0 σcbc σcbc d≥
2 0.130 σcbc * Rwb
Rw = 0.179
σcbc * b) (width of section) could be chosen as
115 mm, 230mm, 250mm, 300mm, 350 for
economy of form work
4.2.1 Relation between percentage
reinforcement & moment of resistance  x 
M = A st .σst  d-  
M RWO = Moment of resistance for balanced  3 
section bx 2 
= mA st ( d-x ) 
• For p < p0 , MOR increases rapidly and 2 
nearly proportional to p. Hence OA line Simultaneous solution of these two will
can be approximated to be a straight yield the value of A st
line
• Rate of gain of MOR increases with use Note: -
of higher grade steel A st can be approximately calculated as
• Balanced section is reached at smaller M
percentage steel with use of higher =A st
σst J 0 d
grade steel. For p>po, the rate of gain of
MOR with increase in percentage steel Where = J 0 lever arm coefficient
drops off rapidly b) If the size of beam is given, find out
moment of resistance of balanced section
i.e.,
(i) If applied moment M < M RWO , area of
steel can be found out by the
simultaneous solution of equation (a)
and (b) as given below
 x
M=A st .σst  d-  . . . . . . . . . (a)
• Most economical utilization of steel (in  3
WSM) occurs in under reinforced bx 2
= mA st ( d − x ) . . . . . . . . . . (b)
design or balanced design. In over 2
reinforced design, steel is not (ii) If applied moment > (MOR) balanced
economically utilized. section simultaneous solution of (a) and
(b) will lead to over-reinforced design,
Note:- A section which is over reinforced in which is not recommended. Hence we
WSM may be under reinforced in LSM. will design doubly reinforced section.
Except when the percentage reinforcement • Provision of doubly reinforced
is large. section ensures ductile failure.
• Doubly reinforced section is the one
4.2.2 Design of section for a given in which compression reinforced is
moment provided in addition to tension
reinforcement.
a) If section dimension, are not given, fix
the depth and width of section such that Note:- Under reinforced or balanced design
M ≤ ( MOR )balanced section , is ensured by checking that percentage
Where M = maximum applied BM steel of the designed section
i.e., M ≤ R w bd 2

© Copyright Reserved by Gateflix.in No part of this material should be copied or reproduced without permission
 A st  in some special cases it can be
 ×100  ≤ P0 (percentage tensile advantageous.
 bd 
reinforcement) • Advantage of compression steel
Where, a) It permits smaller size which look
σ aesthetic
P0 = cbc ×n 0 ×100 b) It reduces the long term deflection
2σst
and increases ductility of the beam
c) It can be used as anchor bar for
4.3 DOUBLY REINFORCED SECTION positioning the shear reinforcement
d) As the compression reinforcement
Whenever the size of beam is restricted increases ductility of beam, they are
and beam has to bear higher value of B.M. provided (even when not required
than the moment of Resistance of the for strength) in the seismic zone to
( M.R.balance ) balance section of given beam, with stand repeated reversals
then to resist this higher moment doubly produces.
reinforced section is to be provided.
There are two methods to design such Note:- In tension steel, shrinkage reduces
beams: the tensile stress and creep produces
1) Increase the concrete mix to increase additional tensile stress but in compressive
the capacity of the section steel both shrinkage and creep add
2) Reinforcement are provided in additional stress so we have to use
compression zone to give additional different value of modular ratio (m).
strength to the concrete in In case of Doubly reinforced section stress
compression. Such beam are called in steel on tension side = m × stress in
double reinforced beams. surrounding concrete. But as per IS456, for
• A concrete structure when loaded steel in compression zone, Stress in steel =
undergoes elastic as well as plastic 1.5m × stress in surrounding concrete =
deformation. 1.5m × f cbc .
• The plastic deformation is termed as
creep. 4.4 ANALYSIS OF DOUBLY
• Elastic deformation is an instantaneous REINFORCED SECTION: (RECTANGULAR)
process while creep is a long process.
• The creep deformation of concrete
produces additional strain in
compression steel and gradually raises
the level of stress.
• To account for this increases in stress,
the modular ratio is increased.
A sc = Area of compression steel
• IS 456 states that the permissible stress
for compression in bars account, shall A st = Area of tension steel
be taken as the calculated compressive m’ = 1.5 m = modular ratio of compression
stress in the surrounding concrete steel.
multiplied by 1.5 times the modular m = modular ratio of tension steel
ratio or σsc whichever is lower where f sc = Stress in compression steel
σsc is the permissible stress in f st = Stress in tension steel
compression in column bars. M = applied moment
• Generally the use of compression steel • When compression reinforcement is
is considered uneconomical. However, provided in addition to tension

© Copyright Reserved by Gateflix.in No part of this material should be copied or reproduced without permission
reinforcement in beam, such beams (x − d ')
= (1.5m − 1)ASC .f cbc
are termed as doubly reinforced beams. x
• Hanger bars of nominal dia used for the M = C1{d-(x/3)} + C2(d-d’)
purpose of holding stirrups, do not From this equation (α) find fcbc ....
normally qualify as compression
(α )
reinforcement, unless the area of such
bars is significant (greater than 0.2%) fsc/1.5m = fcbc(x-d’)/x

a) To find depth of neutral axis Find f sc from this


bx 2 (fst/m) = fcbc{(d-x)/x}
+ ( m'-1) A sc ( x-d') =mA st ( d-x )
2
bx 2 Find f st from this
+ (1.5m-1) A sc ( x-d') =mA st ( d-x )
2
. . . . (A) 4.5 MOMENT OF RESISTANCE OF
DOUBLY REINFORCED SECTION
b) To find out stress in concrete and
steel for a given applied moment I) Find depth of N.A. (i.e., x)
(i) By flexure formula:- x
II) Find n =
bx 3 d
I = + (1.5m − 1)A sc (x − d ') 2 ; III) Find n 0 = Neutral axis coefficient for
3
= mA st (d − x) 2 singly reinforced balanced section
I = M.O.I. about N.A. VI) If n < n 0 Tension steel reaches
Mx permissible stress
f cbc = ⇒ Find f cbc from this, f cbc =
I ⇒ f st = σst
stress in concrete in bending  x  σst
compression f cbc =  
 d-x  m
fsc/1.5m = M(x-d’)/I
M.O.R.
Find f sc from this; f sc = stress in
 x
compression steel = C1  d-  +C2 ( d-d')
 3
f st M(d-x)
M
=
I
⇒ Find f st from this; f st 1
C1 = f cbc xb,C2 = (1.5m-1) A sc f cbc
( x-d')
= stress in tension steel 2 x
(ii) By internal couple method V) If n ≥ n 0 , concrete, reaches its
permissible stress
⇒ f st =σ cbc
 x 1
⇒ MOR = C1  d-  +C2 (d-d')C1 = σ cbc bx
 3 2

C2 = (1.5m-1)A sc σ cbc
( x-d')
f sc = Stress in compression steel x
C1 = Compression carried by
concrete = (1/2)fcbc xb 4.6 DESIGN OF DOUBLY REINFORCED
C2 = Additional force carried by SECTION (RECTANGULAR)
• When the size of the beam is restricted
compression steel and applied moment is more than the
C2 = (1.5m − 1)ASC × fSC moment of resistance of balanced

© Copyright Reserved by Gateflix.in No part of this material should be copied or reproduced without permission
section, we would design the section as 20 and Fe 415 grade of steel is used. Take,
doubly reinforced section. Compression m = 13, σcbc = 7N/m 2 , σst = 230 N/m3 .
steel provided in doubly reinforced Solution:
section is useful even otherwise d = 600 – 40 = 560 mm
because Critical depth of N.A.
• It permits smaller size beam
 mσcbc  13 × 7
• Reduces long term deflection and xm =  d = × 560
increases ductility of beam  σst + mσcbc  230 + 13 × 7
• Can be used as anchor bar for x m = 158.75 mm
positioning shear reinforcement
Actual depth of N.A.
• As compression steel increase ductility,
they are provided (even if not required bx 2
+(1.5m-1)A sc (x-d c ) = mA st (d-x)
for strength) in seismic zone to 2
withstand repeated reversals produced. 300x 2
A doubly reinforced section is designed +(1.5×13-1)×π×122 (x-40) =
2
as a combination of (a) A balanced
section (singly reinforced) and (b) A 13×π252 (560-x)
section containing only tension and 150x2 + 8369.20(x – 40)
compression reinforcement. = 25525.4(560 – x)150x2 + 8369.20x –
Applied moment 334768
M = Rwbd2 + (M-Rwbd2) = 14.29 × 106 –25525.4x
= M1 + M 2
i.e., M 105x2 + 33894.6x – 14.62 × 106= 0
X = 219.03 mm
x > xm section is over reinforced

For over reinforced section


x > xm
fcbc = σ cbc
fst< σ st
1  x
A st =A st 1 +A st 2 MR c = bxσ cbc  d-  +
2  3
(1.5m-1)A sc f cbc '(d-d c )
R w bd 2 (M-R w bd 2 )
A st = +
σst (J 0cl) σst (d-d') f cbc ' can be find similar triangle
σ cbc f cbc
2
M - R w bd = [(1.5m-1)A sc ]
=
 x-d'  x x-d c
 σ cbc .  (d - d')
 x  f cbc ' = 7 × (219.03 − 40) = 5.72 N/mm2.
219.03
M-R w bd 2
⇒ A sc = 1  219.33 
  x-d'   MRc= × 300 × 219.33 × 7  560 − +
(1.5m-1) σ cbc   [ d-d'] 2  3 
  x  
(1.5 × 13 − 1) × π × 122 × 5.72(560 − 40)
= 111.99 × 106 + 24.89 × 106
Example
Find out the moment of resistance of the
MRc = 136.9 kN.m
section given below? Grade of concrete is M
4.7 FLANGED BEAM

© Copyright Reserved by Gateflix.in No part of this material should be copied or reproduced without permission
• The compressive stress variation due to
4.7.1 INTRODUCTION flexure is non-uniform over the width of
the slab.
• In monolithic construction slab and • For ease in calculation, we assume the
beams are cast together. If slab in such stress distribution across the width to
cases is in compression zone they be uniform.
become effective (partially or wholly) in • The width of flange with constant
adding significantly to the area of compressive stress equal to the peak
concrete in compression in beam. actual flexural compressive stress ( α )
However if flanges (slab) are located in which leads to the same longitudinal
tension zone, concrete in the flange compressive force as due the
(slab) becomes ineffective in cracked original stress distribution is called
section analysis. effective width of flange.

• In support region of a continues beam,


B.M. is negative (-ve) i.e., hogging and
slab therefore is in tension. Thus beam
in that region even if cast Note:-
monolithically, is designed as Effective flange width increases with
rectangular beam. a) Increased span
b) Increased web width
c) Increase flange thickness
d) The actual variation of flexural
compressive stress with max value near
web and decreasing away from web
location is attributed to shear lag
phenomenon.
• Away for the support region, the slab Effective flange width also depends on type
will be in compression (BM being of loading (concentrated or distributed)
sagging). Hence in this region, beam is and on the support conditions.
designed as flanged beam in which part
of the slab is taking as a part of beam. 4.9 IS CODE SPECIFICATIONS

4.7.2 Inverted beam 4.9.1 Effective flange with for T and L


beams
Inverted beams are recommended for
architectural requirement only (i.e., to L-beam
provide greater overhead clearance) such l0
b f =b w +3Df +
beam are designed as rectangular beam, 12
because slab is a tension zone and does not
resist any compression.

4.8 EFFECTIVE FLANGE WIDTH

© Copyright Reserved by Gateflix.in No part of this material should be copied or reproduced without permission
For fixed condition also we can take l0 to
be equal to that corresponding to
continuous case.
• Examples of isolated T beams are
stringer beam of strain case
• For T-or L-beam action, i.e., slab will act
l1 as compression flange of a T-beam or L-
bf bw + beam only when
2
T-beam a) Concrete of slab and web is
monolithic
l0
bf =bw + 6 D f + b) Web reinforcement is adequately
6 anchored in slab
l1 + l2 c) Adequate transverse reinforcement
bf ≥ b w +
2 must be provided near the top of the
Where flange such transverse
bf = effective width of the flange, reinforcement is usually present in
bw = breadth of the web. the form of (-ve) moment
Df = thickness of the flange, reinforcement in the continuous
l0 = distance between point of zero moment slab which spans across and form
in the beam the flanges of the T-beam.
When this is not the case (as in slabs
l0 = effective span beam for simply
where the main bars run parallel to
supported (SS) beam and beam), transverse reinforcement
= 0.7 × (effective span of beam) for must be provided in the flange of the
continuous beam. T-beam or L-beam. The area of such
transverse reinforcement is 60% of
the main area of steel provided at
mid span of the slab and should
extend on either side of beam to a
1
distance not less than 4 th of the
span of the beam.

Analysis of flanged section:-


Effective width for flange in T or L beam bf Df 2
(Isolated):- 1) ≥ mA st (d-Df ) then x will be less
2
than Df i.e., neutral axis lies in the
flange

2) If (bfDf2/2)< mAst(d-Df) then x will be


more than D f i.e., neutral axis lies in
the web.

Note:-

© Copyright Reserved by Gateflix.in No part of this material should be copied or reproduced without permission
(b) Using flexure formula:
bf x 3 (x-Df )3
I= -(b f -b w ) +
3 3

m Ast (d-x) 2
Cases – I: - When neutral axis lies in the
flange, analysis and design will be same as Mx
f cbc =
that of a singly reinforced I
rectangular section of width bf.
Cases – II: - When neutral axis lies in web M(d-x)
fst=m
I

(ii)To find out moment of resistance of


the section:
x
a) Find x and hence n = (check if x > Df)
d
σst  x 
C1 = compression carried by (+ ve) area • If n < n 0 ,f st =σ st and fcbc =  
m  d-x 
[bf x]
C2 = Compression carried by (− ve) area • If n >n 0 ,f cbc =σ cbc
[bf – bw) (x – Df)] • Moment of resistance
MOR = C1[d-(x/3)]-C2[d-Df -{(x-Df)/3}]
(i) To find out N.A. 1
C1= f cbc b f x
 ( x-Df ) 
2
bf x 2 2
-(b f -b w )   =mA st (d-x) ..(i) 1   x-Df 
2  2  f cbc    × ( x-Df ) ( b f -b w )
Solve the above equation to find out x. C2 = 2   x 

(ii)To determine stress in concrete Note:T-beams hardly requires comp. steel


and steel:
(a)By internal couple method: Doubly reinforced flanged beam:
Df 2
 x  x-Df  x1 =b f +1.5m-1) A sc (Df -d') x2 = mAst (d
C1  d-  -C2  d-Df -  =M . . . . .(ii) 2
 3  3  – Df)
Where a) x1> x2, Neutral axis lies in flange
C1 = (1/2)fcbc xbf b) x1< x2, Neutral axis lies in web
1
C2 = f ck (x-Df )×(b f -b w )
2
1 f 
= ×  cbc (x-Df )  [ x-Df ] (b f -b w )
2  x 
by solving equation (ii) we can find out fcbc
mf cbc
⇒ f st = (d-x)
x

© Copyright Reserved by Gateflix.in No part of this material should be copied or reproduced without permission
x
n= (check if x > Df)
d
ii) If n < n0, fst = σ st ,
σst  x 
f cbc =  
m  d-x 
n > n, fcbc = σ cbc

Moment of resistance is given by


Case:- 1 when the neutral axis lies in
flange, the beam will be analysed  x  x-Df 
designed in the same way as doubly
C1  d-  -C2  d-Df - 
 3  3 
reinforced rectangular section of M.O.R.=
width bf   x-d'  
Case: - 2 When neutral axis lies in web
+(1.5m-1)Asc f cbc    ( d-x')
  x 
1
C1= f cbc b f x
2
 x-Df 
C 2 =  f cbc  ( b f -b w )
 x 
4.10 DESIGN OF FLANGED SECTION

1) To find neutral axis: a)Fixed the width of beam from


architectural requirement and also to
(bf x2/2)- (bf-bw)⨉{(x-Df)2/2} + (1.5m-1)Asc(x – d’) = accommodate the reinforcement fulfilling
mAst (d – x) …….(A) the requirement of spacing of bars.
b)Normally, bw is taken as less than (l/25)
Solve this equation (A) to find x . . . . . (A) and should never be < 230 mm (l = span of
beam)
2) To find out stresses: l l
c)Adopt overall depth as to
 x  x-Df  10 12
M=C1  d-  -C 2  d-Df - +
 3  3   D 
d) Assume the lever arm to be  d- f  and
 2 
 x-d' 
(1.5m-1)Ast fcbc ( d-d')
x 
M
 Ast=
 Df 
σst  d- 
C2 = (1/2)[fcbc{(x-Df)/x}](bf-bw)  2 
Solve this equation (B) to find out f cbc
Note:- MOR is calculated after providing Ast
 d-x  and it should be greater than Mapplied.
f st =m   f cbc
 x  Note that T-beam hardly requires
 x-d'  compression steel.
f sc =1.5mf cbc  
 x  Example
Find the moment of the resistance of
3) Calculation of moment of resistance: isolated T-beam shown in figure below is
i) Find x and hence simply supported over a span of 6 m use M

© Copyright Reserved by Gateflix.in No part of this material should be copied or reproduced without permission
20 grade of concrete and Fe 415 grade of 230
steel. Take σ cbc = 7 N/mm2, σ st =230 f cbc
= 13
N/mm2, m = 13. 114.20 500 − 114.20
Solution :
Effective width of flange is lesser of (bf) (1) f cbc = 5.24 N/mm2.
=
l0
+b w . . . . . (i)  114.20 
M.R.= 1 ×966.67×114.20×5.2  500- 
l0 2  3 
+4
b M.R. = 133.60 kN-m
6000
= + 300 = 966.67 mm O.K. 4.11Limit State of Collapse in Shear
6000
+4 The section of a structural member may
1200 be subjected to shear force due to
flexure, punching or torsion. Accordingly,
1) b = 1200 mm . . . . . (ii) the shear may be flexural shear, punching
shear or torsional shear.
bf = 966.67 mm
1) Flexural shear: The shear associated
Critical depth of N.A. xm = with change of bending moment along the
[mσcbc/(σst+mσcbc)]d span is known as flexural shear, or simply
shear.
 13 × 7  Flexural shear force present in beam is
=  × 500
 230 + 13 × 7  dM
given by =V
xm= 141.74 mm dx
The horizontal and vertical shear stresses
Actual depth of N.A. are to be accounted for in the designs of
Assume N.A. is lies in flange portion beams. Exact analysis of shear in a
x2 reinforced concrete beam is quite
bf . =mA st (d-x) complex, several experimental studies have
2
been conducted to understand the various
x2
966.67 = 13 × 1256.64 (500-x) modes of failure which could occur due to
2 possible combination of shear and bending
48.3x2 = 8.168 ×106 – 16336.28x moment acting at a given section.
X =114.19 mm less than 150 mm so our
assumption in correct. And x < xm section
is under reinforced 2) Punching shear:
For under reinforced section • The shear associated with the
x < xm possibility of punching a thin member
f cbc ,σ cbc by a concentrated load is called
f st =σst punching shear.
• A slab carrying a concentrated wall
 x
M.R. = σ st A st  d-  =230×π× load, a beamless floor slab supported
 3 directly by columns (called flat slab)or a
202  500 − 114.20  = 133.51 kN-m footing slab carrying a concentrated
 3  column load are subjected to punching
OR shear.
1  x For the member subjected to both the
M.R. = b f xf cbc  d-  above types of shear, the flexural shear
2  3
is referred to as one-way shear,

© Copyright Reserved by Gateflix.in No part of this material should be copied or reproduced without permission
whereas punching shear is called two 2
σ σ
way shear. A footing slab carrying by σ1 or σ2= ±   + ( τ)
2

concentrated column load is subjected 2 2


to both these shears.

3) Torsion shear:
When a member is subjected to torsion, it
is subjected to torsional shear.
• The beams are usually subjected to
flexural shear, and sometimes to torsion
shear also. the inclination of principal planes is given
• The slab is the plate elements and 2τ
usually subjected to flexural shear. by tan2θ=
σ
However, sometimes they are subjected The major principal stress is tensile and is
to all the types of shears as in case of equal to
restrained two- way slabs and the flat 2
slabs. σ σ
σ1= +   + ( τ)
2

• Usually the shear failures of shallow 2 2


RCC beams may not lead to immediate The minor principal stress is compressive
failure, however it considerably reduces and equal to
its flexural strength and thus there is a
σ σ
2
state of impending shear failure. Hence
σ2= -   + ( τ)
2

the shear design is considered as limit 2 2


state of collapse. Two important cases are discussed below
• If the shear failures take place before
flexural failures, they are brittle and (i)If B.M. = 0, i.e., σ =0, then σ 1 = τ and σ 2 =
occur without warning. lf the flexural
- τ tan2θ = ∞ and θ = 450 or 1350
failure takes place prior to shear failure,
• This means that near the support for
the ductile failure of the beam is
simply supported beam, where B.M. is
ensured.
zero, and at N.A. of any section the
Note:
principal stress is equal to shear stress
Concrete is very strong in compression and
and is inclined at 450.
also quite strong in shear, however the
combination of vertical and horizontal • This is known as diagonal tension. As
shear stress along with tension due to the concrete is weak in tension, the
bending produces diagonal tension which concrete near the support cracks at 450
is quite and will be now examined. The with
complementary diagonal compression horizontal (i.e., perpendicular to
should also be taken into account. diagonal tension) as shown in fig below.
• To avoid the shear cracks, the beam
4.12DIAGONAL TENSION AND should be reinforced across the cracks
DIAGONAL COMPRESSION (i.e., along the principal tension)
Consider a small element along the length • The other principal stress is inclined at
of the beam. Subjected to pure shear. The 1350 and is compressive.
principal stress on this element are given • This is known as diagonal compression
and is of the same value as the shear
stress.
• The concrete is strong in compression
and for usual cases diagonal

© Copyright Reserved by Gateflix.in No part of this material should be copied or reproduced without permission
compression is below the permissible
value. Thus at mid-section, where bending
• However, if the shear stress is very stresses are predominant, the cracks will
high, precautions to avoid the diagonal start developing vertically. These cracks
compression failure also have to be are called flexural cracks.
taken. From above discussion, it is clear that
• The diagonal tension and compression between the two limits, the cracks will
near the support are shown in fig. The change from a vertical direction at a point
shear cracks are shown in fig. of zero shear to a direction inclined at an
angle of 450 at a point where bending
stress is zero.

From above the discussion, different modes


of failure are:

1. Diagonal Tension failure:

Which occur under large shear force and


less bending moment?
Such cracks are normally at 450 with the
longitudinal axis of beam

2. Flexural shear failure:


Which occurs under large bending moment
(ii)When bending stress ( σ ) is maximum, and less shear force? Which occurs
while shear stress is zero (i.e. τ = 0), we normally at closer to 900 with horizontal?
get, σ1=σ2=σ and θ = 900 When flexural crack occurs in combination
i.e. principal plane is perpendicular to the with a diagonal tension crack (as in usually
beam axis. the case), the crack is sometimes called
This means that principal tensile stress acts flexural shear crack.
in horizontal direction and cracks will be
vertical as shown in Fig. below

It should be noted that it is due flexure


crack that usually forms Ist and due to

© Copyright Reserved by Gateflix.in No part of this material should be copied or reproduced without permission
increased shear stress at the tip of the
crack this flexural crack extends into The relative proportion of various
diagonal tension crack. mechanisms depends on the loading stage,
the extent of cracking and material and
3.Diagonal compression failure: geometric properties of the beam.
Which occurs under large shear force? It is • Prior to flexural crack All shear resisted
characterized by the crushing of concrete. by Vcz
Normally it occurs in beams which are • At the commencement of flexural
reinforced against heavy shear. cracking Va and Vd develop.
• At the diagonal tension cracking Vs
develops (all the four mechanism exist
at this stage)
• Increase in longitudinal reinforcement
not only increases Vd but also controls
Note: the propagation of flexural crack and
For deep beam and thin walled sections contribute to increase the depth of N.A
subjected to large concentrated loads, and thereby the depth of uncracked
maximum tensile stress is located near N.A. concrete due to increased % tensile
resulting in crack which generally starts at reinforcement crack formed as are
N.A. and propagate to support. smaller which improves the aggregate
This crack is also called web shear crack or interlocking.
diagonal tension crack tension crack.
Such type of failure occurs when area of 4.14 SHEAR STRESS
shear reinforcement exceeds a certain limit
such that concrete section becomes
stronger in diagonal tension compared to
diagonal compression.
Thus diagonal compression failure may
occurs even before the shear reinforcement
has yielded.

4.13 MECHANISM OF SHEAR RESISTANCE

Distribution of shear stress in reinforced


concrete is shown as below.

© Copyright Reserved by Gateflix.in No part of this material should be copied or reproduced without permission
However for the solve of simplification we Mu Mu
C= T= =
use nominal shear stress as the design u u
Z d
stress for shear Vu net= Vu + Tu tan β
V
Nominal shear stress = u =τ v Vu net =
Vu − Tu tan β
bd
For uniform depth Mu
=
Vu net =
Vu − tan β
Vu = shear force due to design load d
Mu
b = breadth of member (which for flanged =
Vu + tan β
d
section shall be taken as the breadth of web
b w)
4.15 DESIGN SHEAR STRENGTH OF
d = effective depth
CONCRETE IN BEAMS
For beam of varying depth
The design shear strength of concrete in
beams shall be taken as follows:
𝑀𝑀𝑢𝑢 tan 𝛽𝛽
𝑉𝑉𝑢𝑢 ± � �
𝑑𝑑
𝜏𝜏𝑣𝑣= (1) Without shear reinforcement:
𝑏𝑏𝑏𝑏 From table 19 of IS: 456. The values given
in table are based on the following
equation:
M u =bending moment at the section
β = angle between the top and bottom
τc =
0.85 0.85f ck 1+5β-1 ( )
edges of the beam. 6β
(-) sign = when BM (numerically increases
0.8f ck
in the direction in which depth increases) β= , but not less than 1.0.
i.e., when BM decreases in the direction of 6.89p t
effective depth increase (+) other Wise β 100A s
pt = ; for flanged section b shall be
bd
taken as bw , the width of rib.

Table: Design shears strength of


Note:
concrete
τ c N/mm2.
100 As Concrete grade
bd M20 M25 M30
(1) (2) (3) (4)
≤0.15 0.28 0.29 0.29
0.25 0.36 0.36 0.37
0.50 0.48 0.49 0.50
0.75 0.56 0.57 0.59

© Copyright Reserved by Gateflix.in No part of this material should be copied or reproduced without permission
1.00 0.62 0.64 0.66 not be considered for calculation of
1.25 0.67 0.70 0.71 100A s
1.50 0.72 0.74 0.76 bd
For solid slabs (resisting on beam)
Note: design shear strength of concrete shall
The term As is the area of longitudinal be K τ c where the value of K has given
tension reinforcement which continues at as below
overall ≥ 300 275 250 225 200 175 ≤ 150
least one effective depth beyond the section depth of
being considered except at support where slab mm
the full area of tension reinforcement may K 1 1.05 1.1 1.15 1.20 1.25 1.30
be used provided the detailing conforms to
the code requirements. Note:
• From above it is clear that the design Experiment studies have shown that slabs
shear strength τ c in concrete depends and shallow beams fail at loads
on the percentage of tension steel. This corresponding to nominal stress that is
is because: higher than that applicable for beam of
(i) When the amount of tension steel usual proportion
increase, the depth of neutral axis • Thinner is the slab greater is the
increases and thus, the depth of increase in shear strength.
uncracked concrete increases. This • Slabs subjected to normal distributed
increases the capacity of concrete in loads satisfy τ v < k τ c , and hence do not
shear.
need.
(ii) When the amount of tension
Shear reinforcement. This is because
steel increases,the cracks formed
thickness of slab (controlled by Limiting
are smaller, which improves the
deflection criteria is usually adequate in
aggregate inter-lock. Also because of
term of shear capacity)
larger steel area the dowel action is
improved. This further improves the • lf the nominal shear stress does not
capacity of section in shear. exceed the value of τ c , the section is safe
• Further it is indicates that shear for shear and shear reinforcements are
strength of concrete is also related to not required theoretically. However,
the compressive strength of concrete, In some minimum shear reinforcement
fact the shear strength of concrete is a shall be provided. lf the nominal shear
function of f ck where fck is the stress exceeds the design shear
strength τ c , the section shall be suitably
characteristic compressive strength of
reinforced with shear reinforcements.
concrete.
• When the longitudinal bars are not
Max shear stress with shear
required to resist moment, they are
reinforcement:
sometimes curtailed or bent up. lf the
• Under no circumstances, even with
bars are curtailed, they create
shear reinforcement shall the nominal
complicated stresses at the point of
curtailment, thereby reducing the shear τ
shear stress in beams exceed c max
capacity of the section. Therefore while τ c max Depends on grade of concrete
using table any longitudinal bars which
are terminated within a distance of d of
the section under consideration, shall

© Copyright Reserved by Gateflix.in No part of this material should be copied or reproduced without permission
Grad M M M M4 M M
e 15 20 25 0 30 35
and
abo
ve
τ c max 2.5 2.8 3.1 3.5 3.7 4.0
N/m
The first stirrups shall be provided within
m2
50 mm from the face of the support
The primary functions of the stirrups are:
• By this provision failure of beam by
(i) to resist a part of the shear
diagonal compression is prevented.
(ii) to resist the growth of the inclined
• If τ v exceeds τ c max section shall be cracks and improve aggregate interlock,
redesigned either by using higher and
concrete mix or by increasing the size of (iii)to tie the longitudinal bars in place,
section. thereby increased the dowel action.
• In solid slabs, τ v ≯ 0.5 τ c max Closed stirrups are better because it resist
torsion and helps in confining the
compression reinforcement (in doubly
4.16 SHEAR REINFOCEMENT IN BEAM reinforced beam)
Due to diagonal tension concrete cracks
and if this crack is not intercepted,
concreted section will split under
overloading condition. To avoid it, we
provide shear reinforcement in the form of
(a) Vertical stirrups
(b) Inclined stirrups
(c) Bent up bars along with stirrups,
Shear reinforcement restrains the growth
of shear crack and resisting part of shear
not resisted by the concrete.

(1)Vertical stirrups:
Vertical Stirrups are the commonly used
shear reinforcement. These consist of a
series of vertical open closed bars spaced
Hook of the closed stirrups should be
along the beam span.
located in compression zone because in
The various types of vertical stirrups are
that case it helps in improving the
anchorage and avoid crack initiation. The
shear resisted through shear reinforcement
is explained as follows:
When the applied shear less than the
concrete capacity, the stirrups carry
practically negligible shear because the
concrete is not cracked in diagonal tension.
When the concrete capacity is exhausted, a
shear crack forms. Simultaneously, the
stirrups crossing the potential crack are

© Copyright Reserved by Gateflix.in No part of this material should be copied or reproduced without permission
put into action. The additional shear, i.e., 0.87f y .A sv .d 0.87f y .A sv .d
=
the shear in excess to the concrete capacity s =
(τ v − τc ) bd
v
Vus
is now carried by the stirrups which cross
this potential crack. This also means that 0.87f y .A sv
=
any stirrup not crossing the crack b(τ v − τc )
essentially remains unstressed. …. (b)
Thus, the stirrups will be designed
observing that they cross a crack, say the
first crack at 450 near the support as shown
• Note that the value of f y is limited to
in fig. The stirrups are spaced at a distance
sv apart and they cross the crack. 415 N/mm2 by the code. This is because
the width of the diagonal crack is
Let Asv = area of legs of one stirrup
directly related to the strain in the
f y = characteristic strength of stirrup (or stirrups. Limiting the stress in the
bent up bar) which shall not be greater stirrups will limit the strain and
than 415 N/mm 2 consequently the width of the shear
0.87 f y = design tensile strength of shear crack. Limiting the width of crack also
enables more aggregate interlock to
reinforcement
develop. If higher grade steel, say Fe
Vus = strength of shear reinforcement.
500 is used for shear reinforcement,
Now, The horizontal length of crack = d –
then also use f y = 415 N/mm2 for shear
d’≈ d (d’ neglected),
d − d' reinforcement.
tan 450 = , x =d − d ' =d
x
d
No of stirrups crossing the crack n =
sv
Shear taken by stirrups
d
= × A sv × 0.87f y
sv
This shall be equal to the required strength
Vus , of the shear reinforcement, • The stirrups are usually two-legged.
However stirrups of four-legged, six-
d legged, etc., also can be formed as
Vus = ×A sv ×0.87f y
sv shown in fig. More than two-legged
0.87f y .A sv .d stirrups are used for heavy shear.
Vus = …. (a) Unless otherwise specified, stirrups are
Sv always two-legged.
This formula is given in IS: 456:2000 and
will be used to design stirrups. Grade Fe Maximum spacing:
250 (mild steel) and Fe 415 (HYSD) bars The shear reinforcements are provided to
are used as stirrups. prevent the shear cracks in the beam. The
For design, usually we fix the grade of horizontal distance between two
reinforcement (i.e., f y ), and the diameter of successive cracks is approximately equal to
stirrups (i.e., A sv ). the effective depth d. Stirrups shall be
provided such that they cross the crack and
Then we use above equation (a) to
also no crack shall remain unreinforced.
determine the spacing of stirrups as:
To ensure this, the maximum spacing of
stirrups as per code shall not exceed:

© Copyright Reserved by Gateflix.in No part of this material should be copied or reproduced without permission
(i) 0.75 d for vertical stirrups This shall equal to Vus , the strength of the
(ii)d for inclined stirrups maximum spacing
stirrups required.
shall be limited to 300 mm.
• It may be emphasized that closely Thus for a series of inclined stirrups
spaced stirrups prevent the failure
d
caused by tearing of main Vus =0.87f y A svsinα(1+cotα)
reinforcement through concrete cover Sv
and slipping relative to the concrete.  cosα d
=0.87f y A sv  sinα+ .sinα 
 sinα  Sv
(2) Inclined stirrups: d
A series of inclined stirrups near the Vus =0.87fyA sv ( sinα+cosα )
Sv
support crossing a 450 crack is shown in fig,
lf A sv is the area of legs of inclined stirrups The inclined stirrups are very effective in
carrying flexural shear and restrain the
and 0.87 f y is the design tensile strength in
inclined crack very effectively. However, in
the shear reinforcements, the tensile force case of stress reversal (due to earthquake
in an inclined stirrup = 0.87f y .A sv . loads for example), the
direction of crack will be reversed and
inclined stirrups become ineffective
α ≮ 450 because for α <450 there is a
possibility of inclined stirrups slipping
along the longitudinal bars.
So that at least one reinforcement intercept
the crack
Max. Spacing ( Sv ) of included stirrups (with
α = 450) ≯ d also, Sv ≯ 300 mm
f y ≯ 415 N/mm2
Let inclination of stirrups is α with
horizontal. and crack is inclined at 450 (3) Bent up bars
(normally assumed) with the horizontal. When a single bar or a group of bars, all
from ∆ obs bent up at the same section
d-d' Vus =0.87.f y .A sv ….. (11)
tanα=
x π
A sv = No. of bars bent × d 2
x=(d-d')cotα 4
D= dia. of bent-up bars.
Neglecting d', No. of stirrups crossing the
d+x d+dcotα d
crack, n= = = (1+cotα )
Sv Sv Sv
Tensile force in an inclined stirrups
= 0.87f y .A sv
Shear taken by stirrups For series of bars bent up at different cross
= (d/Sv)(1+ cotα ) 0.87 fy Asv section,
Vertical component of this force effective in d
Vus =0.87f y A sv ( sinα+cosα )
shear Sv
d The bent-up bars alone are not satisfactory
= (1+cotα ) 0.87f y A sv sin α
Sv for the shear reinforcements. This is
because the exact behavior of bent bars in

© Copyright Reserved by Gateflix.in No part of this material should be copied or reproduced without permission
resisting shear is not clearly understood. ‘T’ or ‘L’ beams the b shall be equal to
Also, the bent bars do not resist the bw , the width of web.
reversal of shear force, IS: 456 states that
The minimum shear reinforcement is
when bent bars are provided, their
provided for the following:
contribution towards shear resistance shall
(i)Any sudden failure of beams is
not be taken as more than half of the total
prevented if concrete cover
shear to be resisted by shear
bursts and the bond to the
reinforcement, The remaining half of the
tension steel is lost.
shear to be resisted through shear
(ii)Brittle shear failure is arrested
reinforcement shall be resisted by
which would have occurred without
vertical/inclined stirrup which are
shear reinforcement.
provided along with the bent up bars.
(iii)Tension failure is prevented which
would have occurred due to
4.17 MINIMUM SHEAR REINFORCEMENT
shrinkage, thermal stresses and
internal cracking in beams.
Minimum shear reinforcement in the form
(iv)To hold the reinforcement in place
of vertical stirrups shall be provided such
when concrete is poured.
that
(v) Section becomes effective with the
A sv 0.4
≥ tie effect of the compression steel.
bsv 0.87f y
Where A sv = total cross-sectional area of 4.18 DESIGN STEPS
stirrup legs effective in shear
Sv = stirrup spacing along the length of the Vu
(a) Find =τ v
member in mm bd
b = width of the beam or width of rib of (b) Find τ cmax (depends on the grade of
flanged (i.e., bw for flanged beam) concrete)
(c) Check τ v >τ cmax if satisfied, redesign the
f y = characteristic strength of the stirrup
section.
reinforcement in N/mm2 which shall not be
(d) Find τ c from the steel and grade of
taken greater than 415 N/mm2.
• However, where the maximum shear concrete
stress calculated is less than half the (e) If τ v <0.5τ c , no shear reinforcement
permissible value and in the members provided
of minor structural importance such as (f) If 0.5τ C ≤ τ V ≤ τ C ,provided min shear
lintels, this provision need not be reinforcement
complied with. (g) If τ V >τ C , provide shear reinforcement
The above provision of minimum shear
reinforcement provides a shear for ( τ V -τC ) . This reinforcement not be
resistance of 0.4 N/mm2. For minimum less than the min shear reinforcement.
shear reinforcement, the spacing of
stirrup shall not exceed Example:
0.87.f y . A sv A ‘T’ beam section having 230 mm with of
sv ≤ ….. (12 b) web and 460 mm effective depth is
0.4b
It can be seen that for given type of steel reinforced with 5 no. of 16 mm dia bar as
and selected diameter of stirrups, the tension reinforcement, which continues for
spacing that provides minimum shear distance greater than effective depth , past
resistance is inversely proportional to the section, the section is subjected to a
the Width ‘b’ of the member. In case of factored shear force of 52.5 kN check in

© Copyright Reserved by Gateflix.in No part of this material should be copied or reproduced without permission
shear stress and design the Shear stress, remain plane even after bending,
the material M20, HYSD Fe415 for stirrups becomes valid in reinforced concrete
use 6 mm mild steel bar. only if the mechanism of bond is fully
Solution : effective.
Vu • The term ‘bond' describes the means by
i) τv=
bw d which the relative movement between
52.5×103 concrete and steel is prevented and the
τv =
230×460 intensity of adhesive force is called
τ v =0.4962N/mm 2 bond stress.
• Thus, bond stress is defined as
ii) Hence τc value corresponding to Pt = τ c
longitudinal shear stress acting on
= 0.608N/mm2 the surface between steel and
Pt =A s /b w d concrete.
π • Bond between steel and concrete is due
5× × (16 )
2

to combined effect of adhesive


Pt = 4
230×460 resistance, frictional resistance and
Pt =0.95% mechanical resistance (for deformed
bars).
iii)Check: τ v >τ cmax 0.4962>2.81
• The adhesive resistance is provided by
( τ cmax =2.81 forM20) …..(O.K.) ‘chemical gum' produced by concrete
τ v (0.4962)<τ c (0.608) during setting.
iv) So provide minimum reinforcement in • The bond due to friction is provided by
form of stirrup. gripping of bars due to shrinkage. The
A sv 0.4 friction gives considerable bond
≥ resistance. With increasing force in bar,
bSv 0.87f y
the adhesion is lost first than the
Use 2 logged 6 mm φ mild steel. friction between concrete and steel.
• The mechanical resistance is provided
4.19 BOND AND ANCHORAGE by deformed bars only (not by the plain
bars). The deformed bars have lugs, or
4.19.1 INTRODUCTION corrugations and give higher bond
resistance by providing an interlock
• While analyzing and designing the between steel and concrete.
reinforced concrete structures, the • ln deformed bar, adhesion and friction
basic assumption is that there is a become minor elements, and the bond
perfect bond between concrete and strength is primarily dependent on
steel, i.e., there is absolutely no slippage bearing of concrete against the lugs or
between the concrete and steel. corrugations.
• It is this bond which is responsible for Note:If bond is absent, the stress at all
the transfer of axial force from a points on a straight bar would be
reinforcing bar to the surrounding constant, as in a string or a straight
concrete, thereby providing strain cable.
compatibility and ‘Composite action’ of
concrete and steel. 4.19.2 TYPES OF BOND STRERSS
• If this bond is inadequate, ‘slipping’ of
the reinforcing bar will occur, The bond stress in reinforced concrete
destroying full ‘composite action’. members arises due to two distinct
Hence, the fundamental assumption of situations.
the theory of flexure, viz. plane sections

© Copyright Reserved by Gateflix.in No part of this material should be copied or reproduced without permission
a) The change in the bar force along its
length due to variation in bending
moment in this length, This type of
bond stress is called flexural band
stress.
b) From the anchorage of bar in case of
tension or compression. This type of
bond force is known as Anchorage bond
stress. Anchorage bond is also called
development bond.
• lt may be noted that the actual bond
stress will be influenced by flexural
cracking, local slip, splitting and
If φ is the nominal diameter of a bar,
other secondary effects. In
particular, flexural cracking has a Then
major influence in governing the Tension, T=(0.87f y )A st
magnitude and distribution of local π 
bond stresses. T=(0.87fy)  φ 2 
4 
This force must be transferred from
4.19.3 Anchorage Bond Stress and steel to concrete through bond acting
Development Length over the perimeter of the bar along its
length of embedment Ld.
• Anchorage bond stress arises when a lf τ bd is the average bond stress, than
bar is carrying certain force. ln
anchorage bond, it is necessary to Force =τ bd ×(πφ ).L d
transfer this force in the bar to the π 
For equilibrium (0.87f y )  φ 2 
surrounding concrete over a certain 4 
length. The length of bar ‘Ld’ is required = τ bd ×(πφ ).Ld
to transfer the force in the bar.
0.87f y
• This condition arises near the extreme Ld = φ
end (or cutoff point) of a bar subjected 4τ bd
to tension (or Compression). The concept underlying ‘development
• Following figure shows a steel bar length’ is that a certain minimum length
embedded in concrete and subjected to of the bar is required on either side of a
a tensile force T. Due to this force, point of maximum steel stress, to
there will be a tendency of the bar to prevent the bar from pulling out
slip out and this tendency is resisted by under tension (or pushing in, under
the bond stress developed over the compression).
perimeter of the bar, along its length of The value of design bond stress for
embedment. plain bars in tension prescribed by IS
• This required length ‘Ld' is called code are reproduced in table below :
anchorage length in case of axial
tension (or compression) & Table: Permissible bond stress in
development length in case of flexural tension
tension.
• The development length is an Grade of M40
embedded length of the bar required to concrete M20 M25 M30 M35 and
above
develop the design strength of
reinforcement at the critical section.

© Copyright Reserved by Gateflix.in No part of this material should be copied or reproduced without permission
Design bond called primary equilibrium torsion such
stress τ bd , torsion is induced in beams curved in plan
1.2 1.4 1.5 1.7 1.9
(N/mm2) and subjected to gravity loads and also in
beams where the transverse loads are
• In the above discussion φ is defined as eccentric with respect to the shear centre
of the x- section.
the nominal diameter of the bar. For the For example a slab cantilevered from a
plain bars, nominal diameter and actual
beam which is assumed to be fixed at
diameter are the same. supporting columns.
• However, for deformed bars, the cross- Slab load W induces a torsional moment of
section is not perfectly circular. Wx in beam, or a small beam BC is
• The actual area of the cross-section of a cantilevered from beam B1. Beam B1 is
deformed bar is equated with an area of considered fixed at columns. Here negative
a circle and corresponding diameter of moment of beam BC will be the torsional
the circle is known as the normal moment of beam B1.
diameter.
• Thus, although the shapes of cross-
sections of plain and deformed bars are
not equal but for a particular diameter,
their cross-sectional areas and mass per
metre length are the same.

Note:
1. τ bd for deformed bar is increased by
60%. This is because for deformed bars,
the actual contact area of a bar with
concrete is taken into account which is
much more than contact area based on
nominal diameter.
2. For bars in compression, the above
values may be increased by 25 percent.
This is because the end bearing of the
bar helps in resisting compression.
Also the possibility of cracking the
concrete is nil in compression and
hence, the allowable bond stresses are
increased.

4.20 TORSION

4.20.1 INTRODUCTION

Torsion produced in a beam is classified as Secondary torsion is induced in a structural


member by rotation (twist) applied at one
a) Primary torsion or equilibrium torsion or more points along the length of member
b) Secondary torsion or compatibility through inter connected members, instead
torsion of by directly applied load. Twisting
Torsion which can be determined only by moment induced in this case is
using the static equilibrium condition is

© Copyright Reserved by Gateflix.in No part of this material should be copied or reproduced without permission
proportional to the torsional stiffness of the reinforcement is provided in the form
member. of bars distributed around, the x-
These moments are generally statically section, close to periphery and
indeterminate and their value is obtained transverse reinforcement in the form of
using compatibility equation. This is the closed rectangular stirrups placed
reason these torsion are called perpendicular to the beam axis.
compatibility torsion. • The longitudinal reinforcement helps in
reducing the crack width through dowel
action and stirrups crossing the cracks
resist shear due to vertical loads and
torsion,
As a simplification, the effect of
torsional moment is split up into
a) Equivalent shear, and
b) Equivalent bending moment

As per IS code where torsion can be Code Provisions: (Clause 41)


eliminated by releasing redundant • If in the analysis of structure, the
restraints, no specific design for torsion is torsional resistance or stiffness of
necessary provided torsional stiffness is members has not been taken into
neglected in the calculation of internal account, no specific calculation for
forces. torsion is necessary, adequate control
The maximum torsional shear stress occurs of any torsional cracking being
at the middle of the wider face and has a provided by the required nominal shear
value given by reinforcement.
T
τ t; max =2 • Where the torsional resistance or
αb D stiffness of members is taken into
account in the analysis, the members
4.21 EFFECT OF TORSIONAL MOMENT shall be designed for torsion.
• For design of torsion, section located at
a distance less than ‘d' from the face
of the support may be designed for the
same torsion as computed at a distance
‘d’, where d is the effective depth.
The design rules for torsion are based
on equivalent moment. They are
• Torsional moment induces shear explained as per IS code 456: 2000
stresses in the beam because of the
torsion a beam fails in diagonal tension (a) Shear and Torsion (Clause 41.3.1):
forming spiral cracks around the beam The combined effect of torsion and shear is
as shown in Fig. 4.2. considered and called equivalent shear.
• Hence the ideal Way of providing Equivalent shear Ve is calculated from the
reinforcement against torsion is by formula
providing the steel in the form of a Tu
spiral along the direction of principal Ve =
Vu + 1.6 −
b
tensile stress. But this is not practical
Where, Ve = Equivalent shear at the section
and hence torsional reinforcement is
(in kN or N)
provided in the forms of longitudinal &
Vu = Ultimate vertical shear at the section
transverse reinforcement, longitudinal
(in kN or N)

© Copyright Reserved by Gateflix.in No part of this material should be copied or reproduced without permission
Tu = Ultimate torsional moment (in kNm or
Nmm) (c) Transverse reinforcement (Clause
And 41,4.3):
b = Breadth of beam. (in m or mm) Two-legged closed hoops enclosing the
The equivalent nominal shear stress τ ve is corner longitudinal bars shall have an area
given by, of cross-section A sv , given by
V Tu .Sv Vu .Sv
τ ve = e A sv = +
b.d b1d1 ( 0.87f y ) 2.5d1 (0.87f y )
The equivalent nominal shear stress τ ve
(τve - τc)bd = (0.87fyAsvd/Sv)
shall not exceed the values of τ c,max as given But the total transverse reinforcement shall
in table 20 of lS-456. not be less than
If τ ve does not exceed τ c as given in table Asv ≥ [{(τve-τcb)bSv}/(0.87 fy)]
19 of IS 456, only minimum shear Where,
reinforcement shall be provided min shear Tu = Ultimate torsional moment
reinforcement is given
Vu = Ultimate shear force
0.87f y (A sv )d
=0.4bd Sv = Spacing of the stirrup reinforcement
Sv
b1 = c/c distance between corner bars in
lt τ ve exceeds τ c as given in table 19 of IS
the direction of width
456, both longitudinal and traverse
d1 = c/c distance between corner bars in
reinforcement shall be provided (clause
41.3.3). the direction of depth
b = Breadth of the member
(b) Longitudinal Reinforcement (Clause f y = Characteristics strength of shear
41.4.2): reinforcement ≤ 415N/mm2
The longitudinal reinforcement shall be V
designed to resist an equivalent bending τ ve = e = Equivalent shear stress
b.d
moment Me1 , given by τ c = Shear strength of concrete, as in table
M e1 =M u +M t
19 of IS 456.
Where, M u = bending moment at the cross-
section (factored)
D
1+
Mt = Tu � b�
1.7

Where Tu = Torsional moment


D = Overall depth of beam
b = breadth of the beam
When Mt<Mu there is resultant torsion due
to ( M t - M u ). Therefore, the longitudinal
reinforcement shall be provided on the
flexural compression face, to resist the
equivalent moment
Me2 = Mt - Mu
When M t <M u , no additional steel is
required on bending compression side,

© Copyright Reserved by Gateflix.in No part of this material should be copied or reproduced without permission
5 BEAM COLUMN

5.1 DESIGN OF BEAM AND SLAB

A beam is a horizontal structural element whichever is less, the effective span shall be
that is capable of withstanding load taken as under:
primarily by resisting bending. The
bending force induced into the material of Case (1):
the beam as a result of the external loads, span  l0 
own Weight, span and external reactions to If width of support <  w< 
12  12 
these loads is called a bending moment. Then effective span is calculated same as
for simply supported case,
5.2 I.S 456 PROVISIONS w w
leff =l0 +d Or leff =l0 + +
1. Effective Span (Clause 22.2) 2 2
(a) Simply supported Beam or Slab Whichever in less

Case(2):
If width of support > [(span/12) or 600
mm]
(a) (i) For one end fixed other continuous
The effective span of a member that is (ii) Both end continuous (Intermediate
not built integrally with its supports span)
shall be taken as clear span plus the leff =l0 = clear span
effective depth of slab or beam or (b) One end discontinuous other continuous
centre of supports, whichever is less. (simply supported)
leff (Effective span)=l0 +d eff=l0 +d/2
OR w
w w
Or leff =l0 +
leff =l0 + + 2
2 2 Whichever is less?
Whichever is less Case(3):
d = effective depth; Cantilever
w = width of support 1eff =l0 +d/2
l0 = clear span;
leff = lo+(w/2)
leff = effective span

(b) Continuous Beams or Slab


in case of continuous beam or slab, if
the width of support is less than (1/12)
of the clear span, the effective span shall
be as in
(a) above. lf the supports are wider than
1
of the clear span or 600 mm,
12

© Copyright Reserved by Gateflix.in No part of this material should be copied or reproduced without permission
occurring after erection of partition and
span
application of finishes. or 20 mm
350
whichever is less.
Note:
1. Deflection can be reduced by providing
more tensile steel than required for
flexure, the service stress in steel is
reduced and hence the deflection.
However care must be taken so that
section will not become over reinforced.
2. Calculation of Deflection are required.
(a) When designer wishes to exceed
span/ depth ratio
(b) Where specific deflection control is
required.
(c) Where the structure is abnormal
due to loading or behavior.
Case (4) :
5.3 CONTROL OF DEFLECTION
For frame its centre to centre distance
between members.
The deflection of structure or part thereof
shall not adversely affect the appearance or
efficiency the structure or finishes of
partitions.
• For beam and slabs, the vertical
deflection limits may generally be
assumed to be satisfied, provided that
the span to depth ratio are not greater
leff = Centre to Centre distance between than the values obtained as below
a) Basic values of span to effective
members. depth ratios for spans upto 10m.
Cantilever 7
Case (5) : In the case of spans with roller Simply supported 20
or rocket bearings, the effective span Continuous 26
shall always be the distance between the b) For spans above 10m, the values in
centres of bearings. (a) may be multiplied by 10/span in
meters, except for cantilever in
2. Check for Deflection (cl 23.2/P-37) which case deflection calculations
The deflection shall generally be limited to should be made.
the following c) Depending on the area the stress of
1. The final deflection due to all loads steel for tension reinforcement, the
including the effect of temperature, values in (a), or (b) shall be
creep and shrinkage and measured modified by multiplying with the
from at cast level of support of floor, modification factor obtained as per
roof and all horizontal. Deflection Fig. 5.7 depending on reinforcement
should not normally exceed span/250. percentage and type.
2. The deflection including the effect of
• In above formula of finding out the
creep, temperature and shrinkage
modification factor for tension

© Copyright Reserved by Gateflix.in No part of this material should be copied or reproduced without permission
reinforcement indicates that if the dimensions. So slab is a two dimensional
designer wishes to use shallow member element. Slabs form roof or floor of the
he or she increase the area of building. Slabs are designed same as beams
reinforcement there by reduced the with unit width.
service stress hence the deflection,
however it should be remember that by 5.5 INTRODUCTION
increased Ast, section should not
become over reinforced. • Slabs, used in floors and roofs of
• The total depth D can be determined by buildings mostly integrated with the
adding nominal cover and half of the supporting beams, carry the distributed
diameter of the bar used to the effective loads primarily by bending.
depth. • lt has been mentioned that a part of the
integrated slab is considered as flange
Selection of Diameters of Bar of Tension of T - or L – beams because of
Reinforcement monolithic construction.
• Reinforcement bars are available in • However, the remaining part of the slab
different diameters such as 6, 8, 10, 12, needs design considerations.
14, 16, 18, 20, 22, 25, 28, 30, 32, 36 and • These slabs are either single span or
40 mm. continuous having different support
• Some of these bars are less available. conditions like fixed, hinged or free
• The selection of the diameter of bars along the edges. Though normally these
depends on its availability, minimum slabs are horizontal, inclined slabs are
stiffness to resist while persons walk also used in ramps, stair cases and
over them during construction, bond inclined roofs.
requirement etc. • While square or rectangular plan forms
• Normally, the diameters of main tensile are normally used, triangular, circular
bars are chosen from 12, 16, 20, 22, 25 and other plan forms are also needed
and 32 mm. for different functional requirements.
• This lesson takes up horizontal and
Selection of Grade of Concrete rectangular/square slabs of buildings
Besides strength and deflection, durability supported by beams in one or both
is a major factor to decide on the grade of directions and subjected to uniformly
concrete. Table of lS 456 recommends M 20 distributed vertical loadings.
as the minimum grade under mild
environmental exposure and other grades
of concrete under different environmental
exposures also.

Selection of Grade of Steel


Normally, Fe 250, 415 and 500 are in used
in reinforced concrete work. Mild steel (Fe
250) is more ductile and is preferred for
structures in earthquake zones where
there are possibilities of vibration, impact,
blast etc.
5.4 ONEWAY SLAB

Slabs are plate elements having depth


much smaller than its other two

© Copyright Reserved by Gateflix.in No part of this material should be copied or reproduced without permission
near the edges for the slab panel of Fig.
5.13 (a).

5.7 CODE REQUIREMENTS ON


REINFORCEMENT AND DETAILING

Longitudinal Reinforcement
5.7.1 Minimum Reinforcement
The longitudinal bars must, in general, have
a cross-sectional area not less than 0.8% of
the gross area of the column section. Such a
minimum limit is specified by the Code:
• To ensure nominal flexural resistance
under unforeseen eccentricities in
loading; and to prevent the yielding of
the bars due to creep and shrinkage
effects, which result in a transfer of
Note:
load from the concrete to the steel.
1. One – way slab if l y >2l x
• However, in the case of pedestals which
2. Indicates that no support is needed if are designed as plain concrete columns,
l y >2l x and is needed if l y ≤ 2l x the minimum requirement of
3. End supports may be simply supported longitudinal bars may be taken as 0.15
or clamped. per cent of the gross area of cross-
section.
5.6 ONE-WAY AND TWO-WAY SLABS • In the case of reinforcement concrete
• Figures 5.13 (a) and (b) explain the walls, the Code (Cl.32.5) has introduced
share of loads on beams supporting detailed provisions regarding minimum
solid slabs along four edges when reinforcement requirements for vertical
vertical loads are uniformly distributed. (and horizontal) steel.
• lt is evident from the figures that the • The vertical reinforcement should not
share of loads on beams in two be less than 0.15% of the gross area in
perpendicular directions depends upon general.
• This may be reduced to 0. 12% if
the aspect ratio l y / lx of the slab, lx
Welded wire fabric or deformed bars
being the shorter span. (Fe 415/Fe 500 grade steel) is used,
• For large values of l y , the triangular provided the bar diameter does not
area is much less than the trapezoidal exceed 16 mm.
area. Hence, the share of loads on • This reinforcement should be placed in
beams along shorter span will gradually two layers if the wall is more than 200
mm thick.
reduce with increasing ratio of l y / lx .
• ln all cases, the bar spacing should not
• In such case, it may be said that the exceed three times the wall thickness
loads are primarily taken by beams of 450 mm, whichever is less.
longer span. The deflection profiles of
the slab along both directions are also 5.7.2 Maximum Reinforcement:
shown in the figure.
• The deflection profile is found to be • The maximum cross-sectional area of
constant along the longer span except longitudinal bars should not exceed 6

© Copyright Reserved by Gateflix.in No part of this material should be copied or reproduced without permission
per cent of the gross area of the column
section.
• However, a reduced maximum limit of 4
per cent is recommended in general in
the interest of better placement and
compaction of concrete and, in
particular, at lapped splice locations.
• In tall buildings, columns locate in the
lowermost storeys generally carry
heavy reinforcement (~ 4 per cent). The
bars are progressively curtailed in
stages at higher levels.
5.7.3 Minimum diameter/number of
bars and their location
• Longitudinal bars in column (and
pedestals) should not be less than 12
mm in diameter and should not be
5.7.4 Arrangement of transverse
spaced more than 300mm apart
reinforcement
(centre-to-centre) along the periphery
of the column [Fig. 6.3(a)].
lf the longitudinal bars are not spaced more
• At least 4 bars (one at each corner)
than 75 mm on either side, transverse
should be provided in a column with
reinforcement need only to go round
rectangular cross-section, and at least 6
corner and alternate bars for the purpose
bars (equally spaced near the
of providing effective lateral supports.
periphery) in a circular column.
If the longitudinal bars spaced at a distance
• ln ‘spiral columns' (including of not exceeding 48 times the diameter of
noncircular shapes), the longitudinal the tie are effectively tied in two direction,
bars should be placed in contact with additional longitudinal bars in between
the spiral reinforcement, & equidistant these bars need to be tied in one direction
around its inner circumference [Fig, by open ties.
6.3(b)]. In columns with T-, L-, other
cross sectional shapes, at least one bar
should be located at each corner or
apex [Fig 6.3(c)].
• Longitudinal bars are usually located
close to the periphery (for better
flexural resistance), but may be placed
in the interior of the column when
eccentricities in loading are minimal,
When a large number of bars need to be
accommodated, they may be bundled,
or, alternatively, grouped, as shown in
[Fig 6.3(d)].

© Copyright Reserved by Gateflix.in No part of this material should be copied or reproduced without permission
5.7.5 Cover to Reinforcement: column) of the ties should be small
enough to reduce adequately the
A minimum clear cover of 40mm or bar unsupported length (and hence,
diameter (whichever is greater), to the slenderness ratio) of each longitudinal
column bars is recommended by the Code bar. The code recommendations are as
(Cl. 26.4.2.1) for columns in general; a follows:
reduced clear cover of 25 mm is permitted tie diameter φt ≥ φlong,max / 4 or 6mm
in small-sized columns (D ≤ 200 mm and whichever is maximum
whose reinforcing bars do not exceed 12
tie spacing St ≤ 16φlong ,min or D or 300 mm
mm) and a minimum clear cover of 15 mm
(or bar diameter, whichever is greater) is whichever is minimum
specified for walls. however, in aggressive • Minimum tie diameter was specified as
environments, it is desirable, in the interest 5 mm, instead of 6 mm in earlier
of durability, to provide increased cover version of code.
but preferably not greater than 75 mm. • Maximum tie spacing was specified as
48 φt instead of 300 mm in earlier
5.7.6 Transverse Reinforcement (Cl. version of code.
26.5.3.2 of the Code)
Where φlong denotes the diameter of
• General All longitudinal reinforcement longitudinal bar to be tied and D
in a compression member must be denotes the least lateral dimension of
enclosed with in transverse the column.
reinforcement comprising either lateral • When the spacing of longitudinal bars is
ties (with infernal angles not exceeding less than 75mm, lateral support need
1350) or spirals. This is required: only be provided for the corner and
 To prevent the premature buckling of alternate bars.
individual bars; • If the longitudinal bars spaced at a
 to confine the concrete in the ‘core’, distance not exceeding 48 φ are
thus improving ductility and strength; effectively tied in two directions, then
 to hold the longitudinal bars in position the additional longitudinal bars in
during construction; and between these bars need be tied only in
 to provide resistance against shear and one direction by open ties.
torsion, if required.
5.7.8 Spirals
5.7.7 Lateral Ties
Helical reinforcement provides very good
• The arrangement of lateral ties should confinement to the concrete in the ‘core’
be effective in fulfilling the above and enhances significantly the ductility of
requirements. the column at ultimate loads. The diameter
and pitch of the spiral may be computed as
• They should provide adequate lateral
in the case of ties except when the column
support to each longitudinal bar,
is designed to carry a 5 per cent overload
thereby preventing the outward
(as permitted by the Code), in which case
movement of the bar.
Pitch st < { 75mm or (core diameter/6)
• The diameter of the tie φ , is governed
St> {25mm or 3фt
by requirements of stiffness, rather The ends of the spiral should be anchored
than strength, and so is independent of properly by providing one and a half extra
the grade of steel. turns.
• The pitch st (centre-to centre spacing
along the longitudinal axis of the

© Copyright Reserved by Gateflix.in No part of this material should be copied or reproduced without permission
5.7.9 Design of Short Columns under conditions, the design strength of a short
Axial Compression column is
It load P is applied on a column of width B Pu =0.447f ck A c +0.87f y A sc … (iii)
and Depth D. However the code requires all columns to
be designed for minimum eccentricities in
loading hence equation (iii) cannot be
applied directly.
Nevertheless, where the calculated
minimum eccentricity (in any plane) does
not exceeds 0.05 time the least lateral
dimension (in the plane considered), the
Then Pu =Pc +Psc …. (i) code (Cl. 39.3) permits the use of the
Pc = load taken by concrete following simplified formula, obtained by
Psc =load taken by steel reducing the Pu by 10%.
Pu = total load taken up by the column Now, Pu =0.4f ck A c +0.67f y A sc
(ultimate load)
Load Example
We know that stress = Design A R-Column of size 450 mm × 600
Area
P P mm subject to an axial total of 2000 kN
f cc = c and fsc = sc
Ac Asc under service load condition. Unsupported
length of column is 3m, use M20 concrete
where f cc = Stress in concrete
and Fe 415 steel. Assume leff = l .
f sc =Stress in steel
Solution:
A c =Area of concrete
1) Check whether long or short column =
A sc =Area of steel 3000
= 6.67 < 12 so column is short
A = Gross area 450
=
Pu f cc A c + f sc A sc or 2) Check e min along 450 mm e min is greater
…(ii)
Pu = f cc A g + (f sc − f cc )A sc of
Maximum compressive strain in concrete (i) 3000 + 450 =
21mm
500 30
under axial loading at the limit state of
(ii) 20 mm
collapse in compression is specified of εc =
0.002 by the code. Corresponding to this e min = 21
limiting strain of 0.002 stress in the emin must be less than 0.05×450 =
0.67 22.5mm
concrete is f ck = 0.45fck and the design
1.5 e min (21) < 0.05B (22.5) O.K
strength of a short column is e min along 600mm
Pu =0.45f ck A g +(f sc -0.45f ck )A sc
3000 600
f sc =0.87f y for Fe250 e min is greater of (1) + =
26
500 30
0.790f y for Fe415 (1) 20 mm
0.746f y for Fe500 e min = 26 < 30 (0.05 D)
The design stress in concrete is So we can use Pu =0.4f ck A c +0.67f y A sc
0.67f ck
= 0.447f ck =0.4 × 20 × ( A − A sc ) + 0.67 × 415 × A sc
1.5
And design stress in steel is 0.87f ck in case
of Fe 250 under ‘pure’ axial loading

© Copyright Reserved by Gateflix.in No part of this material should be copied or reproduced without permission
2000 ×103 ×1.5 = 0.4 × 20 × (450 × 600 − A sc ) • But the design of member subjected to
+0.67 × 415A sc combined axial load and uniaxial
bending involves lengthy calculations
=216×104 -8A sc +278.05A sc by trial and error.
A sc =3110.5mm 2 • ln order to overcome these difficulties,
Provide 4 nos. 25 mm φ at corner interaction diagrams may be used.
These have been prepared and
4×491=1964mm 2
published by BIS in “SP: 16 design aids
An additional 4 nos. 20 φ
for reinforced concrete to IS 456".
=4×314=1256mm 2
A sc =3220mm 2 >3110.5mm 2 5.8.2 Interaction Curve
A 3220
p'= sc ×100=
bd 450×600
1.197 > 0.8 Minimum reinforcement
Lateral ties is greater of

i) 25/4 = 6.25 • The ‘interaction curve' is a complete


ii) 6 mm use 8 mm φ Spacing is mini. of graphical representation of the design
i) 450 mm strength of a uniaxially eccentrically
ii) 16 × 20 = 320 mm loaded column of given proportions.
iii) 300 mm • Each point on the curve corresponds to
Hence use 8 mm φ @ 300 mm c/c the design strength values of Pu and
M u associated with a specific
5.8 COLUMN SUBJECTED TO AXIAL eccentricity (e) of loading.
COMPRESSION AND UNIAXIAL BENDING • That is to say, if load ‘P’ is applied on a
(CLAUSE 39.5 ) short column with an eccentricity ‘e’
and if this load is gradually increased
5.8.1 INTRODUCTION till the ultimate limit state is reached
and that ultimate load at failure is given
• The load on the column is rarely axial. by P=Pu and the corresponding moment
There is always some minimum
by M=M u =Pu .e, then the co-ordinates
inherent eccentricity on account of
inaccuracies in loading, bad (M u ,Pu ) form a unique point on the
workmanship etc. diagram.
• ln such cases, the column is subjected to • The interaction curves define the
axial compression Pu and bending different (M u ,Pu ) combinations for all
moment M u . This loading system can be possible eccentricities of loading 0≤ e <
reduced to a single resultant load Pu ∞
acting at an eccentricity e=M u /Pu as • For design purposes, the calculations of
M u and Pu are based on the design
shown in Fig. 6.5.
stress strain curves (including partial
safety factors) and the resulting

© Copyright Reserved by Gateflix.in No part of this material should be copied or reproduced without permission
interaction curve is referred to as the values for this ‘balanced failure’
‘design interaction curve' (which is condition are denoted as Pub and M ub
different from the characteristic .For Pu <Pub (i.e e>e b ) the mode of failure
interaction curve).
is called tension failure.
• Using the design interaction curve for a
e) The point (5) corresponds to a ‘pure’
given column section, it is possible to
bending condition (e=∞,Pu =0) . The
make a quick judgment as to whether or
not the section is safe under a specified resulting ultimate moment of resistance
factored load effect combination is denoted by M ub and the
(Pu ,M u ) . corresponding neutral axis depth takes
• lf the point given by the co-ordinates on a minimum value X u,min . This case is
(Pu ,M u ) falls within the design the same as the under- reinforced
interaction curve, the column is ‘safe', section of beam.
otherwise it is not.
5.9 COLUMN SUBJECTED TO AXIAL
5.8.3 Salient Point on the Interaction COMPRESSION AND BIAXIAL BENDING
Curve (CLAUSE 39.6)

The salient points, marked 1 to 5 on the 5.9.1 INTRODUCTION


interaction curve corresponds to the failure
strain profiles, marked 1 to 5 in Fig. 6.6. The condition of biaxial bending along with
a) The point 1 corresponds to the axial compression is of common occurrence
condition of axial loading with e=0 for design of R.C. columns due to its
For this case of ‘pure’ axial monolithic construction with beams in two
compression, M u =0 And different directions. This is especially true
for corner columns in frames. The strength
Pu =Pu =0.45f ck A c +0.75f y A sc
of the section under axial compression and
b) The point (1) corresponds to the biaxial bending is a function of Pu , M ux and
condition of axial loading with the
M uy .
minimum eccentricity. e min = 0.05×D
The ultimate moment M ux about the
[clause 25.4 & 39.3] The corresponding
ultimate resistance is written as major axis of bending (i.e. X-axis) and M uy
Pu =0.4f ck A c +0.67f y A sc about the minor axis of bending (i.e. Y-axis)
c) The point (3) corresponds to the can be expressed in terms of axial
condition e = eD (i.e. xu = D). For e< or compression Pu acting at eccentricities
eD, the entire section is in under M ux M uy
compression and neutral axis is located ex = and e y =
Pu Pu
outside the section (xu> D)with 0.002 <
ε u < 0.0035. 5.10 CODE PROCEDURE FOR DESIGN
For e > eD, the natural axis is located OF BIAXILLY LOADED COLUMNS
within the section (xu< D) & ε u =0.0035 The column subjected to axial compression
at the “highly compressed edge”. and biaxial bending may be designed by the
Point(2) represents a general case, with following equation
the neutral axis outside the section (e  M ux 
αn
M 
αn

<eD).   +  uy  ≤ 1.0
d) The point (4) corresponds to the  M ux1   M uy1 
balanced failure condition, with e = eb
and xu = xu. lim. The design strength

© Copyright Reserved by Gateflix.in No part of this material should be copied or reproduced without permission
Where M ux and M uy = Moment about X member, whose l/D ratio is less than 12, is
and Y axes due to design loads. not in danger of buckling prior to achieving
M uxl and M uyl = Maximum uniaxial its ultimate strength based on the
properties of the cross-section.
moment capacity for an axial load of Moreover, the lateral deflection short
Pu , bending about X and Y axis respectively compression members subjected to bending
P moments are small, thus, contributing little
α n is related to u
Puz secondary bending moment P- ∆ as shown
Where in Fig. 6.8. These buckling and additional
Puz = 0.45fckAc + 0.75fyAsc deflection effects are more pronounced
= 0.45fckAg + (0.75fy-0.45fck)Asc in slender compression members and
reduce their ultimate strength as
compared to that of a short column having
Pu
For values of = 0.2 to 0.8, the values of the same cross-section and amount of steel.
Puz
α n vary lineraly from 1.0 to 2.0 For values 5.10.2 P - ∆ Effect in a column
less than 0.2, α n is 1.0; for values greater
than 0.8, α n is 2.0 as shwon in Fig. 6.7 The CEB-FIP recommendations for
buckling in compression and bending
advise a check for columns with effective
slenderness ratio more than 35. The
effective slenderness ratio should not
exceed 140 for normal aggregate concrete
and 80 for light weight aggregate. However,
clause 25.3.1 of the Code restricts
maximum slenderness ratio of a column to
60. If in any given plane, one end of a
Accordingly,
column is unrestrained, its effective length
Pu
α n = 1.0 for <0.2 l should not exceed.
Puz
b2
P 100
α n = 2.0 for u >0.8 D
Puz Where, b = width of cross-section
P
α n =0.667 + 1.667 u otherwise D = depth of cross-section measured in
Puz the plane under consideration
In the code, the effect of slenderness is
5.10.1 Slender Columns approximated by using a moment
magnifier approach, whereby the sum of
the primary and secondary moment is used
as total design moment. The additional or
secondary moments are given by:
May = (Pb/2000){(ley/b)2}
And Max= (PD/2000){(lex/D)2}
Where,
lex= effective length in respect of major axis,
that is, bending occurs about the major axis
An essential step in the design of a column
ley= effective length in respect of minor
is to determine whether the proposed
axis, that is, bending occurs about the
dimensions will make it a short column or a
minor axis
slender column. A short compression

© Copyright Reserved by Gateflix.in No part of this material should be copied or reproduced without permission
D = depth of cross-section at right angles to not be affected by seasonal change of
the major axis weather to cause swelling & shrinking
b = breadth of cross-section of the soil. Further, frost also may
These expressions are applicable to a endanger the foundation if placed at a
balance design of a slender column very shallow depth. Rankine formula
subjected to uniaxial loading as well as gives a preliminary estimate of the
biaxial bending. As the axial load increases minimum depth of foundation and is
from zero, the tensile stress in the steel expressed as
decreases to zero and changes to a q  1-sinφ 
2

compressive stress. As this occurs, the d= c  


γ  1+sinφ 
curvature and deflection decreases. Clause
39.7.1.1 of the code permits a reduction in Where
the additional moments by a factor k given d = minimum depth of foundation
by: qc = gross bearing capacity of soil
Pz − P γ = density of soil
=k ≤1 φ = angle of repose of soil
Pz − Pb
Pz = capacity of cross section under pure • Though Rankine formula considers
three major soil properties qc, γ and φ .
axial load
It does not consider the load applied to
Pz = the foundation. However, this may be a
Pz =0.45f ck A c +0.75f y A sc
guideline for an initial estimate of the
Pb = balance axial load corresponding to minimum depth which shall be checked
the condition of maximum compressive subsequently for other requirements of
strain of 0.0035 in concrete and tensile the design.
strain of 0.002 in the outermost layer of
tension steel.The value of Pb will depend 5.12 DESIGN CONSIDERATION
on arrangement of reinforcement and the (a) Minimum Nominal Cover (cl. 26.4.2.2
cover ratio d’/D, in addition to the grades of IS 456)
of concrete and steel. The values of Pb can The minimum nominal cover for the
be determined using the following equations: footings should be more than that of
For rectangular sections, other structural elements of the
 q p superstructure as the footing are in
Pb  q1 + 2  f ck bD
= direct contact with the soil. Clause
 σck 
26.4.2.2 of IS 456 prescribes a minimum
For Circular sections,
cover of 50 mm for footings. However,
 q p
Pb  q1 + 2  f ck D 2
= the actual cover may be ever more
 σck  depending on the presence of harmful
The coefficients q1 & q 2 are given in tables. chemicals or minerals, water table etc.
5.11 DEPTH OF FOUNDATION (b)Thickness at the Edge of Footing (cls.
34.1.2 and 34.1.3 of IS 456)
• All type of foundation should have a The minimum thickness at the edge of
minimum depth of 50 cm as per IS reinforced and plain concrete footings
1080-1962. This minimum depth is shall be at least 150 mm for footings on
required to ensure the availability of soils and at least 300 mm above the top
soil having the safe bearing capacity of piles for footing on piles, as pr the
assumed in the design. stipulation in cl. 34.1.2 of IS 456.
• Moreover, the foundation should be For plain concrete pedestals, the angle
placed well below the level which will α between the plane passing through

© Copyright Reserved by Gateflix.in No part of this material should be copied or reproduced without permission
the bottom edge of the pedestal and the
corresponding junction edge of the
column with pedestal & the horizontal
plane shall be determined from the
following expression (cl. 34.1.3 of IS 456)
tanα ≥ 0.9 √{(100qa/fck)+1} . . . . .
(11.3)
Where q a = calculated maximum
bearing pressure at the base of pedestal For round or octagonal concrete column or
in N/mm2, & f ck = characteristic strength pedestal, the face of the column or pedestal
of concrete at 28 days in N/mm2. shall be taken as the side of a square
inscribed within the perimeter of the
(c) Bending Moments (cl. 34.2 Of IS 456) round or octagonal column or pedestal (see
1. It may be necessary to compute the cl. 34.2.2 of IS 456 and Figs. (a) and (b).
bending moment at several section of
the footing depending on the type of d) Shear Force (cl. 31.6 & 364.2.4 of IS 456)
footing, nature of loads and the Footing slabs shall be checked in one-way
distribution of pressure at the base of or two-way shears depending on the nature
the footing. However, bending moment of bending. If the slab bends primarily in
at any section shall be determined one-way, the footing slab shall be checked
taking all forces acting over the entire in one-way vertical shear. On the other
area on one side of the section of the hand, when the bending is primarily two-
footing which is obtained by passing way, the footing slab shall be checked in
vertical plane at that section extending two-way shear or punching shear. The
across the footing (cl. 34.2.3.1 of IS 456.) respective critical sections and design
2. The critical section of maximum shear strength are given below:
bending moment for the purpose of
designing an isolated concrete footing 1. One – way Shear (cl. 34.2.4 of IS 456)
which supports a column, pedestal or One-way shear has to be checked across
wall shall be: the full width of the base slab on a vertical
i) At the face of the column, pedestal section located from the face of the column,
or wall for footing supporting a pedestal or wall at a distance equal to (Figs.
concrete column, pedestal or (a), (b) and (i).
reinforced concrete wall, Figs (a_), i) Effective depth of the footing slab in
(b) and (i), and case of footing slab on soil, and
ii) Halfway between the centre-line ii) Half the effective depth of the footing
and the edge of the wall, for footing slab if the footing slab is on piles Fig. (k).
under masonry wall (Fig. (i). this is The design shear strength of concrete
stipulated in cl. 34.2.3.2. of IS 456. without shear reinforcement is given in
The maximum moment at the critical Table 19 of Cl. 40.2 of IS 456.
section shall be determined as 2. Two – way or Punching Shear (cls.
mentioned in 1 above. 31.6 and 34.2.4)
Two-way or punching shear shall be
checked around the column on a perimeter
half the effective depth of the footing slab
away from the face of the column or
pedestal (Figs. (a) and (b).
The permissible shear stress, when shear
reinforcement is not provided, shall not

© Copyright Reserved by Gateflix.in No part of this material should be copied or reproduced without permission
exceed ksTc, where ks = ( 0.5+β c ) , but not uniformly across the full width/length
of the footing.
greater than one, β c being the ratio of short
iii) In two-way reinforced rectangular
side to long side of the column, and Tc = footing slabs, the reinforcement in the
 1
 long direction shall be distributed
0.25  ( f ck ) 2  in limit state method of design,
  uniformly long direction shall be
as stipulated in cl. 31.6.3 of IS 456. distributed uniformly across the full
Normally, the thickness of the base slab is width of the footing slab. In the short
governed by shear. Hence, the necessary direction a central band equal to the
thickness of the slab has to be provided to width of the footing shall be marked
avoid shear reinforcement. along the length of the footing, where
e) Bond (cl. 34.2.4.3 of IS 456) the portion of the reinforcement shall
The critical section for checking the be determined as given in the equation
development length in a footing slab shall below. This portion of the reinforcement
be the same planes as those of bending shall be distributed across the central
moments in part (c) of this section. band:
Moreover, development length shall be Reinforcement in the band = {2/ β +1}
checked at all other sections where they (Total reinforcement in the short
change abruptly. The critical sections for direction) where β is the ratio of longer
checking the development length are given dimension to shorter dimension of the
in cl. 34.2.4.3 of IS 456, which further footing slab. Each of the two end bands
recommends to check the anchorage shall be provided with half of the
requirements if the reinforcement is remaining reinforcement, distributed
curtailed, which shall be done in uniformly across the respective end
accordance with cl. 26.2.3 of IS 456. band.

f) Tensile Reinforcement (cl.34.3 of IS g)Transfer of Load at the base of Column


456) (cl. 34.4 of IS 456)
The distribution of the total tensile All forces and moments acting at the base
reinforcement, calculated in accordance of the column must be transferred to the
with the moment at critical sections, as pedestal, if any, and then from the base of
specified in part (c) of this section, shall be the pedestal to the footing, (or directly
done as given below for one-way and two- from the base of the column to the footing if
way footing slabs separately. there is no pedestal) by compression in
i) In one-way reinforced footing slabs like concrete and tension in steel. Compression
wall footings, the reinforcement shall be forces are transferred through direct
distributed uniformly across the full bearing while tension forces are transferred
width of the footing i.e., perpendicular through developed reinforcement. The
to the direction of wall. Nominal permissible bearing stresses on full area of
distribution reinforcement shall be concrete shall be taken as given below from
provided as per cl. 34.5 of IS 456 along cl. 34.4 of IS 456 :σ br =0.25f ck , in working
the length of the wall to take care of the stress method, and . . . . . (i)
secondary moment, differential
σ br =0.45f ck , in limit state method . . . . (ii)
settlement, shrinkage and temperature
effects. The permissible bearing stress of concrete
ii) In two-way reinforced square footing in footing is given by (cl. 34.4 of IS 456):
1
slabs, the reinforcement extending in σ br =0.45f ck (A1 /A 2 ) 2
each direction shall be distributed

© Copyright Reserved by Gateflix.in No part of this material should be copied or reproduced without permission
1 length of the column bar and into the
With a condition that ( A / A 1
)
2 2
≤ 2.0 footing, a distance equal to the
development length of the dowel, as
stipulated in cl. 34.4.4 of IS 456 and as
shown in Fig.

Where A1 = maximum supporting area of


footing for bearing which is geometrically
similar to and concentric with the loaded
area A 2 , as shown in Fig.
A 2 = loaded area at the base of the column.
The above clause further stipulates that in
sloped or stepped footings, A1 may be
taken as the area of the lower base of the
largest frustum of a pyramid or cone
contained wholly within the footing and
having for its upper base, the area actually
loaded and having side slope of one
vertical to two horizontal, as shown below:
i) Sufficient development length of the
reinforcement shall be Provided to
transfer the compression or tension to
the supporting member in accordance
with cl. 26.2 of IS 456, when transfer of
force is accomplished by reinforcement
of column (cl. 34.4.2 of IS 456.)
ii) Minimum area of extended longitudinal
bars or dowels shall be 0.5 percent of
the cross sectional area of the
supported column or pedestal (cl.
34.4.3 of IS 456.)
iii) A minimum of four bars shall be
provided (cl. 34.4.3 of IS 456.)
iv) The diameter of dowels shall not exceed
the diameter of column bars by more
than 3 mm.
v) Column bars of diameter larger than 36
mm, in compression only can be
doweled at the footings with bars of
smaller size of the necessary area. The
dowel shall extend into the column, a
distance equal to the development

© Copyright Reserved by Gateflix.in No part of this material should be copied or reproduced without permission
6 BASIC ELEMENT OF PRESTRESS CONCRETE

6.1 LOSSES IN PRESTRESS

• In prestressed concrete application, the


most important variable is the
prestressing force. In the early days, it
was observed that the prestressing
force does not stay constant, but
reduces with time.
• Even during prestressing of the tendons
and the transfer of prestress to the
concrete member, there is a drop of the The different types of losses encountered in
prestressing force from the recorded pretensioning and post tensioning are as
value in the jack guage. given below.
• The various reductions of the
prestressing force are termed as the Pre-tensioning Post-tensioning
1. Elastic 1. No loss due to elastic
losses in prestress.
deformation of shortening when all bars are
• The losses are broadly classified into concrete simultaneously tensioned. If
two groups, immediate & time- however, wires are
dependent. successively tensioned there
• The immediate losses occur during would be loss of prestress
due to elastic deformation of
prestressing of the tendons and the concrete
transfer of prestress to the concrete 2. Relaxation of 2. Relaxation of stress in
member. stress in steel steel
• The time-dependent losses occur 3. Shrinkage of 3. Shrinkage of Concrete
during the service life of prestressed Concrete
4. Creep of 4. Creep of concrete
member. concrete
• The losses due to elastic shortening of 5. Frictional losses
the member, friction at the tendon- 5. Anchorage slip
concrete interface and slip of the
anchorage are the immediate losses. Note:
• The losses due to the shrinkage and The total losses are around 15 to 20% of
creep of the concrete and relaxation of the initial prestressing.
the steel are the time dependent losses. • In addition to the above, there may be
• The various losses in prestress are losses due to sudden changes in
shown in the following chart. temperature, especially in steam curing
of pretensioned units.
• The rise in temperature causes a partial
transfer of prestress (due to elongation
of the tendons between adjacent units
in the long – line process) which may

© Copyright Reserved by Gateflix.in No part of this material should be copied or reproduced without permission
cause a large amount of creep if the • For small values of μα+kx , the above
concrete is not properly cured. expression can be simplified by the
• If there is a possibility of a change of Taylor series expansion.
temperature between the times of Px = Po [1-(µα+kx)]
tensioning and transfer, the • Thus, for a tendon with single
corresponding loss should be allowed curvature, the variation of the
for in the design. prestressing force is linear with the
distance from the stretching end. The
6.2LOSS OF PRESTRESS DUE TO following figure shows the variation of
FRICTION prestressing force.

• The friction generated at the interface 6.3 LOSS OF PRESTRESSE DUE


of concrete and steel during the ANCHORAGE SLIP
stretching of a curved tendon in a post- • In a post-tensioned member, when the
tensioned member, leads to a drop in prestress is transferred to the concrete,
the prestress along the member the wedges slip through a little distance
from the stretching end. before they get properly seated in the
• The loss due to friction does not occur conical space.
in pre-tensioned members because • The anchorage block also moves before
there is no concrete during the it settles on the concrete.
stretching of the tendons. • There is loss of prestress due to the
• The friction is generated due to the consequent reduction in the length of
curvature of the tendon and the vertical the tendon.
component of the prestressing force. • The total anchorage slip depends on the
• In addition to friction, the stretching type of anchorage system. In absence of
has to overcome the wobble of the manufacture’s data, the following
tendon. The wobble refers to the change typical values for some systems can be
in position of the tendon along the duct. used.
The losses due to friction and wobble EΔ
are grouped together under friction. This loss due to anchorage slip = s
L
• Force in cable at a distance x from
Δ
 
jacking end, after frictional loss – Px =   Es
L
E s = Young modulus of steel in N/mm2
∆ = Anchorage slip in mm
L = Length of cable in mm
Table:- Typical values of anchorage slip
Px = Poe-(µα+kx) Anchorage system Anchorage
Where Px = Prestressing force at a slip
distance x from jacking end. (∆)
P0 = Prestressing force at jacking end. Freyssinet 4 mm
k = coefficient called wobble correction 12-5mm φ strands 6 mm
factor
12-8 mm φ strands 8 mm
μ = Coefficient for friction in curve
α = Cumulative angle in radian through Magnel 1 mm
which the tangent to the cable profile Dywidag system
turned between any two point under
• Since the loss of stress is caused by a
consideration.
definite total amount of shortening. The

© Copyright Reserved by Gateflix.in No part of this material should be copied or reproduced without permission
percentage loss is higher for short • Note that elastic shorting loss
members than for long ones. multiplied by creep co-efficient is equal
• With the long line pretensioning to loss due to creep.
system, the slip at the anchorage is Age at loading Creep co-efficient
normally very small in comparison with 7 days 2.2
28 days 1.6
the length of the tensioned wire and
1 year 1.1
hence is generally ignored.
• While prestressing a short member, due
• Intermediate values of creep coefficient
care should be taken to allow for the
may be interpolated by assuming that
loss of stress due to anchorage slip,
creep coefficient decreases linearly with
which forms a major portion of the total
the log of time in days.
loss.
For example creep coefficient for 15
days as age of loading is
EXAMPLE:
In a post tensioned beam the cable is (2.52 − 1.6) (log10 28 − log10 15)
= 1.6 +
subjected to stress of 1200 N/mm2. If the log10 28 − log10 7
slip at the jacking end is found to be 4mm, • Creep losses are generally 2-3% of
find the percentage loss of stress due to initial prestressing force.
this case if the beam is 25 m long. Take Es
= 210 kN/mm2. EXAMPLE
Solution: A prestressed concrete beam of rectangular
Loss of stress due to anchorage slip section 120 mm wide and 300 mm deep is
Δ  4  prestressed by 6 wires of 6 mm diameter,
 ⨉210 × 10
3
=   Es =  provided at an eccentricity of 55 mm. The
L  25×1000  initial stress in the wires is 1150 N/mm2.
= 33.6 N/mm2 Find the loss of stress in steel due to creep
33.6 of concrete. Take E s =2×105 N/mm 2 ,
%loss = × 100 = 2.8%
1200 E c =3×104 N/mm 2 ,φ =1.50
6.4LOSS OF PRESTRESS DUE TO CREEP Solution : Modular ratio an
= 2 × 104 = 20
OF CONCRETE Es 5
=
E c 3 × 10 3
Creep is the property of concrete by which Area of the beam section A = 120 × 300 =
it continues to deform with time under 36000 mm2.
sustained loading. Moment of Inertia of the beam section,
Creep coefficient is defined as
I = 120 × 300 = 2.70 × 108 mm4.
3

= εcp
creep strain
φ= 12
elastic strain εc π
Prestressing force = P = 6 × × 62× 1150
ε cp =φ ε c ……(1) 4
= 195093 N
εc = fc/Ec
Stress in concrete at the level of steel,
Loss of stress =ε cp ×E s
Pe 2 195093 195093× ( 55 )
2

fc E fc = = +
= φ. ×E s m= s I 36000 2.70×108
Ec Ec
= 7.60 N/mm2.
Loss of stress = m φ fc • Loss of stress in steel due to creep of
concrete= φ m fc = 1.50 20 ×7.60 N/mm2.
3

© Copyright Reserved by Gateflix.in No part of this material should be copied or reproduced without permission
= 76 N/mm2. due to shrinkage of concrete using IS: 1343
recommendations assuming the beam to
6.5LOSS DUE TO SHRINKAGE OF be, (a) pre-tensioned and (b) post-
CONCRETE tensioned. Assume Es = 210 KN/mm2 and
age of concrete at transfer = 8 days.
The shrinkage of concrete in prestressed Solution:
members results in a shortening of  300 ×103 
tensioned wires and hence contributes to Initial stress in wires =  
the loss of stress.  300 
• In the case of pre-tensioned members, = 1000 N/mm2.
generally moist curing is resorted to in
order to prevent shrinkage until the a) If the beam is pre-tensioned, the total
time of transfer. residual shrinkage strain = 300 × 10–6
• Consequently, the total residual units.
shrinkage strain will be larger in ∴ Loss of stress
pretensioned members after transfer of = (300 × 10–6) (210 × 103) = 63 N/mm2.
prestress in comparision with post- ∴ Percentage loss of stress
tensioned members, where a portion of =  
63
×100  = 6.3%
shrinkage will have already taken place  1000 
by the time of transfer of stress. b) If the beam is post-tensioned, the total
• This aspect has been considered in the shrinkage strain
recommendations made by the Indian  200 ×10−6 
standard code (IS: 1343) for the loss of =  = 200 × 10–6 units
prestress due to the shrinkages of  10
log (8 + 2) 
concrete and as detailed below. ∴ Loss of stress
ε cs = total residual shrinkage strain = (200 × 10–6) (210 × 103) = 42 N/mm2.
∴ Percentage loss of stress
having values of 3 × 10−4 for pre-
tensioning and  42 
= ×100  = 4.2%
 1000 
εcs = [ (2⨉10-4)/log10(t+2)]
for post-tensioning
6.6LOSSS OF PRESTRESS DUE TO
Where, t = age of concrete at transfer in
RELAXATION OF STEEL
days.
• The value may be increased by 50
• Relaxation of steel is defined as the
percent in dry atmospheric conditions,
decrease in stress with time under
subject to a maximum value of 3 × 10−4
constant strain.
units.
• Due to the relaxation of steel, the
The loss of stress in steel due to the
prestress in the tendon is reduced with
shrinkage of concrete is estimated as,
time.
Loss of stress = ε cs ×E s
• The relaxation depends on the type of
Where Es = modulus of elasticity of steel, initial prestress (fpi) and the
steel. temperature.
• To calculate the drop (or loss) in
EXAMPLE prestress ( ∆ fp), the recommendations
A concrete beam is prestressed by a cable of IS: 1343 – 1980 can be followed in
carrying an initial prestressing force of 300 absence of test data.
KN. The cross-sectional area of the wires in
the cable is 300 mm2. Calculate the
percentage loss of stress in the cable only

© Copyright Reserved by Gateflix.in No part of this material should be copied or reproduced without permission
Table: Relaxationlosses for prestressing • Short term deflection under uncracked
steel at 1000 H at 270 C (as per IS 1343 – condition in a prestressed beam can be
1980 computed using the elastic theory. In
Initial Stress (1) Relaxation Loss N/mm2 this we account for dead load, live load
0.5 fp 0 (under service load condition) and
0.6 fp 35
moment due to eccentricity of
0.7fp 70
0.8 fp 90 prestressing cable.
Note • BM at any section due to all such
fp is the characteristic strength of loading can be taken and BMD can be
prestressing steel. made. With the help of this, deflection
Final conclusion of above discussions:- can be computed as done in elastic
theory by talking complete x-section to
Sr. Type of loss Equation be effective.
No. • Computation of deflection can be
1 Wobble & curvature ( µα + kx)P0 conveniently done using area moment
effect theorem (Mohr’s theorem.)
2 Anchorage slip Es ∆ /L
3 Shrinkage loss ε Sc. Es 6.7.1 Effect of tendon profile on
4 Creep of concrete m φ fc
deflection
5 Elastic shortening of mfc
concrete
6 Relaxation in steel 2 to 5% for initial • In most of the cases of prestressed
stress in steel. beams, tendons are located with
eccentricities towards the soffit of
Type of loss Pretensione Post beams to counteract the sagging
d (%) tensioned bending moments due to transverse
(%) loads.
Elastic shorting 3 1
of conc. • Consequently, the concrete beam
Shrinkage 7 6 deflect upwards (camber) on the
Creep 6 5 application or transfer of prestress.
Relaxation 2 3 Since the bending moment at every
Total Loss 18% 15% section is the product of the
prestressing force and eccentricity, the
Losses Pretensioning Post tendon profile itself will represent the
tensioning
shape of the BMD.
Length effect No Yes
Curvature effect No Yes • The method of computing deflection of
Anchorage slip No Yes beams with different cable profiles is
Shrinkage of Yes Yes outlined below.
concrete
Creep of concrete Yes Yes 1. Straight tendons :
Elastic Yes No (If all wires
deformation or are
Figure below shows a beam with a straight
shortening of simultaneously tendon at a uniform eccentricity below the
concrete tensioned) Yes centroidal axis,if upward deflection are
(If considered as negative and,
wires are P = effective prestressing force
successively
tensioned)
E = eccentricity
L = length of the beam
6.7 DEFLECTION OF RESTRESSED BEAM a = -(Pe/2)L(L/4)/(EI) = -(PeL2/8EI)

© Copyright Reserved by Gateflix.in No part of this material should be copied or reproduced without permission
4. Parabolic tendons (eccentric anchors):
Fig, below shows a beam, with a parabolic
tendons having an eccentricity e1 at the
2. Trapezoidal tendons : centre of span and e2 at the support
A draped tendon with a trapezoidal profile section.
is shown in Fig. Considering the B.M.D., the • The resultant deflection at the centre is
deflection at the centre of the beam is obtained as the sum of the upward
obtained by taking the moment of area of deflection of a beam with a parabolic
the B.M.D. over one – half of the span. Thus, tendon of eccentricity (e1 + e2) at the
𝑃𝑃𝑃𝑃
a = - ( 𝐸𝐸𝐸𝐸 )[ L2 (L1+ L2/2)+ (L1/2)(2/3L1)] centre and zero at the supports and the
= -(Pe/6EI)(2L12+6L1L2+3L22) downward deflection of a beam
subjected to a uniform sagging bending
moment of intensity Pe2 throughout the
length.
• Consequently, the resultant deflection
becomes,

3. Parabolic tendons (central anchors) :


The deflection of a beam with parabolic  -5 PL2   PeL2 
tendons having an eccentricity e at the
a=  ( e1 +e2 )  +  
 48 EI   8EI 
centre and zero at the support is given by,
PL2
Pe  2 L 5 L   5PeL2  a= ( -5e1 +e2 )
α=-  × × ×  =-   48EI
EI  3 2 8 2   48EI 
5. Sloping tendons (eccentric anchors) :
From fig below the deflection is computed
in a way similar to method 4 above. Thus

© Copyright Reserved by Gateflix.in No part of this material should be copied or reproduced without permission
a) Neglecting all losses, find the deflection
at the centre of span when it is
supporting its own weight.
b) Allowing for 20 percent loss in
prestress, find the final deflection at the
centre of span when it carries an
imposed load of 18 kN/m. Dc = 24
kN/m3.
Solution:
Self-weight of the beam, g = (0.3 ×0.5×24)
 PL2   Pe L2  = 3.6 kN/m = 0.0036 kN/mm
A=  - ( e1 +e2 )  +  2   300×5003 
 12EI   8EI  Second moment of area, (I) =  
PL 2  12 
= ( -2e1 +e2 ) = 3125 × 10 mm
6 4
24EI
Deflections due to self – weight and Prestressing force, P = (2 × 600 × 1600)
imposed loads. = 1920 × 103 N = 1920 kN
Downward deflection due to self-weight =
• At the time of transfer of prestress, the  5gL4   5 × 0.0036 × (10 ×1000) 4 
beam hogs up due to the effect of  = 
 384EI   384 × 38 × 3125 ×10 
6

prestressing.
= 3.95 mm
• At this stage, the self-weight of the Upward deflection due to the prestressing
beam induces downwards deflections, force
which further increase due to the effect
of imposed loads on the beam.  PeL2   1920 × 100 × (10 ×1000) 2 
= =  
 8EI   8 × 38 × 3125 ×10
6
If g = self-weight of the beam/m 
q = imposed load/m (uniformly = 20.2 mm
distributed). Net upward deflection of the beam, when it
The downward deflection is computed support its own wt = (20.2 – 3.9) = 16.3
as, mm
5(g+q)L4 Downward deflection due to live load
a=
3.9 ×18 
384EI =   = 19.5 mm
Deflection due to concentrated live  3.6 
loads can be directly computed by using Net upward deflection due to prestress
Mohr’s theorems. after losses = (0.8 × 20.2) = 16.16 mm. Final
downward deflection of the beam due to
EXAMPLE 1 (self – weight + prestress + live load) =
A concrete beam with a rectangular section (3.95 – 16.16 + 19.50) = 7.29 mm.
300 mm wide and 500 mm deep is
prestressed by 2 post tensioned cables of 6.8 PRESTRESSED CONCRETE
area 600 mm2 each. Initially stressed to • Prestressed concrete is basically a
1600 N/mm2. The cables are located at a concrete in which internal stress of
constant eccentricity of 100 mm suitable magnitude and distribution are
throughout the length of the beam having a introduced so that the stresses resulting
span of 10m. The modulus of elasticity of from external load are counteracted to a
steel and concrete is 210 and 38 kN/mm2. desired degree.
Respectively. • A prestressed concrete is different from
a conventional RCC structure due to the

© Copyright Reserved by Gateflix.in No part of this material should be copied or reproduced without permission
application of an initial load on the • The prestress is transferred to the
structure prior to its use. concrete from the tendons, due to the
• For concrete internal stress are induced bond between them.
(usually by mean of tensioned steel) • The stages of prestensioning are shown
due to the following reasons. schematically in the following figures.
1. Tensile strength of concrete is less and
cracks may develop at early stages of 6.9 REQUIRMENT OF HIGH STRENGTH
loading in flexural members such as STEEL & CONCRETE IN PRESTRESSING
beams and slabs. Hence if the members
are compressed prior to application of 6.9.1 Need for high strength steel
flexural load, the cracking will not occur.
2. Prestressing enhances the bending,
shear and torsional capabilities of the
flexural members.
3. In pipes and liquid storage tanks the
hoop tensile stresses can be effectively
counteracted by circular prestressing.
How do we apply prestress by means of
tensioned wire?
There are two ways (1) Pretensioning (2)
Post tensioning. Prestressed concrete requires high
strength steel and high strength concrete.
6.8.1 Pre-tensioning This can be explained as follows:-
Δ
Strain in cable initially =
ls
Δ
Stress in cable initially = Es
ls

(Force in cable initially s .A s =P
ls
= prestressing force)
δ
(Loss of strain in cable due = to
ls
shortening of concrete over a period of
time)
δ
Loss of stress in cable= E s .
• In pre-tensioning system, the high- ls
strength steel tendons are pulled δ
between two end abutments (also Loss of prestressing force =E s . .A s
ls
called bulkheads) prior to the casting of
Normally due to creep and shrinkage in
concrete.
long term the strain lost is approximately
• The abutments are fixed at the ends of a
0.0008
prestressing bed.
Stress lost = 0.0008 × 2 × 105 = 160
• Once the concrete attains the desired
N/mm2.If we use Fe250 or F415, all of the
strength for prestressing, the tendons
initial stress in it will be lost in due course.
are cut loose from the abutments.
Hence there would not be any
pressuressing force remaining in concrete,
thus the beam will fail.

© Copyright Reserved by Gateflix.in No part of this material should be copied or reproduced without permission
Hence we use high strength steel such that
the initial prestress in it would be 1200 –
2000 N/mm2 in which the loss would be
around 200 N/mm2.

6.9.2 Need for High Strength Concrete


in Pre-stress Concrete

• High strength concrete offers high


resistance to tension, shear, bond and
bearing.
• In case of pre-tensioned members,
tensile stress in steel of very high
magnitude should be transferred to
concrete as prestress through bonding
between steel and surrounding
concrete.
• In post-tensioned members transfer of
stress is through bearing at end sector.
Hence concrete of appreciable bond and
bearing strength is quite essential for
pre-stressed concrete.
• In addition to above, and bearing
strength is quite essential for pre-
stressed concrete.
1) High strength concrete is less liable
to shrinkage cracks and has higher
modulus of elasticity and smaller
ultimate creep strain. As a result
loss of prestress in steel is reduced.
2) Use of high strength concrete results
in reduction of cross sectional
dimensions of prestressed concrete
structural elements, with reduced
dead weight longer spans becomes
economically and practically viable.

© Copyright Reserved by Gateflix.in No part of this material should be copied or reproduced without permission
GATE QUESTIONS
Topics Page No

1. WORKING SRESS & LIMIT STATE METHOD 11

2. SHEAR, TORSION, BOND, ANCHORAGE & DEVELOPMENT LENGHTH 656

3. FOOTING, COLUMNS, BEAMS AND SLABS 44

4. PRESTRESSED CONCRETE 11

© Copyright Reserved by Gateflix.in No part of this material should be copied or reproduced without permission
1 WORKING SRESS & LIMIT STATE METHOD

Q.1 The following two statements are 4. Fineness modulus test


made with reference to a simply 5. Pull out. Lest
supported under reinforced RCC Codes:
beam: A B C D
I. Failure takes place by crushing a) 2 l 5 3
of concrete before the steel has b) 4 5 1 3
yielded. c) 2 1 5 4
II. The neutral axis moves up as the d) 2 5 1 4
load is increased. [GATE-2003]
With reference to the above
statements, which of the following Q.4 For avoiding the limit state of
applies? collapse, the safety of RC structures
a) Both the statements are false is checked for appropriate
b) I is true but II is false combinations of Dead Load (DL),
c) Both the statements are true imposed Load or Live Load (M),
d) I is false but II is true Wind Load (WL,) and Earthquake
[GATE-2001] Load (EL). Which of the following
load combinations is NOT
Q.2 Maximum strains in an extreme considered?
fibre in concrete and in the tension a) 0.9 DL + 1.5 WL
reinforcement (Fe-415 grade and b) 1.5 DL + 1.5 WL
Es-200kN/mm2) in a balanced c) 1.5 DL + 1.5 WL + 1.5 EL
section at limit state of flexure are d) 1.2 DL + 1.2 IL + 1.2 WL
respectively [GATE-2004]
a) 0.0033 and 0.0038
b) 0.002 and 0.0018 Q.5 In the limit state design method of
c) 0.0035 and 0.0041 concrete structure, the
d) 0.002 and 0.0031 recommended partial material
[GATE-2003] safely factor (yn) for steel according
to IS: 436-2000 is
Q.3 List-I contains some properties of a)1.5 b)1.13
concrete cement and List-II contains c)1.00 d)0.87
list of some tests on concrete/cement [GATE-2004]
Match the property with the
corresponding test. Q.6 The partial factor of safety for
List-I concrete as per IS : 456-2000 is
A. Workability of commie a) 1.50 b) 1.15
B.Direct tensile strength of coma-etc c) 0.87 d) 0.446
C. Bond between concrete and steel [GATE-2005]
D. Fineness of cement
List-II Q.7 The flexural strength of M30
1. Cylinder splitting lest concrete as per IS: 456-2000 is
2. Tiee-Bee test a) 3.83 MPa b) 5.47 MPa
3. Surface area test

© Copyright Reserved by Gateflix.in No part of this material should be copied or reproduced without permission
c) 21.23 MPa d) 30.0 MPa 3. Creep and shrinkage are
[GATE-2005] independent of the water –
cement ratio in the concrete mix.
Q.8 In a random sampling procedure for The TRUE statements are
cube strength of concrete, one a) 1 and 2 b) 1, 2 and 3
sample consists of X number of c) 2 and 3 d) only 2
specimens. These specimens are [GATE-2007]
tested at 28 days and average
Strength of these specimens is Q.11 A reinforced concrete structure has
considered as test result of the to be constructed along a sea coast.
sample, provided the individual The minimum grade of concrete to
variation in the strength of be used as per IS : 456-2000 is
specimens is not more than ∓ Y 5 a) M 15 b) M 20
percent of the average strength. The c) M 25 d) M 30
values of X and Y as per IS : 456 – [GATE-2008]
2000 are
a) 4 and 10 respectively Q.12 Un-factored maximum bending
b) 3 and 10 respectively moments at a section of a reinforced
c) 4 and 15 respectively concrete beam resulting from a
d) 3 and 15 respectively flame analysis are 50, 80, 120 and
[GATE-2005] 180 kN-m under dead, live, wind
and earthquake loads respectively.
Q.9 As per IS : 456 – 2000, consider the The design moment (kNm) as per IS:
following statements: 456-2000 for the limit state of
1. The modular ratio considered in collapse (flexure) is
the working stress method a) 195 b) 250
depends on the type of steel c) 345 d) 372
used. [GATE-2008]
2. There is an upper limit on the
nominal shear stress in beams Q.13 The modulus of rapture of concrete
(even with shear reinforcement) terms of its characteristic cube
due to the possibility of crushing compressive strength (fck) in MPa
of concrete in diagonal according to IS 45 6:2000 is
compression. a) 50000 fck b) 0.7 fck
3. A rectangular slab whose length c) 50000 Fck d) 0.7 Fck
is equal to its width may not be a [GATE-2009]
two way slab for some support
conditions. Q.14 For limit state of collapse, the partial
The TRUE statements are safety- factors recommended By IS
a) 1 and 2 b) 2 and 3 456:2000 for estimating the design
c) 1 and 3 d) 1, 2 and 3 strength of concrete and reinforcing
[GATE-2006] steel are respectively
a) 1.15 and 1.5 b) 1.0 and 1.0
Q.10 Consider the following statements: c) 1.5 and 1.15 d) 1.5 and 1.0
1. The compressive strength of [GATE-2009]
concrete decreases with increase Q.15 Match List-I (List of test methods for
in water-cement ratio of the mix. evaluating properties of concrete)
2. Water is added to the for with
hydration of workability.

© Copyright Reserved by Gateflix.in No part of this material should be copied or reproduced without permission
List-II (List of properties) and select ultimate load analysis
the correct answer using the codes a) P – 1; Q – 2; R – 3; S – 4
given below the lists: b) P – 2; Q – 1; R – 4; S – 3
c) P – 3; Q – 4; R – 2; S – 1
List-I d) P – 4; Q – 3; R – 2; S – 1
A. Resonant frequency test [GATE-2014]
B. Rebound hammer test Q19. While designing, for a steel column
C. Split cylinder teat of Fe250 grade, a base plate resting
D. Compacting factor lest on a concrete pedestal of M20 grade,
List - II the bearing strength of concrete (in
1. Tensile strength N/mm2) in limit state method of
2. Dynamic modulus of elasticity design as per IS:456-2000 is _________
3. Workability [GATE-2014]
4. Compressive strength
Codes: Q.20. The first moment of area about the
A B C D axis of bending for a beam cross-
a) 2 4 1 3 section is
b) 2 1 4 3 a) moment of inertia
c) 2 4 3 1 b) section modulus
d) 4 3 1 2 c) shape factor
[GATE-2009] d) polar moment of inertia
[GATE-2014]
Q.16 As per IS 456:2000, in the Limit
Suite Design of a flexural member, Q.21. Group I contains representative
the swain in reinforcing bars under stress-strain curves as shown in the
tension at ultimate state should not figure, while Group II gives the list
be haw than of materials. Match the stress-strain
a) f/E b) f/E + 0.002 curves with the corresponding
c) f/1.15E d) f/1.15E + 0.002 materials.
[GATE-2012]
Q.17 Maximum possible value of
Compacting Factor for fresh
(green) concrete is:
a) 0.5 b) 1.0
c) 1.5 d) 2.0
Group I Group II
[GATE-2013]
(p) Curve J (1) Cement paste
Q18. Match the information given in (q) Curve K (2) Coarse aggregate
Group – I with those in Group II. (r) Curve L (3) Concrete
Group - I Group - II a) P-1; Q-3; R–2 b) P-2; Q-3; R-1
(p) Factor to decrease (1) Upper bound c) P-3; Q-1; R-2 d) P-3; Q-2; R-1
ultimate strength to on ultimate load [GATE-2014]
design strength
(q) Factor to increase (2) Lower bound Q.22. The target mean strength fcm for
working load to on ultimate load concrete mix design obtained from
ultimate load for
design the characteristic strength fck and
(r) Statical method of (3) Material partial standard deviation σ, as defined in
ultimate load analysis safety factor IS:456-2000, is
(s) Kinematical (4)Load factor a) Fck +1.35σ b) Fck +1.45σ
mechanism method of

© Copyright Reserved by Gateflix.in No part of this material should be copied or reproduced without permission
c) Fck +1.55σ d) Fck +1.65σ Q.27 Consider the singly reinforced beam
shown in the figure below:
[GATE-2014]

Q.23 The flexural tensile strength of M25


grade of concrete, in N/mm2, per
IS:456-2000 is ________
[GATE-2014]

Q.24 The modulus of elasticity, E = 5000


 Fck where  Fck is the At cross-section XX, which of the
characteristic compressive strength following statement is TRUE at the
of concrete, specified in IS:456-2000 limit state?
is based on a) The variation of stress is linear
a) tangent modulus and that of strain is non-linear
b) initial tangent modulus b) The variation of strain is linear
c) secant modulus and that of stress is no-linear
d) chord modulus c) The variation of both stress and
[GATE-2014] strain is linear
d) The variation of both stress and
Q.25 In a reinforced concrete section, the strain is non-linear
stress at the extreme fibre in [GATE-2015]
compression is 5.80 MPa. The depth
of neutral axis in the section is 58 Q.28 Consider the following statements
mm and the grade of concrete is for air-entrained concrete:
M25. Assuming linear elastic (i) Air-entrainment reduces the
behavior of the concrete, the water demand for a given level
effective curvature of the section (in of workability
per mm) is (ii) Use of air-entrained concrete is
a) 2.0×10-6 b) 3.0×10-6 required in environments where
c) 4.0×10 -6 d) 5.0×10-6 cyclic freezing and thawing is
[GATE-2014] expected.
Which of the following is TRUE?
Q.26 Workability of concrete can be a) Both (i) and (ii) are True
measured using slump, compaction b) Both (i) and (ii) are False
factor and Vebe time. Consider the c) (i) is True and (ii) is False
following statements for workability d) (i) is False and (ii) is True
of concrete: [GATE-2015]
(i) As the slump increases, the Vebe
time increases Q.29 The composition of an air-entrained
(ii) As the slum increases, the concrete is given below:
compaction factor increases Water : 184kg/m3
Which of the following is TRUE? Ordinary Portland Cement(OPC) :
a) Both (i) and (ii) are True 368 kg/ m3
b) Both (i) and (ii) are False Sand : 606kg / m3
c) (i) is True and (ii) is False Coarse aggregate : 1155 kg/m3
d) (i) is False and (ii) is True Assume the specific gravity of OPC,
[GATE-2015] sand and coarse aggregate to be

© Copyright Reserved by Gateflix.in No part of this material should be copied or reproduced without permission
3.14, 2.67 and 2.74, respectively.
The air content is _________liters/ m3.
[GATE-2015]

Q.30 Consider the singly reinforced beam Q.34 Let the characteristic strength be
section given below (left figure). The defined as that value, below which
stress block parameters for the not more than 50% of the results are
cross-section from IS:456-2000 are
expected to fall. Assuming a
also given below (right figure). The
moment of resistance for the given standard deviation of 4 MPa, the
section by the limit state method is target mean strength (in MPa ) to be
__________kN-m. considered in the mix design of a
M25 concrete would be

a) 18.42 b) 21.00
c) 25.00 d) 31.58
[GATE-2017]

Q.35 Group I gives a list of test methods


and test apparatus for evaluating
some of the properties of Ordinary
[GATE-2015] Cement (OPC) and concrete. Group
Q.31 According to the concept of Limit II gives the list of these properties
State Design as per IS456: 2000, the
probability of failure of a structure Group I Group II
is ________ P. Le Chatelier test 1. Soundness of OPC
[GATE-2015]
Q. Vee- Bee test 2. Consistency and
Q.32 A reinforced concrete (RC) beam setting time of OPC
with width of 250 mm and effective R. Blaine air 3. Consistency or
depth of 400 mm is reinforced with permeability test workability of concrete
Fe415 steel. As per the provisions of
IS 456-2000, the minimum and S. The Vicat apparatus 4. Fineness of OPC
maximum amount of tensile
reinforcement (expressed in mm2)
The correct match of the items in Group I
for the section are, respectively
with the items in Group II is
a) 250 and 3500 b) 205 and 4000
c) 270 and 2000 d)300 and 2500 A) P-1, Q-3, R-4, S-2 C) P-4,Q-2, R-4, S-1
[GATE-2016] B) P-2, Q-3, R-1, S-4 D) P-1,Q-4, R-2, S-3
[GATE-2017]
Q.33 For M25 concrete with creep
coefficient of 1.5, the long-term
static modulus of elasticity Q.36 According to IS 456:2000, which one
(expressed in M Pa) as per the of the following statements about the
provisions of IS:456-2000 is _____ depth of neutral axis xu,bal for a
[GATE-2016]
balanced reinforced concrete section
is correct?

© Copyright Reserved by Gateflix.in No part of this material should be copied or reproduced without permission
A) xu,bal depends on the grade of concrete supported at two opposite ends on
only. 230 mm thick masonry walls. The
B) xu,bal depends on the grade of steel only. centre-to-centre distance between
the walls is 3.3 m. As per IS
C) xu,bal depends on both the grade of
456:2000, the effective span of the
concrete and grade of steel . slab (in m, up to two decimal places)
D) xu,bal does not depend on both the is __________
grade of concrete and grade of steel .
[GATE-2017] [GATE-2018]

Q.42 A singly-reinforced rectangular


Q.37 The Le Chatelier apparatus is used
concrete beam of width 300 mm and
to determine
a) compressive strength of cement effective depth 400 mm is to be
b) fineness of cement designed using M25 grade concrete
c) setting time of cement and Fe500 grade reinforcing steel.
d) soundness of cement For the beam to be under-
[GATE-2018] reinforced, the maximum number of
16 mm diameter reinforcing bars
Q.38 The setting time of cement is
that can be provided is
determined using
a) Le Chatelier apparatus a) 3 b) 4
b) Briquette testing apparatus c) 5 d) 6
c) Vicat apparatus [GATE-2018]
d) Casagrande’s apparatus
[GATE-2018] Q.43 Two rectangular under-reinforced
concrete beam sections X and Y are
Q.39 The deformation in concrete due to similar in all aspects except that the
sustained loading is longitudinal compression
a) creep reinforcement in section Y is 10%
b) hydration more. Which one of the following is
c) segregation the correct statement?
d) shrinkage a) Section X has less flexural
strength and is less ductile than
[GATE-2018] section Y
b) Section X has less flexural
Q.40 As per IS 456:2000, the minimum strength but is more ductile than
percentage of tension reinforcement section Y
(up to two decimal places ) required c) Section X and Y have equal
in reinforced- concrete beams of flexural strength but different
rectangular cross-section ( ductility
considering effective depth in the d) Section X and Y have equal
calculation of area ) using Fe500 flexural strength and ductility
grade steel is _______________ [GATE-2018]
[GATE-2018]
Q.44 The frequency distribution of the
Q.41 A reinforced-concrete slab with compressive strength of 20 concrete
effective depth of 80 mm is simply
cube specimens is given in the table

© Copyright Reserved by Gateflix.in No part of this material should be copied or reproduced without permission
f(Mpa) Number of
specimens with
compressive
strength equal to f
23 4
28 2
22.5 5
31 5
29 4

If μ is the mean strength of the


specimens and σ is the standard
deviation, the number of Specimens
(out of 20) with compressive
strength less than μ-3σ is ________

[GATE-2018]

ANSWER KEY:
1 2 3 4 5 6 7 8 9 10 11 12 13 14
(b) (a) (a) (c) (c) (b) (d) (d) (d) (a) (d) (d) (a) (a)
15 16 17 18 19 20 21 22 23 24 25 26 27 28
(c) (b) (b) (c) 9 (b) (b) (d) 3.5 (b) (c) (d) (b) (a)
29 30 31 32 33 34 35 36 37 38 39 40 41 42
51 42.82 0.097 (b) 10000 (c) (a) (b) (d) (c) (a) 0.17 3.15m (c)
43 44
(a) zero

© Copyright Reserved by Gateflix.in No part of this material should be copied or reproduced without permission
EXPLANATIONS

Q.1 (b) = 0.7 f ck

(a) Q.2
Q.14 (a)
εc =0.0035

E steel =
fy
+ 0.002 Q.15 (c)
1.15E 5
415 Q.16 (b)
= + 0.002
1.15 × 200 ×103
Q.17 (b)
= 0.0038
Compaction factor = weight of
partially compacted concrete/
Q.3 (a)
weight of fully compacted concrete
For full workability weight of
Q.4 (c)
partially compacted concrete will be
equal to weight of fully compacted
Q.5 (c)
concrete, therefore maximum
possible value of compaction factor
Q.6 (b)
is equal to 1
Q.7 (d)
Q.18. (c)
Static method → Upper bound
Q.8 (d)
method of ultimate load analysis
Refer : IS : 456 – 2000 clause 15.3
Kinematic method → Lower bound
on ultimate load QDesign = Qw × load
Q.9 (d)
factor M0 = Mu × Ym' Ym = material
partial safety factor
Q.10 (a)
Q.19 (9 to 9)
Q.11 (d)
Permissible bearing strength = 0.45
Q.12 (d) fck = 0.45 × 20 = 9N / mm 2
Load combinations
(1) 1.5 (DL + LL) Q.20 (b)
= 1.5 (50 + 80) = 195 kN-m I A.r 2
Section modulus, z =  = =A.r ,
(2) 1.5 (DL + EQ) r r
= 1.5 (50 + 180) = 345 kN-m i.e. Moment of Area.
(3) 1.2 (DL + LL + EQ)
= 1.2 (50 + 80 + 180) Q.21 (b)
= 372 kN-m So, P = 2,
Use maximum of above Q=3
combination. R=3

Q.13 (a)

© Copyright Reserved by Gateflix.in No part of this material should be copied or reproduced without permission
Q.28 (a)
Q.22 (d) (i) Air-entrainment reduces the
water demand for a given level
f m = f ck + 1.65σ (As per IS : of workability-True
456.2000) (ii) Use of air-entrained concrete is
required in environments where
Q.23 (3.5 to 3.5) cyclic freezing and thawing is
Flexural tensile strength, expected. -True
f cr = 0.7 fck N/mm 2
Q.29 (51)
= 0.7 × 25
Mc Ms Ma
= 3.5 N/ mm 2 + + +Vw+Va = 1
Pc Ps Pa
368 606 1155
Q.24 (b) ⇒ + +
Initial tangent modulus. 3.14×1000 2.67×1000 2.74×1000
184
+ + Vv = 1.0
Q.25 (c) 1000
⇒ 0.117 + 0.227 + 0.421 + 0.184 + VV  1.0
=
⇒ VV =
0.051
= 0.051×1000 = 51 ; 50.32 1/ m3

Q.30 (42.82)
π
A st = 4× × (12 ) = 453mm 2
2
E = 5000 =
f ck 5000 ×   25 = 25000N/mm 2 4
M F E
= = 0.36f ck b.x u = 0.87f y A st
I Y R 0.87 × 415 × 453
0.87f y A st
Curvature, ⇒ Xv =
1 F 5.8 0.36f ck .b 0.36 × 25 × 200
= = = 4 ×1 06 per mm = 90.86mm
R yE 58×25000
X v,max = 0.48d
F 5.8
= = 4 ×1 06 per mm =0.4 × 300 =120mm
yE 58×25000
 X v < Xv, max so U.R. section
Q.26. (d) M v = 0.87 × f y × A st × (d - 0.42X v )
As the slump increases, the Vebe = 0.87 × 415 × 453 × ( 300 – 42 × 90.86 ) = 42.82kNm
time decreases
Q.31 0.097
Q.27 (b)
At
Q.32 (b)

Q.33 (10000)

© Copyright Reserved by Gateflix.in No part of this material should be copied or reproduced without permission
Q.34 (c) 25MPa 0.36×25×300×1.84
=
If fck is the value below which not more 0.87×500
than 50% of test results are expected then
fm = fck = 1142.06 𝑚𝑚𝑚𝑚2
(Mean strength=Characteristics strength)
So target mean strength of concrete to be No.of bars for limited area of steel
considered in design mix = Mean strength 1142.06
= 𝜋𝜋 2
fm = fck = 25MPa 16
4

Q.35 (A) = 5.68


So number of bars will be 5.
Q.36 (b)
Q.43 (a)
Q.37 (d) Q.44 Mean Strength
Q.38 (c) µ=
(23×4)+(28×2)+(22.5×5)+(31×5)+(29×4)
Q.39 (a)
20
µ = 26.575MPa
Q.40 Ast/bd = (0.85/fy)
Ast/bd = (0.85/500) n= 20
Minimum Percentage of tension
reinforcement = (Ast/bd)⨉100 ∑(f−μ)2
Standard deviation = σ = �
= (0.85⨉100)/500 (n−1)
= 0.17 𝜎𝜎 =
4(23−26.575)2 +2(28−26.575)2 +5(22.5−26.575)2 +5(31−26.575)2 +4(29−26.575)2

Q.41 effective depth (d) = 80mm 19

lc/c = 3.3 m
Width of support = 230 mm
51.1225+4.06125+83.028+97.903+23.5225
lclear = 3.3-(0.230/2)-(0.230/2) =� 19
= 3.07 m
𝑙𝑙 + 𝑤𝑤
leff = min � 𝑜𝑜 = 3.697 MPa
𝑙𝑙𝑜𝑜 + 𝑑𝑑
𝜇𝜇 − 3𝜎𝜎 = 26.575 − 3(3.697)
3.07 + 0.230 = 3.30 = 15.485MPa
leff = min � Hence No. of specimen with compressive
3.07 + 0.08 = 3.15
strength < 15.485 MPa are zero.
= 3.15 m

Q.42 (c)
xu,lim for Fe500 = 0.46⨉400 = 184
mm
For section to be under-reinforced
Ast ≤ Ast,lim

0.36fck bxu,lim
Ast,lim =
0.87fy

© Copyright Reserved by Gateflix.in No part of this material should be copied or reproduced without permission
2 SHEAR, TORSION, BOND, ANCHORAGE & DEVELOPMENT LENGHTH

Q.1 Consider the following two c) 209 kN-m d) 213 kN-m


statement.' related to reinforced [GATE-2004]
concrete design and identify
whether they are TRUE/FALSE:
I. Curtailment of bars in the flexural Common data for Q.4 & Q.5
tension zone in beams reduces the A reinforced con to beam of rectangular
shear strength at the cut-off crass section of breadth 230 mm and
locations effective depth 400 mm is subjected to a
II. When a rectangular column maximum factored shear force of 120 kN.
section is subject to will be eccentric The grades of concrete, main steel and
compression. The neutral axis will stirrup steel are M20, Fe415 and Fe250
be parallel to the resultant axis of respectively. For the area of main steel
banding. provided, the design shear strength 𝜏𝜏e as
a) Both statements I and II are per IS: 456-2000 is 0.48 N/mm2. The beam
TRUE. is designed for collapse limit state.
b) Statement I is TRUE and
Statement ll is FALSE Q.4 The spacing (nun) of 2 – legged 8
c) Statement I is FALSE and mm stirrups 10 be provided Is
statement II is TRUE a)40 b)115
d) Both Statements I and II an c)250 d)400
FALSE [GATE-2008]
[GATE-2001] Q.5 In addition, the beam is subjected to
a torque whose factored value is
Common data for Questions 2 and 3 10.90 kN-m. The stirrups have to be
At the limit state of collapse, an RC beam is provided to carry a shear (kN) equal
subjected to flexural moment 200 KN-m, to
shear force 20 KN and torque 9 KN-m. The a)50.42 b)130.36
beam is 300 mm wide and has a gross c)151.67 d)200.23
depth of 425mm with an effective cover of [GATE-2008]
25 mm. The equivalent nominal shear
stress (τve ) as calculated by using the Q.6 In the design of a reinforced
concrete beam the requirement for
design code turns out to the lesser than the
bond is not getting satisfied. The
design shear strength (τc ) of the concrete. economical option to satisfy the
requirement for bond is by
Q.2 The equivalent shear force ( Vc ) is
a) Bundling of bars
a) 20 kN b) 54 kN b) Providing smaller diameter bars
c) 56 kN d) 68 kN more in number
[GATE-2004] c) Providing larger diameter bars
less in number
Q.3 The equivalent flexural moment
d) Providing same diameter bars
(M el ) for designing the longitudinal
more in number
tension steel is [GATE-2008]
a) 187 kN-m b) 200 kN-m

© Copyright Reserved by Gateflix.in No part of this material should be copied or reproduced without permission
Q.7 Consider a reinforcing bar [GATE-2014]
embedded in concrete. In a marine
environment this her undergoes Q.9 The development length of a
uniform corrosion which leads to deformed reinforcement bar can be
the deposition of corrosion products expressed as (1/k)(φσs /τbd ). From
on its surface and an increase In the the IS:456-2000, the value of k can
apparent volume of the bar. 'This be calculated as___ .
subjects the surrounding concrete to [GATE-2015]
expansive pressure. As a result,
corrosion induced cracks appear at Q.10 In shear design of an RC beam, other
the surface of concrete. Which of the than the allowable shear strength of
following statements is 'TRUE? concrete (τ c ), there is also an
a) Corrosion onuses circumferential additional check suggested in IS
tensile stresses in concrete and 456-2000 with respect to the
the creeks will be parallel to the maximum permissible shear stress
corroded reinforcing bar. (τ c max ) . The check for τ c max max is
b) Corrosion causes radial tensile
required to take care of
stresses in concrete and the
a) additional shear resistance from
cracks will be parallel to the
reinforcing steel
corroded reinforcing bar.
b) additional shear stress that
c) Corrosion causes circumferential
comes from accidental loading
tensile stresses in concrete and
c) possibility of failure of concrete
the cracks will he perpendicular
by diagonal tension
to the direction of the corroded
d) possibility of crushing of
reinforcing bar.
concrete by diagonal
d) Corrosion causes radial tensile
compression
anemia in concrete and the
[GATE-2016]
cracks will be perpendicular to
the direction of the corroded Q.11 As per IS 456-2000 for the design of
reinforcing bar. reinforced concrete beam, the
[GATE-2009] maximum allowable shear stress
(τcmax ) depends on the
Q.8 A rectangular beam of width (b) 230
mm and effective depth (d) 450mm a) grade of concrete and grade of
is reinforced with four bars of 12 steel
mm diameter. The grade of concrete b) grade of steel only
is M20 and grade of steel is Fe500. d) grade of concrete & percentage
Given that for M20 grade of concrete of reinforcement
the ultimate shear strength, [GATE-2016]
τ∞ =0.36N / mm 2 for steel percentage,
p=0.25, and τ∞  0.48N
= / mm 2 p = Q.12 An RCC beam of rectangular cross
0.50. For a factored shear force of section has factored shear of 200 kN
45kN, the diameter (in mm) of at its critical section. Its width b is
Fe500 steel two legged stirrups to 250 mm and effective depth d is 350
be used at spacing of 375 mm,
mm. Assume design shear strength
should be
τc of concrete as 0.62 N/mm2 and
a) 8 b) 10 maximum allowable shear stress τc,
c) 12 d) 16 max in concrete as 2.8 N/mm2. If

© Copyright Reserved by Gateflix.in No part of this material should be copied or reproduced without permission
two legged 10 mm diameter vertical
stirrups of Fe250 grade steel are
used, then the required spacing (in
cm, up to one decimal place) as per
limit state method will be _________

[GATE-2018]

ANSWER KEY:
1 2 3 4 5 6 7 8 9 10 11 12
(b) (d) (b) (b) (c) (b) (c) (a) 6.4 (d) (b) 8.2 cm

© Copyright Reserved by Gateflix.in No part of this material should be copied or reproduced without permission
EXPLANATIONS

Q.1 (b)
Q.2 (d)
Tu
Ve = Vu + 1.6
b
1.6 ( 9 )
= 20 + = 68 kN
300×10-3
Q.3 (b) Vu 45×1000
==τv = 0.434 N/mm 2
Equivalent bending moment for the bd 230×450
design of tension steel Me1=200 kN-m % tensile reinforcement (p)
π
Q.4 (b) 4× ×(12) 2
= 4 =×100 0.437%
Design shear force for vertical
230×450
Stirrups Vus = VU— τ c bd 0.12
τ c = 0.36 + × (0.437 − 0.25)
= 120 ×103 − 0.48 × 230 × 400 = 44.16 kN 0.25
Spacing for vertical stirrups = 0.45 N/mm2
 π  τ v < τc
0.87 ( 250 )  2 × × 82  ( 200 )
VUS =  4  So, minimum shear reinforcement is
44.16 ×10 3
required
= 115 mm c/c. Minimum shear reinforcement
A sv 0.4 0.4×230×375
Q.5 (c) = ⇒ A sv =
b×Sv 0.87f y 0.80×f y
1.6Tu
V= Vu + π 0.4×230×375
e
b ⇒ 2× ×f 2 =
4 0.80×500
1.6 ×10.9 ×103
= 120 × 10 +3
=196 KN ⇒ φ = 7.10mm
230 So, adopt φ =8 mm
Design shear force, Vus= Ve  − cbd
= 196 ×103 − 0.48 × 330 × 400 Q.9 (6.4)
= 151.67 kN − m φ σs
Ld = But for deformed bars
Q.6 (b) 4τ bd
If bond is not safe it is better τ bd is increased by 60%.
provide smaller diameter bars more So,
in no. φ σst φ σs
Ld = =
Q.7 (c) 4×1.6×τ bd 6.4τ bd
So, k = 6.4
Q.8 (a)
Q.10 (d)

Q.11 (b)
By IS 456:2000

© Copyright Reserved by Gateflix.in No part of this material should be copied or reproduced without permission
τc max = 0.62 f ck
τa max depends on grade of concrete
only.

Q.12 Given, Vu = 200kN, 𝜏𝜏c = 0.62 MPa,


𝜏𝜏𝑐𝑐 𝑚𝑚𝑚𝑚𝑚𝑚 = 2.8 MPa
Vu 200×103
τv = = = 2.286 MPa
bd 250×350
As 𝜏𝜏𝑣𝑣 < 𝜏𝜏𝑐𝑐 𝑚𝑚𝑚𝑚𝑚𝑚
And design shear force = (𝜏𝜏𝑣𝑣 − 𝜏𝜏𝑐𝑐 ) bd
= (2.286-0.62) × 250 × 350
= 145.775 kN
Spacing of shear reinforcement = 𝑆𝑆𝑣𝑣
π 350
Vus = 145775 = 0.87 × 250 × 2 × × 102 ×
4 Sv
Hence Sv = 82.03 mm
Spacing for minimum shear reinforcement

Asv 0.4

bSv 0.87fy

Sv ≤ 341.65 mm
Spacing should be minimum of
(i) 0.75d = 26.25 cm
(ii) 8.2 cm
(iii) 34.16 cm
(iv) 30 cm
So spacing will be 8.2 cm.

© Copyright Reserved by Gateflix.in No part of this material should be copied or reproduced without permission
3 FOOTING, COLUMNS, BEAMS AND SLABS

Q.1 A concrete column carries axial load Q.4 An RC short column with 300mm x
or 450 kN and a bending moment of 300mm square cross-section is
60 kN-m its base. An isolated footing made of M 20 grade concrete and
of size 2m × 3m side along the plane has 4 numbers. 20 mm diameter
of the bending moment is provided longitudinal bars of Fe-415 steel. It
'under the column Centers of gravity is under the action of a concentric
of column and footing coincide. The axial compressive load. Ignoring the
net maximum and the minimum reduction in the area of concrete
pressures in kN/m2 on soil under due to steel bars, the ultimate axial
the Coating are respectively. load carrying capacity of the column
a) 95 and 55 b) 95 and 75 is
c) 75 and 55 d) 75 and 75 a) 1659 kN b) 1548 kN
[GATE-2003] c) 1198 kN d) 1069 kN
[GATE-2004]
Common data for question 2 and 3:
A reinforced concrete beam, size 200 mm Q.5 A rectangular column section of 250
wide and 300 mm deep overall is simply mm x 400 mm is reinforced with
supported over a span of 3 m. It is five stool bars of grade Fe-500, each
subjected to two point loads P of equal of 20 mm Diameter. Concrete mix is
magnitude placed at middle third points. M30. Axial load on the column
The two leads are gradually increased section with minimum eccentricity
simultaneously. Beam is reinforced with 2 as per IS:456-2000 using limit state
HYDC bars of 16mm diameter placed at an method can be applied up to
effective cover of 40 mm on bottom face a) 1707.37 b) 1805.30
and nominal shear reinforcement. The c) 1806.40 d) 1903.7
characteristic compressive strength and [GATE-2005]
the bending tensile strength of the concrete
are 20.0 N/mm2 and d2.2 N/mm2 Common data for question 6 and 7:
respectively. A single reinforced rectangular concrete
beam has a width of 150 mm and an
Q.2 Ignoring the presence of tension effective depth of 330 mm. The
reinforcement, the value of load P in characteristic compressive strength of
kN when the first flexure crack will concrete is 20 MPa and the characteristic
develop in the beam is tensile strength of steel is 415 MPa. Adopt
a) 4.5 b) 5.0 the stress block for concrete as given in LS:
c) 6.6 d) 7.5 456-2000 and take limiting value of depth
[GATE-2003] of neutral axis as 0.48 times the effective
depth of the beam.
Q.3 The theoretical failure load of the
beam for attainment of limit state of Q.6 The limiting value of the moment of
collapse in flexure is resistance of the beam in kN-m is
a) 23.7 EN b) 25.6 KR a)0.14 b)0.43
c) 28.7 kN d) 31.6 kN c)45.08 d)156.82
[GATE-2003] [GATE-2007]

© Copyright Reserved by Gateflix.in No part of this material should be copied or reproduced without permission
Q.7 The limiting area of tension steel in c) 236.3 d) 273.6
mm2 is 0.004 m along the span, the
-1

a) 473.9 b) 412.3 maximum deflection (in m) of the


c) 373.9 d) 312.3 beam at mid-span is _______.
[GATE-2007] [GATE-2015]
Q.8 A reinforced concrete column
contains longitudinal steel equal to Q.13. A haunched (varying depth)
1 percent of net cross-sectional area reinforced concrete beam is simply
of the column. Assume modular supported at both ends, as shown in
ratio as 10. The loads carried (using the figure. The beam is subjected to
the elastic theory) by the a uniformly distributed factored
longitudinal steel and the net area of load of intensity 10 kN/m. The
concrete. are Ps and Pc respectively. design shear force (expressed in kN)
The ratio Ps/Pc expressed as per at the section X-X of the beam is
cent is _______.
a)0.1 b)1 [GATE-2016]
c)1.1 d)10
[GATE-2008] Q.14 An RCC short column (with lateral
ties) of rectangular cross section of
Statements for linked answer question 9 250 mm ⨉ 300 mm is reinforced
and 10:
with four numbers of 16 mm
A doubly reinforced rectangular concrete
beam has a width of 300mm and an diameter longitudinal bars. The
effective depth of 500mm. The beam is grades of steel and concrete are Fe
reinforced with 2200 mm2 of steel in 415 and M20, respectively. Neglect
tension and 628 mm2 of steel in eccentricity effect. Considering limit
compression. The effective cover for state of collapse in compression (IS
compression steel is 50mm. Assume that 456: 2000), the axial load carrying
both tension and compression steel yield.
capacity of the column (in kN, up to
The grades of concrete and steel used are
M20 and re250 respectively. The stress one decimal place), is _________
block parameters (rounded off to first two
[GATE-2018]
decimal places) for concrete shall be as per
IS 456:2000
Q.15 A structure member subjected to
compression has both translation
Q.9 The depth of neutral axis is
and rotation restrained at one end,
a) 205.30 man b) 184.56 mm
while only translation is restrained
c) 160.91 mm d) 145.30 mm
at the other end. As per IS 456:2000,
[GATE-2010]
the effective length factor
recommended for design is
Q.10 The moment of resistance of the
a) 0.50 b) 0.65
section is
c) 0.70 d) 0.80
a) 206.00 kN-m b) 209.20 kN-m
[GATE-2018]
c) 237.80 kN-m d) 231.90 kN-m
[GATE-2010]

Q.11 The value of xu (in mm) computed


per the Limit State Method of IS
456:2000 is
a) 200.0 b) 223.3

© Copyright Reserved by Gateflix.in No part of this material should be copied or reproduced without permission
ANSWER KEY:
Q.12 A simply supported reinforced
1
concrete 2 3 length
beam of 4 10 m5 sags6while 7 8 9 10 11 12 13 14 15
(d) (c) shrinkage.
undergoing (d) (d)Assuming
(a) a(c) (b)
uniform (d) (c) (b) (c) 0.0005 65 918.08 (d)
curvature of

© Copyright Reserved by Gateflix.in No part of this material should be copied or reproduced without permission
EXPLANATIONS

Q.1 (d)
Pu = 0.4 × 30 × 98429.2 + 0.67
Q.2 (c) × 500 ×1570.8 =
1707.3716
F M
=
Y l Q.6 (c)
M u lim = 0.36 f ck bx u,max (d-0.42 x u,max )
 200×3002 
M = f.Z = (2.2)   x u,max = 0.48d
 6 
M = P (1m) Mu lim= 0.36 × 20 ×150 × ( 0.48 330
× )
P = 6.6 kN ( 330 − 0.42 × 0.48 × 330 )
= 45 kN -m
Q.3 (d)
x=u max =
0.48 d 124.8 mm Q.7 (b)
C=T
0.36 f ck b x u = 0.87 f y Ast Q.8 (d)
Longitudinal steel, As = 1% of Ac
0.36 ( 20 )( 200 ) (x u )
Where Ac = Area of concrete
 π  Modular ratio, m = 10
= 0.87 ( 415 )  2 × ×162 
 4  Pl Pl
= c c = s s (here lc =ls )
x u = 100.8 mm A c E c As Es
X u < x u max , under reinforced Ps E s A s
= .
section Pc E c A c
=Mu 0.87 fy Ast(d − 0.42x u ) 1%Ac
=m.
= 31.6 kN -m Ac
Mu= P × 1m = 10 × 1
P = 31.6 kN P
∴  s = 10
Pc
Q.4 (d)
Pu = 0.4 fck Ac + 0.67 fy Asc Q.9 (c)
= 0.4 ( 20 )( 300 × 300 ) + 0.67 ( 415 ) 0.36f ck bx u + f sc Asc =
0.87f y ASt
 π 2 = 0.36 × 20 × 300 × x u 0.87 × 250
 4× ×20  = 1069kN
 4  ×628 × 0.87 × 250 × 2200
x u = 160.91mm
Q.5 (a)
Pu = 0.4 fck Ac + 0.67 fy Asc Q.10 (b)
= Ag − Asc
Ac = Mu 0.36f ck bx u (d − 0.42x u )
π +f sc Asc (d-d1 )
= 250 × 400 − 5 × × 202
4 = 0.36 × 20 × 300 ×160.91(500 − 0.42
= 98489.2 mm 2
×160.91) + 0.87 × 250 ( 628 )( 500 − 50 )
= 209.21 kN-m.

© Copyright Reserved by Gateflix.in No part of this material should be copied or reproduced without permission
neglecting eccentricity effect
Q.11 (c) = 0.45fckAc + 0.75fyAsc
π
Ac = 250 × 300- 4× × 162
Q.12 (0.0005) 4
= 74195.75 mm2
π
Asc = 4× 4 × 162 = 804.25 mm2
Axial load capacity =
(0.45⨉20⨉74195.75)+
(0.75⨉415⨉804.25)
= 918084 N
= 918.08 kN

Q.15 (d)
Recommended value of effective
2 length = 0.8ℓ
1
( 250 )
2-  =249.9995m
OA 2

∆AA’ =
0.0005m

Q.13 (65)
M
Vx ± .tanβ
τv = d
bd
Mx
τ v .bd=Vx ±
⇒ Vd = tanβ
dx
Vx =100-10×5=50kN;dx=500mm
5×5
M x =100×5-10× =375kN-m
2
375
Vd = 50 + × tanβ
0.5
600-400 200
=
tan β =
10×1000 10,000
375 200
Vd = 50+ × = 50+15 = 65kN
0.5 10,000

Q.14 918.08 kN
Considering limit state of collapse in
compression and the permissible
axial strain as 0.002,
the stress in concrete = 0.45fck and
stress in steel = 0.87fy for Fe250
=0.79fy for Fe415
= 0.75fy for Fe500
The above stresses in steel are
derived from stress strain curve of
different grade of steel.
The axial load capacity of column

© Copyright Reserved by Gateflix.in No part of this material should be copied or reproduced without permission
4 PRESTRESSED CONCRETE

Q.1 A simply supported prestressed a) 2.5 N/mm2 (compression), 10


concrete beam is 6 m long and 300 N/mm2 (compression)
mm wide. Its gross depth is 600 mm. b) 10 N/mm2 (tension), 2.5 N/mm2
It is prestressed by horizontal cable (compression)
tendons at a uniform eccentricity of c) 3.75 N/mm2 (tension), N/mm2
100 mm. The prestressing tensile (compression)
force in the cable tendons is 1000 d) 2.75N/mm2 (compression), 3.75
kN. Neglect the self-weight of beam. (compression)
The maximum normal compressive [GATE-2007]
stress in the beam at transfer is
a)12.5 N/mm2 Q.5 A pre-tensioned concrete member of
b)5.55N/mm2 section 200 mm × 250 mm contains
c)11.11 N/mm2 d)15.68 N/mm2 tendon of area 500 mm2 at centre of
[GATE-2004] gravity of the section. The pre stress
in the tendons is 1000 N/mm2.
Q.2 A concrete beam of rectangular Assuming modular ratio as 10, the
cross section of 200 mm × 400 mm stress (N/mm2) in concrete
is pre stressed with a force 400 kN a)11 b) 9
at eccentricity 100 mm. The c) 7 d)5
maximum compressive stress in the [GATE-2008]
concrete
a) 12.5 N/mm2 b) 7.51N/mm2 Q.6 A rectangular concrete beam of
c) 5.0 N/mm2 d) 2.5 N/mm2 width 120 mm and depth 200 mm is
[GATE-2005] prestressed by pretension in to a
Q.3 The percentage loss of prestress due force of 150 kN at an eccentricity: of
to anchorage slip of 3 mm in a 20 mm. The cross-sectional area of
concrete beam of length 30 m which the prestressing steel is 87.5
is post-tensioned by a tendon with mm2.Take modulus of elasticity steel
an initial stress of 1200 N/mm2 and and concrete as 2.1 × 105 MPa and
modulus of elasticity equal to 3.0 × 104 MPa respectively The
2.1×105 N / mm 2 percentage loss of stress in the
a) 0.0175 b) 0.175 prestressing steel due to elastic
c) 1.75 d) 17.5 deformation of concrete is
[GATE-2007] a) 8.75 b) 6.125
c) 4.81 d) 2.19
Q.4 A concrete beam of rectangular [GATE-2009]
cross-section of size 120 mm
(width) and 200 mm(depth) is Q.7 A concrete beam prestressed with a
prestressed by a Straight tendon to parabolic tendon is shown in the
an effective force of 150 kN at an sketch. The eccentricity of the
eccentricity of 20 mm (below the tendon is measured from the
centroidal axis in the depth centroid of the cross-section. The
direction) The stresses at the top applied prestressing force at service
and bottom fibres of the section are is 1620 kN. The uniformly
distributed load of 45 kN/m

© Copyright Reserved by Gateflix.in No part of this material should be copied or reproduced without permission
includes the self-weight. The stress [GATE-2017]
(in N/mm2) in the bottom fibre at
mid span is
Q.10 A simply supported rectangular
concrete beam of span 8 m has to be
prestressed with a force of 1600 kN.
The tendon is of parabolic profile
having zero eccentricity at the
supports. The beam has to carry an
a)tensile 2.90
b)compressive 2.90 external uniformly distributed load
c)tensile 4.32 of intensity 30 kN/m. Neglecting the
d)compressive 4.32 self-weight of the beam, the
[GATE-2012] maximum dip (in meters, up to two
Q.8 In a pre-stressed concrete beam decimal places) of the tendon at the
section shown in the figure, the net mid-span to balance the external
loss is 10% and the final pre- load should be _____________
stressing force applied at X is 750
kN. The initial fibre stresses (in [GATE-2017]
N/mm2) at the top and bottom of
the beam were:
Q.11 A 6 m long simply-supported beam
is prestressed as shown in the
figure.

The beam carries a uniformly


a)4.166 and 20.833
b)-4.166 and -20.833 distributed load of 6 kN/m over its
c)4.166 and -20.833 entire span. If the effective flexural
d)-4.166 and 20.833 rigidity EI = 2⨉104 kNm2 and the
[GATE-2015] effective prestressing force is 200
kN, the net increase in length of the
Q.9 A pre-tensioned rectangular concrete prestressing cable (in mm, up to two
beam 150 mm wide and 300 mm decimal places) is_____________
depth is prestressed with three
straight tendons, each having a [GATE-2018]
cross-sectional area of 50 mm2, to an
initial stress of 1200N/mm2. The
tendons are located at 100 mm from
the soffit of the beam. If the modular
ratio is 6, the loss of prestressing
force (in kN, up to one decimal
place) due to the elastic deformation
of concrete only is ______________

© Copyright Reserved by Gateflix.in No part of this material should be copied or reproduced without permission
ANSWER KEY:

1 2 3 4 5 6 7 8 9 10 11
(a) (a) (c) (a) (c) (d) (b) (d) 4.8 kN 0.15m 0.120 mm

© Copyright Reserved by Gateflix.in No part of this material should be copied or reproduced without permission
EXPLANATIONS

Q.1 (a)
P Pe
- σ min =
A Z
400 ×103 400 ×103100 1620 ×103 1620 ×103 ×145
== ( −) + −
200 × 400 200 × 4002 500 × 750  500 × 7502 
 
6  6 
= 12.5 MPa 299.75 ×10 6

 299.757502 
Q.2 (a)  
 6 
Q.3 (c)
= 2.9N/mm2 (comp)
Q.4 (a)
P 150×103 ×20 Q.8 (d)
σ bot ,σ top = m Loss = 10%
A 120×2002 
  Find force = 750 kN
 6  750
Initial force =  = = 833.33 kN
σ bot = +10 MPa ( C ) 0.9
σ top = +2.5 MPa ( C ) Top & Bottom stress = ±
P m
A z
Q.5 (c) 833.33 833.33×103 ×100×6
= ×103 ±
Stress in concrete at the level of 250 × 400 250×4002
steel is
P Pe = 8.33 ± 12.5
fc = + (e)
A l Top = -4.166 (T)
150 ×103 150103 × (20) 2 Bottom = 20.833 (C)
+
120 × 200 120 × 2003  Q.9 4.8 kN
 
 12 
= 7 MPa Initial prestressing force
=3⨉50⨉1200 = 180000 N
Q.6 (d)
Eccentricity (e) = ( 2 - 100)
Sec IS : 456 = clause 2.19 D

Q.7 (b) 300


P Pa M Dl+LL =( - 100) = 50 mm
σ b= + - 2
A Z Z
Stress in concrete at the location of
𝑃𝑃 Pe
steel = � � + � � e
𝐴𝐴 I

© Copyright Reserved by Gateflix.in No part of this material should be copied or reproduced without permission
180000 180000 × 502 Elongation of the cable =
= + 150×3003 2⨉50⨉1.2⨉10−3
150×300
12
= 0.120 mm
= 4.0 + 1.33 = 5.333 N/mm2

Loss of stress = m ⨉ fc
= 6 × 5.3333 = 32 N/mm2

Loss of prestressing force


3⨉50⨉32
= kN = 4.8 kN
1000

Q.10 (0.15)

wl2
Central dip , e=
8P

30×82
= = 0.15 m
8×1600

Q.11 (0.120mm)

Downward UDL,

w = 6kN/m

Eccentricity, e = 50 mm

Prestressing force, P = 200 kN

EI = 2 ⨉ 104 kN m2

L=6m
PeL
Rotation due to prestress = 2EI

200×50×6×10−3
= = 1.5⨉ 10−3
2×2×104

wL3
Rotation due to UDL =
24EI

6×63
= = 2.7 ⨉ 10−3
24×2×104

Net rotation = 2.7 ⨉ 10−3 - 1.5 ⨉ 10−3

= 1.2 ⨉ 10−3 radian

© Copyright Reserved by Gateflix.in No part of this material should be copied or reproduced without permission
STEEL STRUCTURE

© Copyright Reserved by Gateflix.in No part of this material should be copied or reproduced without permission
1 GENERAL DESIGN SPECIFICATION

1.1 DESIGN OBJECTIVE The following load combinations with


appropriate load factors may be considered
The objective of design is the achievement in designing.
of an acceptable probability that structures a) Dead load + Imposed load
will perform satisfactorily for the intended b) Dead load + Imposed load + Wind or
purpose during the design life. With an Earthquake load
appropriate degree of safety, they should c) Dead load + Wind or Earthquake load
sustain all the loads and deformation d) Dead load + Erection load
during construction and use and have
adequate resistance to accidental loads and 1.5 GEOMETRICAL PROPERTIES
fire.
IS 800-2007 gives the concept of the gross
1.2 METHODS OF DESIGN and effective cross -sections of a member.
• The properties of the gross cross-
Structure and its elements shall normally section shall be calculated from the
be designed by limit state method as per IS specified size of the member or read
800-2007. Where the limit state method from appropriate table.
cannot be conveniently adopted; working • The effective cross -section of a member
stress method shall be used. is that portion of the gross cross –
section that is effective in resisting the
1.3 LOADS AND FORCES stresses.
Highlighting this concept of effective
For the purpose of designing any element, cross - section, IS 800-2007 has
member or structure, the following loads classified the members cross-section as
and their effects shall be taken into follows.
account, where applicable, with partial
safety factors and combinations : 1.6 CLASSIFICATION OF CROSS SECTIONS
a) Dead loads;
b) Imposed loads; (Live load, crane load, Basis of classification
snow load etc.) • The plate elements of a cross -section
c) Wind loads may buckle locally due to compressive
d) Earthquake loads stresses.
e) Erection loads • When plastic analysis is used, the
f) Accidental loads such as those due to blast members shall be capable of forming
g) Secondary effects due to contraction or plastic hinges with sufficient rotation
expansion resulting from temperature capacity (ductility) without local
changes, differential settlements of the buckling to enable the redistribution of
structure as a whole or of its bending moment required before
components, eccentric connections. formation of failure mechanism.
• When elastic analysis is used, the
1.4 LOAD COMBINATIONS member shall be capable of developing
the yield stress under compression
without local buckling.

© Copyright Reserved by Gateflix.in No part of this material should be copied or reproduced without permission
On the above basis, four classes of section from explosions or impact or due to
are defined as follows : consequences of human error to an extent
not originally expected to occur.
a) Plastic : Cross-sections, which can The acceptable limit for the safety and
develop plastic hinges and have the serviceability requirements before failure
rotation capacity required for failure of occurs is called a limit state. The objective
the structure by formation of a plastic of design is to achieve a structure that will
mechanism. not become unfit for use with acceptable
b) Compact : Cross -sections, which can target reliability. In other words, the
develop plastic moment of resistance, probability of a limit state being reached
but have inadequate plastic hinge during its lifetime should be very low. In
rotation capacity for formation of a general, the structure shall be designed on
plastic mechanism. the basis of the most critical limit state and
c) Semi-compact : Cross -Sections, in shall be checked for other limit states.
which the extreme fibre in compression
can reach yield stress, but cannot, In limit state design, structures are
develop the plastic moment of designed on the basis of safety against
resistance, due to local buckling. failure and are checked for
d) Slender : Cross -sections in which the serviceability requirements.
elements buckle locally even before
reaching yield stress. 1.8 CONNECTIONS:RIVETED CONNECTION

• Riveting is a method of joining together


pieces of metal by inserting ductile
metal pins called rivets into holes of
pieces to be connected and forming a
head at the end of the rivet to prevent
ach metal piece from coming out.
• Rivet holes are made in the structural
members to be connected by punching
or by drilling. The size of rivet hole is
kept slightly more (1.5 to 2.0 mm) than
the size of rivet.
• After the rivet holes in the members are
matched, a red hot rivet is inserted
which has a shop made head on one
side and the length of which is slightly
more than the combined thicknesses of
1.7LIMIT STATE DESIGNS OF STEEL the members to be connected.
STRUCTURES • Then holding red hot rivet at shop head
end, hammering is made.
Basis for Limit State Design • It results in to expansion of the rivet to
In the limit state design method, the completely fill up the rivet hole and also
structure shall be designed to withstand into formation of driven head.
safely all loads likely to act on it throughout • Desired shapes can be given to the
its life. It shall also satisfy the serviceability driven head.
requirements, such as limitations of • The riveting is done may be in the
deflection and vibrations and shall not workshops or in the field.
collapse under accidental loads such as

© Copyright Reserved by Gateflix.in No part of this material should be copied or reproduced without permission
• Riveting has the following The shank is left unfinished i.e. rough as
disadvantages. rolled.
a) It is associated with high level of • In structural elements to be connected
noise pollution. holes are made larger than nominal
b) It needs heating the rivet to red hot. diameter of bolts.
c) Inspection of connection is a skilled • As shank of black bolts is unfinished,
work. the bolts may not establish contact with
d) Removing poorly installed rivets is structural member at entire zone of
costly. contact surface.
e) Labour cost in high. • Joints remain quite loose resulting into
• Production of wieldable quality steel large deflections.
and introduction of high strength • These bolts are used for light structures
frication grip bolts have replaced use of under static loads such as trusses,
rivets. bracings and also for temporary
• Design procedure for riveted connections required during erections.
connections is same as that for bolted • It is not recommended for connection
connection except that the effective subjected to impact, fatigue or dynamic
diameter of rivets may be taken as rivet loading.
hole diameter instead of nominal • Bolt of property class 4.6 means,
diameter of rivet. ultimate strength is 400 N/mm2 and
• IS 800-2007 do not discuss riveted yield strength is 400 × 0.6 = 240 N/mm2
connection in it hence it is not discussed • If a bolt is designated as M16, M20,
further here. M24, M30, it means shank diameter of
16 mm, 20 mm, 24 mm, and 30 mm
1.8.1 BOLTED CONNECTIONS
respectively.
• A bolt is a metal pin with a head formed b) Finished/Turned Bolts
at one end and shank threaded at the
other in order to receive a nut. Bolts are • These bolts are also made from mild
used for joining together pieces of steel, but they are formed from
metals by inserting them through holes hexagonal rods, which are finished by
in the metal and tightening the nut at turning to circular shape.
the thread ends. • Tolerance available for fitting is quite
small (0.15 mm to 0.5 mm).
• It needs special methods to align bolt
holes before bolting.
• As connection is tighter, it results in to
much better bearing contact between
the bolts and holes. These bolts are
used in special jobs like connecting
machine parts subjected to dynamic
Bolts are classified as : loadings.
a) Unfinished (black) bolts
b) Finished (turned) bolts c) High strength Friction Grip (HSFG)
c) High strength friction grip (HSFG) bolts Bolts High Strength bolts
a) Made from bars of medium carbon
a) Unfinished/Black Bolts
steel.
• These bolts are made from MILD steel b) Normally class 8.8 and 10.9 are
rods with square or hexagonal head. commonly used

© Copyright Reserved by Gateflix.in No part of this material should be copied or reproduced without permission
c) Less ductile than black bolts Diameter of bolt hole is larger than the
d) Material of the bolt does not have well- nominal diameter (shank diameter) of
defined yield point. Instead of yield the bolt to facilitate erection and to
stress, proof load is used allow for in assurances in fabrication.
e) As per IS 8000: 2007 proof load is taken Holes are
as 0.7 × ultimate tensile stress of bolt a) Standard clearance hole normal
f) M16, M20, M24, M30, are generally b) Oversized holes (i.e. holes of size
used larger than standard clearance hole)
g) Designated like 8.8S, 10.9S denotes high used in slip resistant connection.
strength bolt. c) Short and long slot used in slip
h) Percentage elongation of these bolts at
failure is approx 12% resistant connection following table
i) Special techniques are used for gives the diameter of holes for bolts.
tightening the nuts to induce specified Clearances for fastener Holes
initial tension in the bolts, which caused Sl Nominal size Size of the Hole = Nominal Diameter of the
sufficient friction between the flaying No. of Fastener, d Fastener + Clearances
mm mm
forces. Standard Over size Clearance in
j) These bolts with induced initial tension Clearance in Clearance in the Length of
Diameter and Diameter the slot
called as high strength friction grip Width of Slot Short Long
(HSFG) bolts. Slot Slot
(1) (2) (3) (4) (5) (6)
k) Due to friction, the sleep in the joint is i) 12 - 14 1.0 3.0 4.0 2.5 d
eliminated hence; connection in this ii) 16 - 22 2.0 4.0 6.0 2.5 d
case is called nonslip connection or iii) 24 2.0 6.0 8.0 2.5 d
iv) Larger than 24 3.0 8.0 10.0 2.5 d
friction type connections. From the above table:
1.9 TERMINOLOGY IN BOLTED Diameter of Normal Bolt Holes is:
CONNECTION Nominal 12 14 16 20 22 24 30 36
size of
1. Pitch of the bolts (P) Bolts in
mm
It is the centre to centre spacing of bolts
Diameter 13 15 18 22 24 26 33 39
in a row, measured in the direction of of Bolt
load. hole in
2. Gauge (g) mm
It is the distance between the two 5. Area of Bolt at Root (Anb)
consecutive bolts of adjacent rows and
is measured at right angle to the Area of bolt at root of the thread is less
direction of load. than that at shank of the Bolt. It is taken
3. Staggered Pitch (Ps) approximately equal to 0.78 times the
It is the centre to centre distance of shank area i.e. Anb = 0.78 × Asb
staggered bolts measured in the Where Asb = Area of bolt at shank
direction of load. d=Nominal diameter of Bolt(shank diameter)
Anb = Area of bolt at root

1.10 IS 800-2007 SPECIFICATION FOR


SPACING AND EDGE DISTANCES OF
BOLT HOLES
1. Pitch 'P' shall not be less than 2.5 d,
4. Diameter of Bolt Hole
where d is the nominal diameter of bolt.
2. Pitch 'P' shall not be more than

© Copyright Reserved by Gateflix.in No part of this material should be copied or reproduced without permission
a) 16 t or 200 mm, whichever is less, in distance 40 mm + 4 t, where t is
case of tension members. the thickness of thinner connected
b) 12 t or 200 mm, whichever is less, in plate.
case of compression members where t 7. Apart from the required bolt from the
is the thickness of thinner plate. consideration of design forces,
c) In case of staggered pitch, pitch may be additional bolts called tacking fasteners
increased by 50 percent of values should be provided as specified below.
specified above provided gauge a) Tacking rivets should be providing.
distance is less than 75 mm. i) At 32 t or 300 mm, whichever is
3. In case of compression member where less, if plates are not exposed to
forces are transferred through butting weather.
faces, i.e., (butt joints), maximum pitch ii) At 16 t or 200 mm, whichever is
is to be restricted to 4.5 d for a distance less, if plates are exposed to
of 1.5 times the width of plate from the weather?
butting surface. (Refer Figure Below).

8. In case of a tension member made up of


two flats or angles or tees or channels,
tacking rivets are to be provided along
the length to connect its components as
specified below:
a) Not exceeding 1000 mm, if it is
tension member.
4. The gauge length 'g' should not be more b) Not exceeding 600 mm, if it is
than 100 mm + 4 t or 200 mm compression member.
whichever is less in compression and Countersunk heads
tension member where t is the
thickness of thinner outside plate.
5. Minimum edge and end distance shall
not be
a) Less than 1.7 × hole diameter in case • is neglected in calculating length of
of sheared or hand flame out edges. fastener in bearing
b) Less than 1.5 × hole diameter in case • For Fastener in tension having
of rolled, machine flame cut, sawn countersunk heads, tensile strength is
and planed edges. reduced by 33.3% and no reduction in
6. Maximum edge distance (e) should not shear strength calculation.
exceed
250 1.11 TYPES OF JOINTS
a) 12 tε, where ε = and t is the
fy Types of joints may be grouped into the
thickness of thinner outer plate. following two:
This recommendation does not a) Lap joint
apply to fasteners interconnecting b) Butt joint
the components of back to back a) Lap Joint:
tension members. It is the simplest type of joint. In this the
b) Where the members are exposed to plates to be connected overlap one another.
corrosive environment max edge

© Copyright Reserved by Gateflix.in No part of this material should be copied or reproduced without permission
times the hole diameter in case of rolled,
machine-flame cut, sawn and planed edges.
The maximum edge distance to the nearest
line of fasteners from and edge of any un-
stiffened part should not exceed 12 tε,
250
where ε = � f and t is the thickness of the
y

b) Butt Joint thinner outer plate.


In this type of connections, the two This would not apply to fasteners
main plates butt against each other and interconnecting the components of back to
the connection is made by providing a back tension members. Where the
single cover plate connected to main members are exposed to corrosive
plate or by double cover plates, one on influences, the maximum edge distance
either side connected to the main shall not exceed 40 mm plus 4t, where t is
plates. the thickness of thinner connected plate.

1.12DESIGN STRENGTH OF PLATES IN A


JOINT
Plates in a joint made with bearing bolts
may fail due to any one of the following
1. Shearing or bursting of the edge.
2. Crushing of plates.
3. Rupture of Plates.
4. Block shears failure of plates in tension.
It the maximum distances are ensured in a
joint, the design tensile strength of plate in
the joint is the strength of the thinnest
member against rupture. This strength is
0.9A n f u
given by. Tdn =
γ m1
The brushing or shearing and crushing Where
failures are avoided if the minimum γmt = Partial safety factor for failure at
edge/end distance as per IS 800-2007
ultimate stress = 1.25
recommendations are provided.
fu = Ultimate stress of the material
An = Net effective are of the plate at critical
As per IS 800-2007:
section, which is given by
The minimum edge distance and end 𝑃𝑃 2 𝑠𝑠𝑠𝑠
distances from the centre of any hole to the 𝐴𝐴𝑛𝑛 = �𝑏𝑏 − 𝑛𝑛𝑑𝑑ℎ + ∑𝑖𝑖=𝑡𝑡 � 𝑡𝑡
4𝑔𝑔𝑖𝑖
nearest edge of a plate shall not be less than.
1.7 times the hole diameter in case of b = Width of plate
sheared or hand – flame cut edges; and 1.5 t = Thickness of thinner plate in joint

© Copyright Reserved by Gateflix.in No part of this material should be copied or reproduced without permission
dh = Diameter of the bolt hole (2 mm in or next last row of bolt in plates,
addition to the diameter of the hole, in case perpendicular to the line of force
of directly punched holes) respectively (2-3).
g = Gauge length between the bolt holes fu, fy = Ultimate and yield stress of the
Ps = Staggered pitch length between lines of material respectively.
bolt holes γ m 0 , γ m 1 = Partial factor of safety in
n = Number of bolt holes in the critical
yielding and rupture respectively
section
(γ m 0 1.1,
= = γ m 1 1.25).
It may be noted that, if there is no
staggering Psi = 0 and hence An = (b – ndh)t, 1.14 DESIGN STRENGTH OF BEARING
which is the net area at critical section. BLOTS
1.13 BLOCK SHEAR STRENGTH
The design strength of bearing bolts under
Block Shear failure of plate occurs as shear is the least of the following :
shown below. It's a combination of yielding a) Shear capacity (strength).
and rupture. Block shear failure of a plate b) Bearing capacity (strength)
occurs along a path involving tension on
one plane and shear on a perpendicular a) Shear Capacity (Strength) of Bearing
plane along the fasteners. Bolts in a joint
Design strength of the bolt, Vdsb is given by
Vnsb
Vdsb =
γ mb
Where
Vnsb, nominal shear capacity of bolt and
γ mb = partial safety factor of material of
bolt= 1.25. In the above expression Vnsb is
f ub
given by Vdsb = ( ηn A nb +ηs Asb )
3
Block Shear strength at an end Where, fub=Ultimate tensile strength of a bolt
connection is calculated as given below. ηn = Number of shear planes with threads
It should be taken as the smaller of
intercepting the shear plane.
 0.9f u A tn 
(
Tdb = A vg f y 3 γ m0 ) +
 γm


ηs = Number of shear planes without
threads intercepting the shear plane
 1 
Asb = Nominal shank area of bolt = π d
2

Shear yielding + tensile rupture


4
OR
Anb = Net shear area of the bolt at threads,
 0.9f u A tn   f y A bg  may be taken as the area corresponding to
Tdb =   +
 γm   γ m0  root diameter of the thread.
 1  π π
Anb = 0.78 d2 or (d − 0.9382p)2
Shear rupture + tensile yielding 4 4
Where
Avg, Avn = Minimum gross and net area in Where p is the pitch of thread, d = nominal
shear along a line of transmitted force dia. of bolt i.e. shank dia.
respectively (1-2 and 4-3 as shown in 1.15 BEARING CAPACITY OF BOLTS(Vdpb)
figure above.)
Atg, Atn = Minimum gross and net area in IS 800-2007 Suggest the following
tension from the hole to the toe of the angle procedure to find bearing strength of bolts.

© Copyright Reserved by Gateflix.in No part of this material should be copied or reproduced without permission
Vnpb where,
Vdpb = fub = Ultimate tensile stress of the bolt
γ mb
fyb = Yield stress of the bolt
Where, An = Net tensile stress area, shall be taken
Vdpb = Design bearing strength as the area at the bottom of the threads
Vnpb = Nominal bearing strength Asb = Shank area of the bolt.
γ mb = Partial safety factor of material = 1.25 γ mb , γ m0 = Partial safety factors = 1.25, 1.1
Nominal bearing strength may be found respectively
from the following relation. It means that tensile capacity of bolts is
Vnpb = 2.5 K b dt f u
minimum of
where Kb is smaller of  0.9f tb A n f yb A sb 
e P f Tdb = Min  , 
, – 0.25, ub , 1.0  γ mb γ m0 
3d h 3d h fu
0.9f tb A n
In which e, p are end and pitch distances = tensile rupture
respectively γ mb
dh = diameter of hole f yb A sb
= tensile yielding
fub, fu = ultimate tensile strength of bolt and γ m0
plate respectively
d = nominal diameter of bolt 1.17 WELDED CONNECTIONS
t = summation of the thickness of the
Welding consists of joining two pieces of
connected plates experiencing bearing
metal by establishing a metallurgical bond
stress in the same direction. If bolts are
between them. The elements to be
countersunk, it is to be reduced by the half
connected are brought closer and the metal
depth of countersinking.
is melted by means of electric arc or
In plates the bearing strength is a linear
oxyacetylene flame along with weld rod
function of end distances both bolts and
which adds metal to the joint. After cooling
plates are subjected to significant triaxial
the bond is established between the two
containment. Due to this, bearing behavior
elements.
of plates is influenced by the proximity of
neighbouring holes or boundary (edge 1.18 TYPES OF WELDED JOINTS
distance) There are three types of welded joints:
1. Butt weld
2. Fillet weld
3. Slot weld and Plug weld
1. Butt Weld
Butt weld is also known as groove weld.
1.16 TENSILE CAPACITY OF BOLTS Depending upon the shape of groove made
for welding butt welds.
A bolt subjected to a factored tensile force Types of Butt Welds
Tb shall satisfy:
Tb ≤ Tdb
where,
Tdb = Tnb γ mb
Tnb = Nominal tensile capacity of the bolt
γ 
Tnb = 0.9 f ub A n < f yb A sb  mb 
 γ m0 

© Copyright Reserved by Gateflix.in No part of this material should be copied or reproduced without permission
Figure below shows typical plug welds in
which small holes are made in one plate
and is kept over another plate to be
connected and then the entire hole is filled
with filler material.

2. Fillet Weld
Fillet weld is a weld of approximately
triangular cross-section joining two
surfaces approximately at right angles to 1.19 SPECIFICATION FOR WELDING
each other in lap joint, tee joint or corner Important specification regarding butt
joint. weld, fillet weld and plug and slot weld as
per IS 800 – 2007 is:
Butt Weld
1. The size of butt weld shall be specified
by the effective throat thickness. In case
of a complete penetration butt weld it
shall be taken as thickness of the
When the cross-section of fillet weld is thinner part joined. Double U, double V,
isosceles triangle with face at 45º. It is double J and double level butt welds
known as standard fillet weld. In special may be generally regarded as complete
circumstances 60º and 30º angel are also penetration butt welds.
used. A fillet weld is known as concave fillet The effective throat thickness in case of
weld, convex fillet weld or as mitre fillet incomplete penetration butt weld shall
weld depending upon the shape of weld face. be taken as the minimum thickness of
the weld metal common to the parts
joined excluding reinforcement. In the
absence of actual data it may be taken
as 5/8th of thickness of thinner material.
2. The effective length of butt weld shall
be taken as the length of full size weld.
3. Slot Weld and Plug Weld 3. The minimum length of butt weld shall
Figure below shows a typical slot weld in be four times the size of the weld.
which a plate with circular hole is kept with 4. If intermittent butt welding is used, it
another plate to be joined and then fillet shall have an effective length of not less
welding is made along the periphery of the than four times the weld size and space
hole. between the two welds shall not be
more than 16 times the thickness of the
thinner part joined.

© Copyright Reserved by Gateflix.in No part of this material should be copied or reproduced without permission
5. Lap joint:
1.20 FILLET WELD The minimum lap should be four times the
1. Size of Fillet Weld thickness of thinner part joined or 40 mm
a) The size of normal fillet weld shall be whichever is more. The length of weld
taken as the minimum weld leg size. along either edge should not be less than
b) For deep penetration welds with the transverse spacing of welds.
penetration not less than 2.4 mm, size
of weld is minimum leg size + 2.4 mm. 6. Intermittent welds:
c) For fillet welds made by semi-automatic Length shall not be less than 4 times the
or automatic processes with deep weld size or 40 mm whichever is more. The
penetration more than 2.4 mm, if minimum clear spacing of intermittent
purchaser and contractor agree. weld shall be 12 t for compression joints
S = minimum leg size + Actual and 16 t for tensile joints, where t is the
penetration thickness of thinner plate joined. The
intermittent welds shall not be used in
2. Minimum size of fillet weld specified positions subject to dynamic repetitive and
in 3 mm. alternating stresses.
To avoid the risk of cracking in the absence
of preheating the minimum size specified 1.20.2 DESIGN STRESSES IN WELDS
are: Butt Welds
For less than 10 mm thickness plate – 3 mm Butt welds shall be treated as parent metal
For 10 to 20 mm thickness plate – 5 mm with a thickness equal to the throat
For 20 to 32 mm thickness plate – 6 mm thickness, and the stresses shall not exceed
For 32 to 50 mm thickness plate – 8 mm those permitted in the parent metal. It
means that the strength of Butt weld is
3. Effective throat thickness: equal to the strength of the parent metal.
It shall not be less than 3 mm and shall not
generally exceed 0.7 t (or t under special Fillet Weld, Slot or Plug Welds
circumstances) where t is the thickness of Design strength shall be based on its throat
the thinner plate of the elements being area and shall be given by
welded. If the face of plates being welded is f wn
f wd =
inclined to each other, the effective throat γ mw
thickness shall be taken as K times the fillet
size where K is as given in table below:
fu
Where, f wn =
3
fu = Smaller of the ultimate stress of the
weld or of the parent metal
γ mw = 1.25 for shop welds= 1.5 for field
welds
4. Effective Length:
IS 800-2007 gives the following provisions
The effective length of the weld is the
for the fillet welds:
length of the weld for which specified size
and throat thickness exist. In drawings only 1. If a fillet welds is applied to the square
effective length is shown. While welding edge of a part, the specified size of the
length made is equal to effective length weld should generally be at least 1.5
plus twice the size of the weld. Effective mm less than the edge thickness.
length should not be less than 4 times
the size of the weld.

© Copyright Reserved by Gateflix.in No part of this material should be copied or reproduced without permission
1.21 REDUCTION IN DESIGN STRESSES
FOR LONG JOINTS
If the length of the welded joint lj is greater
than 150 t , where t is throat thickness, the
2. If fillet weld is applied to the rounded design capacity of weld fwd shall be
toe of a rolled section, the specified size reduced by the factor.
of the weld should generally not exceed 0.2l j
3 βlw =1.2 − ≤ 1.0
4
th of the thickness of the section at the 150 t t
toe. lj = length of the joint in the direction of the
force transfer and
t = throat thickness of the weld
The reduction in design strength of long
joint is to cater for non uniform
mobilization of stress in a long joint
causing ineffectiveness of certain length
3. In members subjected to dynamic of joint.
loading, the fillet weld shall be of full
size with its leg length equal to 1.22ECCENTRIC CONNECTION – PLANE
thickness of plate. OF MOMENT AND THE PLANE OF WELDS
IS THE SAME

Figure above shows a typical case. The


eccentric load P is equivalent to
4. End fillet weld, normal to the direction i) a direct load P at the centre of gravity of
of force shall be of unequal size with the group of weld and
throat thickness not less than 0.5 t. The ii) a twisting moment P × e
difference in the thickness of weld shall Let a weld of uniform size be applied
be negotiated at a uniform slope. throughout and 't' be the effective
throat thickness. It’d’ is the depth of
weld and 'b' is the width as shown in
the figure, the direct shear stress in the
weld is
P
q1 =
( 2b+d ) t
The stress in the weld due to twisting
moment is the maximum in the weld at
the extreme distance from the centre of
gravity of the group of weld and acts in

© Copyright Reserved by Gateflix.in No part of this material should be copied or reproduced without permission
the direction perpendicular to the
radius vector. The maximum stress due
to the moment.
P×e×rmax
q2 =
I zz
where rmax is the distance of the
extreme weld from the c.g. of the group.
Izz = Ixx + Iyy, the polar moment of
inertia.
The vector sum of the stress is
q = q12 + q 22 + 2q1 q 2 cos θ.
For the safe design it should be less
than the resistance per unit area.

© Copyright Reserved by Gateflix.in No part of this material should be copied or reproduced without permission
2 DESIGN OF TENSION MEMBERS

2.1TENSION MEMBER Before going in to this discussion, lets first


understand few terms.
Tension members are linear members in
which axial forces act causing elongation 2.3.1Net Effective Area
(stretch). Such members can sustain loads It is defined as the gross area (Ag) of the
upto ultimate load, at which stage they may
section minus the deductions (Ad) made for
fail by rupture at a critical section.
any loss of material. Thus, An = Ag – Ad.
However, if the gross area of the member
yields over a major portion of its length
2.3.2 Diameter of Bolt Hole
before the rupture load is reached, the
member may become non functional due to According to IS 800-2007 diameter of Bolt
excessive elongation, plates and other hole is to be taken as larger than nominal
rolled sections in tension may also fail by dia. of Bolt. Following table gives the
block shear of end bolted regions, under diameter of holes for Bolts.
combined shear along longitudinal section
along bolt lines and normal tensile stresses
on a transverse section across a bolt line.
The factored design tension T, in the
members shall satisfy the following
requirement
T < Td
Where Td = Design strength of member.
The design strength of member under axial
tension Td is the lowest of the design
strength due to yielding of gross section,
Tdg, rupture of critical section, Tdn and block
shear, Tdb.
2.3.3Strength of Plate in Rupture
2.2DESIGN STRENGTH DUE TO YIELDING
OF GROSS SECTION The design strength in tension of a plate Tdn
as governed by rupture of net cross-
The design strength of members under sectional area An, at the hole is given by
axial tension Tdg as governed by yielding of Tdn = 0.9An fu ∕ γm1
gross-section is given by γm1 = Partial safety factor for failure in
Tdg = f y A g γ m0
tension by rupture = 1.25
Where fu = Ultimate stress of the material in MPa
fy = Yield strength of the material in MPa An = Net effective area of the member
Ag = Gross area of cross -section in mm2
2.3.4 Calculation of Net Effective Area
= Partial safety factor for failure in tension
(An)
by yielding = 1.1
2.3DESIGN STRENGTH DUE TO RUPTURE
OF CRITICAL SECTION

© Copyright Reserved by Gateflix.in No part of this material should be copied or reproduced without permission
2.3.6 Strength of single Angles in
Rupture
The tearing/rupture strength of an angle
connected through one leg is affected by
shear lag.

Shear lag : It is a term used for non-


* Rupture can occur along 1-2-3-4-5-6-7 uniform transfer of stresses near the
P2 connection. the connected leg will have
An = �b − ndh + � � �� t higher stresses than the unconnected
4g
leg.
In this case,n = 5
Shear lag effect reduces with increase in
 2P 2  connection length.
Hence A n in this case =  b – 5d h + t
 4g  The design strength of single angle, Tdn, as
Where, governed by tearing at net section is given by
dh = Diameter of Bolt hole Tdn = 0.9 fuAnc / ϒm1 + βAgo fy / ϒm0
b, t = Width and thickness of plate where
respectively w fy
β = 1.40 − 0.076 � � � � � s� ≤ � u
b f γm0
� ≥ 0.7
n = Number of Bolt holes P=Staggered pitch t fu L fy γm1

length (parallel to pull) b 


=1.4 – 0.54  s 
g=Gauge length between bolt holes L
(perpendicular to pull)
Where w & bs are as shown in figure below.
L = Length of end connection i.e. distance
2.3.5 Strength of threaded Rods in
between the outermost bolt in the joint
Rupture
along the force direction or length of the
The design of threaded rods in tension, Tdn , weld along the force direction.
as governed by rupture is
Given by
Tdn = 0.9 f u A n γ m1 Where
An = net root area at the threaded
π
section= 0.78 × ×d2
4
d = Nominal diameter of Bolt

© Copyright Reserved by Gateflix.in No part of this material should be copied or reproduced without permission
Block shear failure is a combination of
tensile and shear failure under tensile
load

Block shear strength at an end connection


is calculated as given below.
Alternatively the tearing strength of net It should be taken as the smaller of
section may be taken as Tdn = (Avgfy/√3 ϒmo) + (0.9fuAtn/ϒm1)
An fu Shear yielding + tensile rupture
Tdn = α OR
γ m1
Where,
(
Tdb = 0.9 f u A vn )
3 γ m1 + f y A tg /γ m0 Shear
α = 0.6 for one or two bolts, 0.7 for three rupture + tensile yielding where,
bolts and 0.8 for four or more bolts in the Avg, Avn = Minimum gross and net area in
end connection or equivalent weld length. shear along a line of transmitted force,
An= Net area of the total cross-section respectively (1-2 and 4-3 as shown in
Anc=Net area of the connected leg. figure(1) and (1-2) as shown in figure (2).
Ag0=Gross area of the outstanding leg Atg, Atn = Minimum gross and net area in
t=Thickness of the leg tension from the hole to the toe of the
angle or next last row of bolt in plates,
2.4DESIGN STRENGTH DUE TO BLOCK perpendicular to the line of force
SHEAR respectively (2-3) as shown in figure (1)
and figure (2).
In this failure made, the failure of the fu, fy=Ultimate and yield stress of the
member occurs along a path involving material respectively.
tension on one plane and shear on a γ m0 , γ m1 =Partial factor of safety in yielding
perpendicular plane along the fasteners as and rupture respectively
shown in the figure below. (γ m0 1.1,
= = γ m1 1.25)

2.5LUG ANGLES

© Copyright Reserved by Gateflix.in No part of this material should be copied or reproduced without permission
Length of the end connection of a heavily b) The strength of fasteners connecting
loaded tension member may be reduced by Lug angle to the gusset should be at
using lug angles. By using Lug angles there least 10% more than the force not
will be saving in gusset plate, but it is upset accounted for by the direct
by additional fasteners and angle required. connection of the member and the
Hence, now-a-days it is not preferred, IS attachment of the Lug angles to the
800-2007 specification for Lug angles are: member shall be capable of
developing 20% in excess of that
force.

1. The effective connection of the Lug


angles shall be as far as possible
terminate at the end of the member
connected.
2. The connection of Lug angle to main
member shall preferably start in
advance of the member to the gusset
plate.
3. Minimum of two bolts, rivets or
equivalent welds be used for attaching
Lug angles to the gusset.
4. If the main member is angle:
a) The whole area of the member shall
be taken as the effective rather than
net effective section (i.e. with
reduction for outstanding Leg area.)
The whole area of the member is the
gross area less deduction for bolt
holes.
b) The strength of Lug angles and
fastener connecting Lug angle to
gusset plate should be at least 20%
more than the force in outstanding
Leg.
c) The strength of the fastener
connecting Lug angle and main
member shall be at least 40% more
than the force carried by the
outstanding leg.
5. In case the main member is a channel
and like:
a) As far as possible Lug angles should
be placed symmetrically.

© Copyright Reserved by Gateflix.in No part of this material should be copied or reproduced without permission
3 DESIGN OF COMPRESSION MEMBERS

3.1 INTRODUCTION restrained ends. In figure below 6 m


While designing as structure there are column is restrained at ends A and B in
member that are in compression. Vertical both y-y and z-z direction. At C it is
compression members in buildings are restrained in z – z direction only. Hence its
called column, posts or sanctions. actual length in -y direction is 6 m while in
Compression members in trusses are called z-z direction it is equal to AC = 3m only.
struts. The jib of crane which carries
compression is called boom. Whatever care
is taken by the engineers to transfer load
axially unexpected eccentricity of load is
unavoidable due to imperfection. This
eccentricity causes lateral bending moment
which results in to bending compression
also. As the axial compression increase the
lateral deflection increases resulting into
additional bending stresses. A stage of 3.2.2 EFFECTIVE LENGTH
instability is reached at load much below
The effective length KL is calculated from
crushing strength of compression members.
the actual length L, of the member
This phenomenon is called buckling of
considering the rotational and relative
columns. Because of buckling tendency the
translation boundary conditions at the end.
load carrying capacity of columns is
IS800-2007 recommends the following.
reduced considerably. The load carrying
capacity depends upon the end conditions a) If End conditions can be assessed
and also on slenderness ratio of the column Where the boundary conditions in plane
sections of buckling can be assessed the effective
length KL can be calculated on the basis
3.2 SLENDERNESS RATIO
of table given below.
Slenderness ratio of a column is defined as
b) Compression Member in Trusses
the ratio of effective length to
i) In the case of bolted riveted or
corresponding radius of gyration of section.
welded trusses and braced frames.
Thus
The effective length KL shall be
le KL
Slenderness ratio = = taken as 0.7 to 1.0 times the actual
r r length, depending upon the degree
Where L = Actual length of compression of end restrained provided.
Member ii) For buckling in the plane
le = KL, effective length perpendicular to the plane of truss,
r = Appropriate radius of gyration the effective length may be taken
as actual length.
3.2.1 ACTUAL LENGTH
c) In Frames
It is centre to centre distance of In frame analysis, if deformed shape is
compression member between the not considered (second order analysis

© Copyright Reserved by Gateflix.in No part of this material should be copied or reproduced without permission
is not used) the effective length depends fy 1 fy
upon stiffness's of the members f cd = × ≤
connecting at the joint. The method of γ m0 φ + (φ 2 – λ )
2 0.5 γ m0
finding effective length factor K are χfy fy
shown in annex D of IS 800-2007. One f cd = ≤
can use the graphs given in annexure. γ m0 γ m0
d) In Case of Stepped Columns Where,
Expressions for finding effective length φ = 0.5[1 + α (λ − 0.2) + λ 2 ]
factor for various stepped columns are
λ =Non-dimensional effective slenderness
presented in IS 800-2007 annexure D2
ratio
and D3. 2
fy  KL 
= = fy   ∏2 E
Effective length of Prismatic Compression f cc  r 
Members Fcc = Euler buckling stress
∏2 E
= 2
 KL 
 
 r 
α = Imperfection factor given in table
below
χ = Stress reduction factor for different
buckling class, slenderness ratio n and
yields stress
1
=
φ + (φ 2 − λ 2 )0.5 
γ m0 = Partial safety factor for material
strength in yielding = 1.1 for steel
The design compressive strength Pd of a
members is given by Pd = Ae fcd
Where Ae is the effective cross -sectional
area, which is the same as gross area if bolt
holes are filled with bolts. Deductions for
bolt holes may be made only if the holes
are not fitted with bolts.
Imperfection Factor,
Buckling a B c d
Class
α 0.21 0.34 0.49 0.76

3.4 HOW TO SELECT THE SHAPES OF


3.3 DESIGN COMPRESSIVE STRESS AND COMPRESSION MEMBERS
STRENGTH
Since the design stress in compression
The design compressive stress fcd of axially member decreases with the least radius of
loaded compression members shall be gyration, the section should be
calculated using the following equation. proportioned to have maximum moment of
inertial for the same sectional area. As far

© Copyright Reserved by Gateflix.in No part of this material should be copied or reproduced without permission
as possible the section should have design stress may be assumed as 200
approximately the same radius of gyration N/mm2.
about any axis. This requirement is fulfilled 2. Effective sectional area required,
by circular tubes. Next best shape may be P
Ae = d
square tubing. Moreover it is preferable to f cd
use equal angles instead of unequal angles Where Pd = Factored load in N
as compression members since such angles fcd = Design stress in N/mm2
have higher rmin values for the same cross – 3. Select a section to give effective are
sectional areas. Various shapes of commonly
required and calculate rmin.
used compression members are shown
4. Determine the effective length and
below.
slenderness ration.
5. Calculate design stress and use it to
calculate capacity of the section Pc.
6. Revise the section of Pc< Pd (factored
load) or if Pc>> Pd.

3.6 BATTENS
Instead of lacing one can use battens to
keep members of columns at required
distances. Figure below shows the use of
batten plates.

3.5 STEPS FOR DESIGN OF


COMPRESSION MEMBERS
The following are the usual steps in the
design of compression members:
1. Design stress in compression member
is too assumed.
For rolled beam section the slenderness 3.7 DESIGN OF BATTENED COLUMNS
ratio varies from 70 to 90. Hence design
stress may be assumed as 135 N/mm2. IS 800-2007 specifies the following rules
For angle struts, the slenderness ratio for the design of battened columns:
varies from 110 to 130. Hence, design 1. Batten plates should be provided
stress for such members may be symmetrically.
assumed as 90 N/mm2. For 2. The battens shall be placed opposite to
compression members carrying large each other at each end of the member
loads, the slenderness ration is and at points where the member is
comparatively small. For such members stayed in its length and as far as

© Copyright Reserved by Gateflix.in No part of this material should be copied or reproduced without permission
practicable, be spaced and shear is sum of this shear and 2.5% of
proportioned uniformly throughout. axial load.
The number of battens shall be such 9. The design shear and moments for
that the member is divided into not less batten plates is given by Vb = Vt C and
than three bays within its actual length NS
from centre to centre of end VC
M = t at each connection.
connections. 2N
3. Battens shall be of plates, angles, Where
channels or I-sections and at their ends Vt = Transverse shear force as defined
shall be riveted, bolted or welded. in 7 and 8
4. By providing battens distance between C = Distance between centre to centre
the members of columns is so of battens longitudinally
maintained that radius of gyration N = Number of parallel planes
about the axis perpendicular to the S=Minimum transverse distance
plane of battens is not less than the between the centroid of the fasteners
radius of gyration about the axis connecting batten to the main
parallel to the plane of battens members.
(ryy > rzz ) 10. The effective depth of end battens
(longitudinally) shall not be less than
the perpendicular distance between the
centroids of the main members. The
effective depth of the intermediate
battens shall not be less than 3 of above
4
distance.
11. In no case the effective depth of battens
shall be less than twice the width of one
member in the plane of the batten. It is
to be noted that the effective depth of a
5. The effective slenderness ration of
batten shall be taken as the longitudinal
battened columns shall be taken as 1.1 distance between the outermost
times the maximum actual slenderness
fasteners.
ration of the column, to account for
12. The thickness of battens shall be not
shear deformation.
less than 1 of the distance between the
6. The vertical spacing of battens, 50
measured as centre to centre distance innermost connecting lines of rivets,
between end connection of consecutive bolts or welds.
battens shall be such that the 13. The length of the weld connecting
slenderness ration of any component of batten plate to the member shall not be
column over that distance shall be less than half the depth of batten plate.
neither greater than 50 nor greater At least one third of the weld shall be
than 0.7 times the slenderness ration of placed at each end of this edge.
the member as a whole about its z-z axis.
7. Battens shall be designed to carry the 3.8 DESIGN OF SLAB BASE
bending moments and shear forces
arising from the transverse shear force 3.8.1 When the Column is subjected to
Vt equal to 2.5% of the total axial force. Pure Axial Load
8. In case columns are subjected to In this case pressure will be uniformly
moments also, the resulting shear distributed under the slab base.
should be found and then the design

© Copyright Reserved by Gateflix.in No part of this material should be copied or reproduced without permission
1× t 2
3.8.2 Size of base slab z x-x =
6
wa 2 ×6 3wa 2
Bending stress = f1 = = 2
2 × t2 t
Similarly, for bending about (2)-(2)
3wb 2
fz = 2
t
f f 
1. Find the bearing strength of concrete Maximum in direction of,
= f1 E  1 − µ 2 
E E
which given by 0.45 fck.
2. Find the required area of base plate Now this stress should be less than equal to
where Pu is factored axial load on fy
column. γ m0
3. Select the size of base plate. For
fy 3w 2
economy as far as possible keep the Hence ≥ a – μb 2 
2 
projections a and b equal. γ m0 t

3.8.3 Thickness of base plate 3w (a 2 – 0.3b 2 )


t≥ γ m0
fy
1. Find the intensity of pressure
P Where, µ = 0.3
W=
Actual area of base plate
2. Find the minimum thickness required Now IS code has modified the above
using the following formula given by IS derived formula to account for elastic-
800 – 2007 plastic behavior of the plate, hence it has
taken minimum thickness
2.5w(a2 −0.3b2 )γm
ts = � o
> tf 2.5w (a 2 – 0.3b 2 ) γ m0
fy t=
fy
where
γmo = 1.1 1× t 2
IS code has taken z = in between
fy = yield strength of base plate 5
tf = thickness of flange of column 1× t 2 1×t2
value of z elastic = and zplastic = 4
The above mentioned formula can be 6
derived by taking Poisson’s ratio µ = 0.3
and using elastic bending theory.

Moment at section (1) - (1) for 1 mm width


a wa 2
=1× a × w × =
2 2

© Copyright Reserved by Gateflix.in No part of this material should be copied or reproduced without permission
4 DESIGN OF BEAMS

4.1 INTRODUCTION The design bending strength Md shall be


taken as
Beam is structural member with length M d = β b z p f y γ m0
considerably larger than corss sectional
dimension subject to lateral loads which To avoid irreversible deformation under
give rise to bending moment, shear forces (serviceability loads)
in the member. 1.2 z e f y
There are mainly two types of beams based Md < For simply supported beams
γ m0
on the lateral supports to compression
flanges 1.5 z e f y
Md < For cantilever beams Where,
a) Laterally supported beams γ m0
b) Laterally unsupported beams
If the compression flanges are laterally β b = 1.0 for plastic and compact sections
supported by flooring it is mainly subjected z
to bending and shear. If the compression β b = e for semi-compact sections
zp
flange of beam is not laterally supported
the lateral buckling of the compression zp , ze = plastic and elastic section module
flange reduces the load carrying capacity of of the cross-section respectively
the beam. fy = yield stress of the material and
4.2BENDING STRENGTH OF LATERALLY γ m0 = partial safety factor = 1.1
SUPPORTED BEAM IS 800 – 2007 does not give the design of
a) According to IS 800-2007 the design members belonging to class 4 (slender
bending strength of a section which is sections).
not susceptible to web buckling
under shear before yielding (where
d / tw ≤ 67 e) shall be determined
as under:
Here,
d = depth of web;
tw = thickness of web
ε = 250 f y

Case I :
If factored design shear force (Vu ) ≤ 0.6
times design shear strength of the cross=
section (Vd). i.e. Vu ≤ 0.6 Vd
Area of web × f yw
where Vd =
3 × γ m0
γm0 = 1.1, area of web = dt w
fyw = yield stress of web

© Copyright Reserved by Gateflix.in No part of this material should be copied or reproduced without permission
(d tw > 67 ε ) , the design bending
strength shall be calculated using one of
the following methods.
a) The bending moment and axial force
acting on the section may be
assumed to be resisted by flanges
only and the web is designed only
4.3 DESIGN PROCEDURE to resist shear.
b) The bending moment and axial force
1. A trial section is selected assuming it is acting on the section may be
going to be plastic section (class 1). assumed to be resisted by the
2. Then it is checked for the class it whole section. In such a case the
belongs. web shall be designed for combined
3. Check for bending strength. shear and normal stresses using
4. Check for shear strength. simple elastic theory in case of semi-
5. Check for the deflection. compact webs and simple plastic
theory in the case of compact and
Case II :If factored design shear force (Vu) plastic webs.
> £ 0.6 times design shear strength of the
cross – section i.e. Vu> 0.6 Vd The design 4.4 SHEAR STRENGTH OF LATERALLY
bending strength shall be taken as Md = Mdv SUPPORTED BEAM
Where, Mdv=design bending strength under The design shear strength of a section is
high shear Mdv is calculated as given below given by
Av fyw
a) Plastic or Compact Section Vd =
1.2 z e f y √3γmo
M dv = M d –β ( M d – M fd ) ≤ Where, Av = Shear area ;
γ m0 fyw = Yield strength of the web
 2V 
2 The shear area may be calculated as given
Where, = β  u –1 below:
 Vd 
For I and Channel Sections
Md = Plastic design moment of the
i) Major axis bending
whole section as calculated in case I.
Hot rolled - htw
Vu = Factored applied force.
Welded - dtw
Vd = Design shear Strength.
ii) Minor axis bending
Mfd = Plastic design strength of the area
Hot rolled or welded - 2btf
of the cross-section excluding the shear
area, considering partial safety factor
For Rectangular Hollow Sections of
𝛾𝛾𝑚𝑚𝑜𝑜 , Ze = Elastic section modulus of the
Uniform Thickness
whole-section.
Ah
i) Loaded parallel to depth (h) –
b) Semi-compact Section (b+h)
ze f y Ab
M dv = ii) Loaded parallel to width (b) –
γ m0 (b+h)
Design bending strength of a section iii) Circular hollow tubes of uniform
which is susceptible to web buckling 2A
thickness −
under shear before yielding ∏

© Copyright Reserved by Gateflix.in No part of this material should be copied or reproduced without permission
iv) Plates and solid bars A  h
Where, Fcdw =  b1 + 2×  t w ×f c
A = Cross-section area:  2
b = Overall breadth of tubular section, Fcdw = ( b1 + h ) t w × f c
breadth of I-section flanges
d = Clear depth of web between flanges
4.6 WEB CRIPPLING
h = Overall depth of the section
tf = Thickness of the flange and
Near the support web of the beam may
tw = Thickness of the web
cripple due to lack of bearing capacity as
4.5 WEB BUCKLING STRENGTH shown in figure below. The crippling
occurs at the root of the radius. IS 800-
Certain portion of beam at supports acts as 2007 has accepted the following formula to
column to transfer the load from beam to find crippling of web.
the support. Hence under this compressive f yw
force the web may buckle. This may happen Fw = ( b1 + n c ) t w ×
under a concentrated load on the beam γm 0
also. The load dispersion angle may be Where,
taken as 45º. Hence, there is need to check b1 = Stiff bearing length
for web buckling. However, the rolled nc = Length obtained by dispersion through
section is provided with suitable thickness the flange to the web junction at a slope 1 :
for web so that web buckling is avoided. 2.5 to the plane of flange
In case of built-up sections it is necessary ⇒ nc = 2.5 tf
to check for buckling of web and provide fyw = Yield stress of the web
web stiffeners.

The care is taken in fixing the web


thickness of rolled steel sections to avoid
Hence as per IS 800-2007, effective web such failure. Hence rolled steel section is
buckling strength is to be found based on selected as a beam section there is no need
the cross -section of web to check for this failure. However, when
 h built-up sections are selected the web
At support, A =  b1 +  tw should be checked for this local failure.
 2
Web buckling strength
 h
= Fcdw =  b1 +  t w fc
 2
And fc is the allowable compressive stress
corresponding to the assumed web column
of effective length = 0.7 d where, d is the
web height.
At concentrated load

© Copyright Reserved by Gateflix.in No part of this material should be copied or reproduced without permission
5 PLASTIC ANALYSIS

5.1 INTRODUCTION In order to find out the fully plastic


Steel has unique physical property, moment of a yielded section of a beam as
ductility, because of which it is able to shown in Fig.1, we employ the force
absorb large deformations beyond the equilibrium equation, namely the total
elastic limit without fracture. Due to this force in compression and the total force in
property, steel possesses a reserve of tension over that section are equal.
strength beyond its yield, which engineers Total compression , C = Total tension , T
have tried to utilize in the plastic method of fy. A1 = fy. A2
design. Plastic design is an aspect of the ∴ A1 = A2
limit state design that extends the A = A1 + A2
structural usefulness up to the plastic A1 = A2 = A/2
strength or ultimate load carrying capacity. Plastic Moment of resistance,
The term plastic has occurred due to the Mp = fy A1 y�1 + fy A2 ���
y2
fact that the ultimate load is found from the A
= fy 2 (y�1 + ���)
y2
strength of steel in the plastic range. Plastic
= fy Zp
analysis is based on idealised stress-strain
curve. The effect of strain hardening is where 𝑍𝑍𝑝𝑝 , the plastic modulus of the
𝐴𝐴
neglected. section= (𝑦𝑦 2
���1 + ���)
𝑦𝑦2
This method is rapid and provides a
rational approach for the analysis of the 5.3 BENDING OF BEAMS SYMMETRICAL
structure. It also provides striking economy ABOUT BOTH AXES
as regards the weight of steel since the The fibres of the beam across the cross
sections required by this method are section are stressed in tension or
smaller in size than those required by the compression according to their position
method of elastic analysis. Plastic analysis relative to the neutral axis and are strained
and design has its main application in the in accordance with Fig. 2
analysis and design of statically
indeterminate framed structures.

5.2 FULLY PLASTIC MOMENT OF A


SECTION
The fully plastic moment Mp, of a section is
defined as the maximum moment of
resistance of a fully plasticized or yielded
cross-section. Fig. 2 Elastic stresses in beams

While the beam remains entirely elastic the


stress in every fibre is proportional to its
strain and to its distance from the neutral
axis. The stress (f) in the extreme fibres
cannot exceed fy. (see Fig. 2)
When the beam is subjected to a moment

© Copyright Reserved by Gateflix.in No part of this material should be copied or reproduced without permission
slightly greater than that, which first
produces yield in the extreme fibres, it does
not fail. Instead the outer fibres yield at
constant stress(fy) while the fibres nearer
to the neutral axis sustain increased elastic
stresses. Fig. 3 shows the stress distribution
for beams subjected to such moments. Such
beams are said to be 'partially plastic' and
those portions of their cross-sections,
which have reached the yield stress, are Fig. 4 Stresses in fully plastic beams
described as 'plastic zones'.
5.4 SHAPE FACTOR
For equilibrium of the cross section,
the areas in compression and tension must be
equal. For a rectangular cross section, the
elastic moment is given by,
bd2
M = 6 fy
The plastic moment is obtained from,
d d bd2
Mp = 2. b. 2 . 4 fy = fy
4

Here the plastic moment Mp is about 1.5


Fig. 3 Stresses in partially plastic beams times greater than the elastic moment
capacity. The ratio of the plastic modulus
The depths of the plastic zones depend (Zp) to the elastic modulus (Z) is known as
upon the magnitude of the applied the shape factor (S).
moment. As the moment is increased, the Shape factor signifies the reserve strength
plastic zones increase in depth, and, it is of the beam section beyond yield point.
assumed that plastic yielding can occur at More shape factor implies, reserve strength
yield stress (fy) resulting in two stress is more beyond yielding.
blocks, one zone yielding in tension and
one in compression. Fig. 4 represents the 5.4.1 SHAPE FACTOR FOR DIFFERENT
stress distribution in beams stressed to this CROSS SECTION
stage. The plastic zones occupy the whole (a) Rectangular Section
of the cross section, and are described as S =1.5
being 'fully plastic'. When the cross section (b) Circle
of a member is fully plastic under a bending S = 1.7
moment, any attempt to increase this (c) I Section
moment will cause the member to act as if S = 1.12 to 1.14
hinged at the neutral axis. This is referred (d) H Section
to as a plastic hinge. S = 1.5
The bending moment producing a plastic (e) Triangle
hinge is called the full plastic moment and S = 2.34
is denoted by 'Mp'. Note that a plastic hinge (f) Diamond Section
carries a constant moment, MP. S=2
(g) Thin Hollow Circular Section
4
S = 𝜋𝜋 = 1.27

© Copyright Reserved by Gateflix.in No part of this material should be copied or reproduced without permission
5.5 LOAD FACTOR (b) A propped cantilever requires two
The load factor is the ratio of the collapse hinges to form a mechanism.
load to the working load: Redundancy, r = 1
No. of plastic hinges formed,
Ultimate Load
Load Factor= Working Load = Working Moment
Plastic Moment =r+1=2

𝑀𝑀𝑝𝑝 𝑓𝑓𝑦𝑦 𝑍𝑍𝑝𝑝


Load Factor= 𝜆𝜆 = =
𝑀𝑀 𝑓𝑓𝑓𝑓

Load Factor= Shape Factor × Factor of Safety

5.6 FUNDAMENTAL CONDITIONS FOR (c) A fixed beam requires three hinges to
PLASTIC ANALYSIS form a mechanism.
Redundancy, r = 2
(i) Mechanism condition: No. of plastic hinges = 2 + 1 = 3
The ultimate or collapse load is reached
when a mechanism is formed. The number
of plastic hinges developed should be just
sufficient to form a mechanism.

(ii) Equilibrium condition:


∑ 𝐹𝐹𝑥𝑥 = 0, ∑ 𝐹𝐹𝑦𝑦 = 0, ∑ 𝑀𝑀𝑥𝑥𝑥𝑥 = 0 Hence the number of hinges needed to form a
mechanism equals the statical redundancy of
(iii) Plastic moment condition: the structure plus one.
The bending moment at any section of the
structure should not be more than the fully 5.7.2 Panel or Sway Mechanism
plastic moment of the section. Fig. 5 shows a panel or sway mechanism for a
portal frame fixed at both ends.
5.7 MECHANISM
When a system of loads is applied to an
elastic body, it will deform and will show a
resistance against deformation. Such a
body is known as a structure. On the other Fig.5 Panel Mechanism
hand if no resistance is set up against
deformation in the body, then it is known 5.7.3 Gable Mechanism
as a mechanism. Fig.6 shows the gable mechanism for a gable
Various types of independent mechanisms structure fixed at both the supports.
are
5.7.1 Beam Mechanism
(a) A simply supported beam has to form
one plastic hinge at the point of
maximum bending moment. Fig.6 Gable Mechanism
Redundancy, r = 0
5.7.4 Joint Mechanism
Fig.7 shows a joint mechanism. It occurs at a
joint where more than two structural
members meet.

© Copyright Reserved by Gateflix.in No part of this material should be copied or reproduced without permission
2. Upper Bound or Kinematic Theorem
A load factor (𝜆𝜆𝑘𝑘 ) computed on the basis of
an arbitrarily assumed mechanism will
always be greater than, or at best equal to
Fig.7 Joint Mechanism
the load factor at rigid plastic collapse (𝜆𝜆𝑝𝑝 )
5.7.5 Combined Mechanism 𝜆𝜆𝑝𝑝 is the lowest value of 𝜆𝜆𝑘𝑘 which can be
Various combinations of independent found.
mechanisms can be made depending upon
whether the frame is made of strong beam 3. Uniqueness Theorem
and weak column combination or strong If both the above criteria are satisfied, then
column and weak beam combination. The the resulting load factor corresponds to its
one shown in Fig.8 is a combination of a value at rigid plastic collapse (𝜆𝜆𝑝𝑝 ).
beam and sway mechanism. Failure is
triggered by formation of hinges at the
bases of the columns and the weak beam
developing two hinges. This is illustrated
by the right hinge being shown
on the beam, in a position slightly away
from the joint.

Fig.8 Combined Mechanism

Note:
The number of independent mechanisms
(n) is related to the number of possible
plastic hinge locations (h) and the number
of degree of redundancy (r) of the frame by
the equation.
n=h–r

5.8 BASIC THEOREMS OF PLASTIC


ANALYSIS
1. Lower Bound or Static Theorem
A load factor (λs ) computed on the basis of
an arbitrarily assumed bending moment
diagram which is in equilibrium with the
applied loads and where the fully plastic
moment of resistance is nowhere exceeded
will always be less than or at best equal to
the load factor at rigid plastic collapse, (λp ).
λp is the highest value of λs which can be
found.

© Copyright Reserved by Gateflix.in No part of this material should be copied or reproduced without permission
GATE QUESTIONS
Topics Page No

1. STRUCTURAL FASTENERS 111

2. TENSION MEMBER 11

3. COMPRESSION MEMBER 111

4. BEAMS 111

5. PLATE GIRDERS & INDUSTRIAL ROOTS 111

6. PLASTIC ANALYSIS 111

© Copyright Reserved by Gateflix.in No part of this material should be copied or reproduced without permission
1 STRUCTURAL FASTENERS

Q.1 Identify the most efficient but joint Q.3 ISA 100 x100 x 10mm (Cross
(with double cover plates) for a sectional area = 1908mm2) serves as
plate in tension from the patterns tensile member. This angle is
(plan views) shown below, each welded to a gusset plate along A and
comprising 6 identical bolts with the B appropriately as shown. Assuming
same pitch and gauge. the yield strength of the steel to be
260 N/mm2 the tensile strength of
the member can be taken to be
approximately
a)

b)

c)

a) 500kN b) 300kN
d)
c) 225kN d) 375kN
[GATE – 2002]
[GATE –2001]
Q.4 When designing steel structures,
Q.2 ISA 100 x 100 x 10mm (Cross one must ensure that local buckling
sectional area = 1908mm2) is in webs does not take place. This
welded along A and B (Refer to check may not be very critical when
figure given below) such that the using rolled steel sections because
lengths of the weld along A and B a) Quality control at the time of
are l1 and l2 respectively. Which of manufacture of rolled sections is
the following is a possibly very good
acceptable combination of l1 and l2? b) Web depths available are small
c) Web stiffeners are in-built in
rolled sections
d) Depth to thickness ratios (of the
web) are appropriately adjusted
[GATE – 2002]

Q.5 An ISMB 500 is used as a beam in a


a) I1 = 60 mm and I2 = 150 mm
multi-storey construction. From the
b) I1 = 150mm and I2 = 60mm
view point of structural design, it
c) I1= 150mm and I2 = 150 mm
can be considered to be 'laterally
d) Any of the above, depending on
restrained' when,
the size of the weld
a) The tension flange is' laterally
[GATE – 2002]
restrained'

© Copyright Reserved by Gateflix.in No part of this material should be copied or reproduced without permission
b) The compression flange is c) The direct stress in compression
`laterally restrained' on the gross sectional area of
c) The web is adequately stiffened axially loaded compression
d) The conditions in (a) and (c) are member shall not exceed 0.6fy.
met d) None of the above.
[GATE – 2002] [GATE -2005]
Q.6 Rivet value is defined as Q.9 A fillet-welded joint of 6 mm size is
a) Lesser of the bearing strength of shown in the figure. The welded
rivet and the shearing strength surfaces meet at 60-90 degree and
of the rivet permissible stress in the fillet weld
b) Lesser of the bearing strength of is 108 MPa. The safe load that can be
rivet and the tearing strength of transmitted by the joint is
thinner plate
c) Greater of the bearing strength
of rivet and the shearing
strength of the rivet
d) Lesser of the shearing strength
of the rivet and the tearing
strength of thinner plate a)162.7 kN b)151.6 kN
[GATE –2004] c)113.4 kN d)109.5 kN
[GATE – 2005]
Q.7 A moment M of magnitude 50 kN-m
is transmitted to a column flange Q.10 In the design of welded tension
through a bracket by using four 20 members, consider the following
mm diameter rivets as shown in the statements:
figure. The shear force induced in I. The entire cross-sectional area
the rivet A is of the connected leg is assumed
to contribute to the effective
area in case of angles.
II. Two angles back-to-back and
tack-welded as per code
requirements may be assumed
to behave as a tee section
III. A check on slenderness ratio
may be necessary in some cases.
a) 250 kN b) 176.8 kN The TRUE statements are
c) 125 kN d) 88.4 kN a) only I and II b)onlyII and III
[GATE – 2004] c) only I and III d) I, II and III
[GATE – 2006]
Q.8 Which one of the following is NOT
correct for steel sections as per IS: Q.11 A steel flat of rectangular section of
800-1984? size 70⨉6 mm is connected to a
a) The maximum bending stress in gusset plate by three bolts each
tension or in compression in having a shear capacity of 15 kN in
extreme fibre calculated on the holes having diameter 11.5 mm. If
effective section of a beam shall the allowable tensile stress in the
not exceed 0.66fy. flat is 150 MPa, the maximum
b) The bearing stress in any part of tension that can be applied to the
a beam when calculated on the
area shall not exceed 0.75fy.

© Copyright Reserved by Gateflix.in No part of this material should be copied or reproduced without permission
flat is permissible bearing stress as 270
MPa in the rivet, the rivet value of
the joint is

a) 56.70 kN b) 43.29 kN
a) 42.3 kN b) 52.65 kN
c) 36.19 kN d) 21.65 kN
c) 59.5 kN d) 63.0 kN [GATE – 2009]
[GATE – 2007]
Q.15 Two plates, subjected to direct
tension, each of 10 mm thickness
Q.12 A bracket connection is made with and having widths of 100 mm and
four bolts of 10mm diameter and 175 mm, respectively are to be fillet
supports a load of 10 kN at an welded with an overlap of 200 mm.
eccentricity of 100mm. The maximum Given that the permissible weld
force to be resisted by any bolt will stress is 110 MPa and the
be permissible stress in steel is 150
Mpa, then length of the weld
required using the maximum
permissible weld size as per IS : 800
— 1984 is

a)5 kN b)6.5 kN
c)6.8 kN d)7.16 kN
[GATE –2007]
Q.13 Rivets and bolts subjected to both
shear stress �τvf,cal � and axial tensile a) 245.3 mm b) 229.2 mm
c) 205.5 mm d) 194.8 mm
stress �σtf,cal � shall be so
[GATE – 2010]
proportioned that the stresses do
not exceed the respective allowable Q.16 For the fillet weld of size's' shown in
stresses τ vf , and σ tf , and the value the adjoining figure the effective
τvf,cal σ throat thickness is
of � + tf,cal � does not
τvf σtf
exceed
a)1.0 b)1.2
c)1.4 d)1.8
[GATE – 2008]

Q.14 A 12 mm thick plate is connected to a) 0.61 s b) 0.65 s


two 8 mm plates, on either side c) 0.70 s d) 0.75 s
through a 16 mm diameter power [GATE – 2011]
driven field rivet as shown in the
figure below. Assuming permissible
shear stress as 90 MPa and

© Copyright Reserved by Gateflix.in No part of this material should be copied or reproduced without permission
Q.17 In a steel plate with bolted
connections, the rupture of the net
section is a mode of failure under
a)tension b)compression
c)flexure d)shear
[GATE – 2012]
a) 30.33 and 20.00
Q.18 Two plates are connected by fillet b) 30.33 and 25.00
welds of size 10 mm and subjected c) 33.33 and 20.00
to tension, as shown in the sketch. d) 33.33 and 25.00
The thickness of each plate is 12 [GATE – 2014]
mm. The yield stress and the
ultimate tensile stress of steel are Q.21 A bracket plate connected to a
250 MPa and 410 MPa respectively. column flange transmits a load of
The welding is done in the working 100 kN as shown in the following
shop (γmw = 1.25). As per the Limit figure. The maximum force for
State Method of IS 800-2007. The which the bolts should be designed
minimum length (rounded off to the is _______ kN.
nearest higher multiple of 5mm) of
each weld to transmit force P equal
to 270 kN is

[GATE – 2015]
Q.22 Prying forces are
a)100 mm b)105mm a) shearing forces on the bolts
c)110 mm d)115 mm because of the joints
[GATE – 2012] b) tensile forces due to the
Q.19 A steel section is subjected to a flexibility of connected parts
combination of shear and bending c) bending forces on the bolts
actions. The applied shear force is V because of the joints
and the shear capacity of the section d) forces due the friction between
is Vs. For such a section, high shear connected parts
force (as per IS:800-2007) is defined [GATE – 2015]
as
a) V > 0.6Vs b)V > 0.7Vs Q.23 Two plates are connected by fillet
c) V > 0.8Vs d) V > 0.9 Vs welds of size 10 mm and subjected
[GATE – 2013] to tension, as shown in the figure.
The thickness of each plate is 12
Q.20 The tension and shear force (both in mm. The yield stress and the
kN) in each bolt of the joint, as ultimate tensile stress of steel are
shown below, respectively are 250 MPa and 410 Mpa, respectively.
___________

© Copyright Reserved by Gateflix.in No part of this material should be copied or reproduced without permission
The welding is done in the The resultant force (in kN, up to one
workshop (γmw =1.25). decimal place) in bolt 1 is ___________
[GATE – 17]

Q.26 A fillet weld is simultaneously


subjected to factored normal and
shear stress of 120 MPa and 50 MPa,
respectively. As per IS 800:2007, the
equivalent stress (in MPa, up to two
decimal place) is_____________
[GATE – 18]
Q.27 Four bolts P, Q, R and S of equal
diameter are used for a bracket
As per the Limit State Method of IS
subjected to a load of 130 kN as
800: 2007, the minimum length
shown in the figure.
(rounded off to the nearest higher
multiple of 5 mm) of each weld to
transmit a force P equal to 270 kN
(factored) is
(a) 90 mm (b) 105 mm
(c) 110 mm (d) 115 mm
[GATE – 2016]

Q.24 Two bolted plates under tension


with alternative arrangement of bolt
holes are shown in figures 1 and 2.
The hole diameter, pitch, and gauge
length are d, p and g, respectively.

a) p 2 > 2gd b) p 2 < 4gd


c) p 2 > 4gd d) p > 4gd The force in bolt P is
(a) 32.50 kN (b) 69.32 kN
[GATE – 16] (c) 82.50 kN (d) 119.32 kN
[GATE – 18]
Q.25 A column is subjected to a load Q.28 In a fillet weld, the direct shear stress
through a bracket as shown in the and bending tensile stress are 50 MPa
figure
and 150 MPa, respectively. As per IS
800:2007, the equivalent stress (in
MPa, up to two decimal places) will
be ________________
[GATE – 18]

© Copyright Reserved by Gateflix.in No part of this material should be copied or reproduced without permission
ANSWER KEY:
1 2 3 4 5 6 7 8 9 10 11 12 13 14
(a) (a) (a) (d) (b) (a) (b) (d) (c) (d) (a) (d) (c) (b)
15 16 17 18 19 20 21 22 23 24 25 26 27 28
(b) (b) (a) (b) (a) (d) 156.20 (b) (b) (c) 6 147.99 69.32 173.2

EXPLANATIONS

Q.1 (a)
For main plate critical most section
is outermost section for that a outer
most section net area should be
more to resist more load in all four
case we will get more A net in case (a)

Q.2 (a)
The C.G of the angle is close to B
compared to A. Therefore, the length
of the weld, l2 shall be more than l1
so that the algebraic sum of Force due to moment effect,
moments of the two welds l1 & l2 M.r
Fm =
about C.G of the angle will be zero ∑ r2
and there is no eccentricity.
=r 502 + 502
Q.3 (a) = 70.71mm
= 0.0707m
Tensile strength of the member 50 × 0.0707
= Ag⨉fy = 1908⨉260 = 500 kN Fm =
4 × 0.0707 2
Q.4 (d) = 176.8kN

Q.5 (b ) Q.8 (d)


Q.6 (a)
Q.9 (c)
Q.7 (b)

© Copyright Reserved by Gateflix.in No part of this material should be copied or reproduced without permission
40
θ
Cos= = 0.8
50
Pr = 2.52 + 52 + (2 × 2.5 × 5 × 0.8)
Pr = 7.16 kN

Q.13 (c)
Total length of weld, Q.14 (b)
l = 100+50+100 = 250 mm Gross diameter of rivet,
Throat thickness of fillet, D = 16 + 1.5 = 17.5 mm
t = 0.7⨉ size Strength of rivet in shearing,
= 0.7⨉ 6 = 4.2 mm π
Ps = 2 × 4 × d2 × τvf (Double
Strength, P = l × t × τvf
= 250⨉4.2⨉108 shear for butt joints)
π
= 113.4 kN =2 × × (17.5 ) × 90
2

4
Q.10 (d) Ps = 43.29 kN
Strength of rivet in bearing,
Q.11 (a) Pb = d.t.σpf = 17.5⨉12⨉270
Steel flat 70⨉6 mm Pb = 56.7 kN
Capacity of each bolt = 15 kN,
No. of bolt = 3 Rivet value R v is smaller of Ps & Pb
So, maximum load transferred R v = 43.29 kN
= 3⨉15 = 45 kN
Tearing Strength of flat Q15 (b)
= (70-2⨉11.5)⨉6⨉150
= 42300 N = 42.3 kN
Maximum tension that can be
applied to the flat
= minimum of above two
= 42.3 kN

Q.12 (d)
Force due to axial load effect, Max. size of weld as per code
w 10 Smax = T - 1.5 = 10 - 1.5 = 8.5 mm
Fa = = = 2.5 kN
N 4 Size, S = 8.5 mm
Force due to moment effect, Throat thickness
M.r ‘t’ = 0.7⨉ 8.5 = 5.95 mm
Fm = ; Strength of the plate ‘P’ is
∑ r2
P = (b.T) σst
r = 302 +402 = 50mm = 0.05m
M = W × e = 10 × 0.1 = 1 kNm = (100 ×10) ×150
1× 0.05 P = 150 ×103 N:
Fm = Let l = total length of the weld
4 × 0.052
Fm = 5 kN required.
Strength of the weld,
Resultant force, Ps = lt × τ vf = l × 5.95 × 110
Pr = Fa2 +Fm2 +2Fa Fm cos θ Equating P = Ps

© Copyright Reserved by Gateflix.in No part of this material should be copied or reproduced without permission
150⨉103 = l⨉5.95⨉110 Pu sinθ 3 Pu 3
= = × 250 = 25 kN
l = 229.2 mm 6 5 6 5×6

Q.16 (b) Q.21 (156.20)


Throat thickness, t = k.S P 100
F=
D =
   = 20 kN
n 5
Angle k
between
fusion faces
60° - 90° 0.70
91° - 100° 0.65
101° - 106° 0.6
107° - 113° 0.55
114° - 120° 0.50

Q.17 (a)

Q.18 (b) (P. d)r 100 × 600 × 75√2


Design strength of fillet weld, Ft =
∑ r2
= 2 = 141.42 kN
4 × �75√2�
fu
Pd = Lw t
√3γmw
410 FR = FD2 +Ft2 +2×FD ×Ft cosθ
270⨉103 = 2Lw (0.7 × 10) ×
√3 × 1.25 1
( 20 ) + (141.42 )
2 2
L w = 101.84mm +2×20×141.42×
2
Rounded off to 105 mm = 156.20kN
1
Q.19 (a) Cos θ =
As per clause 9.2.1 (IS: 800-2007) 2
for combined shear and bending: ⇒ θ = 45°
Factored value of applied shear
force is greater than or equal to Q.22 (b)
shear strength for high shear. i.e., V
> 0.6 Vs Q.23 (b)
Maximum force carried by plates,
Q.20 (d) A g f y 100 × 12 × 250
= P= = 272.73kN
γ m0 1.1
P
Load carried by each weld =
2
= 136.36 kN
3 4 4 For minimum length of weld,
sin θ= , cos θ= Pu cos θ= .Pu
5 5 5 Strength of weld=Load carried by
3 weld
Pu sin θ =   .Pu 𝑓𝑓𝑢𝑢
5 𝑙𝑙𝑤𝑤 × 𝑠𝑠 × = 136.36 × 103
Tension in each bolt = √3𝛾𝛾𝑚𝑚1
Pu cosθ 4Pu 4
= = × 250 =33.33kN
6 5×6 30 410
⇒ l w × (10 × 0.7) × = 136.36 ×103
Shear in each bolt = 3×1.2

© Copyright Reserved by Gateflix.in No part of this material should be copied or reproduced without permission
lw = 102.9mm next multiple of 5 is ≈ Bending tensile stress (σ) = 150 MPa
105mm fR = �σ2 + 3τ2
= √1502 + 3 × 1502
Q.24 (c) = 173.20 MPa
p 2 > 4gd
This question can be solved by trick,
Option (b) and (d) are not
dimensionally correct.

Q.25 (6.0 kN)


P 10
F1 = n = 4 = 2.5 kN

Pe 10×15
F2 = ∑ ri 2
r1 = 4×52
× 5 = 7.5 kN

𝐹𝐹𝑟𝑟 = �𝐹𝐹1 2 + 𝐹𝐹2 2 + 2𝐹𝐹1 𝐹𝐹2 cos 𝜃𝜃

= √2.52 + 7.52 + 2 × 2.5 × 7.5 × cos 135°

= 6.0 kN

Q.26 (147.99 MPa)


Equivalent stress = √𝜎𝜎 2 + 3𝜏𝜏 2
= 147.99 MPa

Q.27 (b) 69.32 kN


Direct shear in bolt P,
130
F1 = 4 = 32.5 kN
Shear due to twisting moment
130×200×√1202 +502
F2 = 2
4×�√1202 +502 �

𝐹𝐹2 = 50 𝑘𝑘𝑘𝑘
120
𝜃𝜃 = tan−1 = 67.38°
50
Resultant force in bolt P

= �𝐹𝐹1 2 + 𝐹𝐹2 2 + 2𝐹𝐹1 𝐹𝐹2 cos 𝜃𝜃

=
√32.52 + 502 + 2 × 32.5 × 50 × cos 67.38°

= 69.32 kN

Q.28 (173.20MPa)

Direct shear stress (τ) = 50 MPa

© Copyright Reserved by Gateflix.in No part of this material should be copied or reproduced without permission
2 TENSION MEMBER

Data for Q.1 and Q.2 is given below:


A truss tie consisting of 2 ISA75 × 75 × 8mm a) 3650 mm2 b) 3450 mm2
carries a pull of 150 kN. At ends the two c) 3076 mm2 d) 2899 mm2
angles are connected, one each on either [GATE – 04]
side of a 10 mm thick gusset plate, by 18
mm diameter rivets arranged in one row. Q.4 The permissible stress in axial
The allowable stresses in rivet are tension in steel member on the net
f s = 90N / mm 2 and f br = 250 N / mm 2 . effective area of the section shall not
exceed the following value (fy is the
Q.1 Maximum tensile stress in the tie in yield stress)
N/mm2 is a) 0.80fy b) 0.75fy
a) 93.6 b) 87.5 c) 0.60fy d) 0.50fy
c) 77.2 d) 66.0 [GATE – 05]
[GATE – 03]

Q.2 Minimum number of rivets required


at each end is
a) 2 b) 3
c) 4 d) 5
[GATE – 03]

Q.3 Two equal angles ISA 100mm ⨉


100mm of thickness 10mm are
placed back-to-back and connected
to the either side of a gusset plate
through a single row of 16 mm
diameter rivets in double shear. The
effective areas of the connected and
unconnected legs of each of these
angles are 775 mm2 and 950 mm2,
respectively. If these angles are NOT
tack riveted, the net effective area of
this pair of angle is

ANSWER KEY:

1 2 3 4
(a) (c) (d) (c)

© Copyright Reserved by Gateflix.in No part of this material should be copied or reproduced without permission
EXPLANATIONS

Q.1 (a) Pb = d.t.τ vf


If angles are not tack riveted they
=19.5 ×10 × 250 = 48.75 ×103 N
behave individually.
= 48.75 kN
Gross diameter of rivet,
Rivet value (Rv) is minimum of
d = 18 + 1.5 = 19.5 mm
above two value
Rv = 48.75 kN
Number of rivets; N = load/Rv
Provide 4 rivets.

Q.3 (d)
If two angles are not tack riveted
Only connected to gusset plate (Not they will be considered as single
tack riveted) angle connected on one side of
𝐴𝐴𝑛𝑛𝑛𝑛𝑛𝑛 = 2 × (𝐴𝐴1 + 𝑘𝑘𝐴𝐴2 ) gusset plate.
A1 = Net Area of connected leg A1 = 775 mm 2 ; A 2 = 950 mm 2
A 2 = Gross c/s area of unconnected 3A1 3×775
K1 = = = 0.71
leg 3A1 +A 2 (3×775)+950
 8  K1 = 0.71
Q A1 = 75 − − 19.5 × 8
 2  A net = 2(A1 + k1A 2 )
2
= 412 mm = 2 × (775+0.71× 950)
A 2 = [ 75 − 4] × 8 =568
∴ Anet = 2899 mm 2
3A1
Q K1 = = 0.685
3A1 +A 2 Q.4 (c)
Q A net = 2[412 + 0.685 × 568] Permissible stress in axial tension
is 0.60fy as per the code of practice,
= 1602.2 mm 2 IS : 800
Q maximum tensile stress,
Pt 150 ×103
σ cal =
= = 93.61 N / mm 2
A net 1602.2

Q.2 (c)
Strength of rivet in shear;
π
PS = 2× d 2 ×τ vf (Rivets are under
4
double shear)
π
PS = 2× (19.5) 2 × 90 = 53.76 ×103 N
4
= 53.76 kN
Strength of rivet in bearing

© Copyright Reserved by Gateflix.in No part of this material should be copied or reproduced without permission
3 COMPRESSION MEMBER

Q.1 Consider the following two a) 93.2 mm b) 62.7 mm


statements related to structural c) 46.6 mm d) 29.8 mm
steel design, and identify whether [GATE – 2004]
they are true or FALSE.
I. The Euler buckling load of a Q.4 Consider the following
slender steel column depends statements
on the yield strength of steel. I. Effective length of a battened
II. In the design of laced column, column is usually increased to
the maximum spacing of the account for the additional
lacing does not depend on the load on battens due to the
slenderness of column as a lateral expansion of columns.
whole. II. As per IS: 800-1984,
a) Both statements I and II are permissible stress in bending
TRUE compression depends on both
b) Statement I is TRUE, and Euler buckling stress & the
Statement II is FALSE yield stress of steel.
c) Statement I is FALSE, and III. As per IS: 800-1984, the
Statement II is TRUE effective length of a column
d) Both Statements I & II are effectively held in position at
FALSE both ends but not restrained
[GATE – 2001] against rotation, is taken to be
greater than that in the ideal
Q.2 In the design of lacing system for end conditions.
a built-up steel column, the The TRUE statements are
maximum allowable slenderness a)only I and II b)only II and III
ratio of a lacing bar is c)only I and III d)I, II and III
a) 120 b) 145 [GATE – 2006]
c) 180 d) 250
[GATE – 2003] Q.5 The square root of the ratio of
moment of inertia of the cross-
Q.3 A strut in a steel truss is section to its cross-sectional area
composed of two equal angles ISA is called
150 mm x 150 mm of thickness a)second moment of area
100mm connected back-to-back b)slenderness ratio
to the same side of a gusset plate. c)section modulus
The cross sectional area of each d)radius of gyration
angle is 2921 mm 2 and moment [GATE –2009]
of inertia ( I xx = I yy ) is 6335000
Q.6 Consider the following
mm4. The distance of the centroid statements for a compression
of the angle from its surface member:
( Cx = Cy ) is 40.8 mm. The I. The elastic critical stress in
minimum radius of gyration of compression increases with
the strut is decrease in slenderness ratio.

© Copyright Reserved by Gateflix.in No part of this material should be copied or reproduced without permission
II. The effective length depends on Q.9 Consider two axially loaded
the boundary conditions at its columns, namely, 1 and 2, made
ends. of a linear elastic material with
III. The elastic critical stress in Young’s modulus 2⨉105 MPa,
compression is independent of square cross-section with side 10
the slenderness ratio. mm, and length 1 m. For column
IV. The ratio of the effective 1, one end is fixed and the other
length to its radius of gyration is end is free. For Column 2, one end
called as slenderness ratio. is fixed and the other end is
The TRUE statements are pinned. Based on the Euler’s
(a) II and III (b) III and IV theory, the ratio (up to one
(c) II,III and IV (d) I,II and IV decimal place) of the buckling
[GATE – 2009] load of column 2 to the buckling
load of column 1 is________
Q.7 The ratio of the theoretical critical [GATE – 2017]
buckling load for a column with
fixed ends to that of another Q.10 A column of height h with
column with the same rectangular cross section of size
dimensions and material, but of a × 2a has a buckling load of P.
with pinned ends, is equal to If the cross-section is changed to
(A) 0.5 (B) 1.0 0.5a × 3a and its height changed
(C) 2.0 (D) 4.0 to 1.5h, the buckling load of the
[GATE – 2012] redesigned column will be
(a) P/12 (b)P/4
Q.8 Two steel columns P (length L and (c) P/2 (d)3P/4
yield strength fy = 250 MPa) and [GATE – 2018]
Q (length 2L and yield strength fy
= 500 MPa) have the same cross-
sections and end conditions. The
ratio of buckling load of column P
to that of column Q is:
(A) 0.5 (B) 1.0
(C) 2.0 (D) 4.0
[GATE – 2013]

ANSWER KEY:
1 2 3 4 5 6 7 8 9 10
(b) (b) (c) (a) (d) (d) (d) (d) 8 (a)

© Copyright Reserved by Gateflix.in No part of this material should be copied or reproduced without permission
EXPLANATIONS

Q.1 (b) Q.4 (a)


π EI
2 For two ends hinged condition
Euler buckling load = (statement No. III), Ie = L; hence III
l2
σy is wrong.
E=
ε
Q Euler buckling load depends on yield Q.5 (d)
strength for riveted or welded lacing Radius of Gyration
system,
L Moment of inertia
= 50 =
rmin Cross Sectional Area
or Q.6 (d)
0.7 times 'λ' (slenderness ratio) of π2 E
fcc = 𝑳𝑳 𝟐𝟐
member as a whole whichever is less. � �
𝒓𝒓

Q.2 (b)
The elastic critical stress in
The slenderness ratio of a lacing bar
compression increases with decrease
should not exceed 145 as per the code
in slenderness ratio.
(IS : 800)
Q.7 (d)
Q.3 (c)
Theoretical critical buckling load
π2 EI
Pcr = (L)2
1
Pcr ∝
L2

Pcr fixed Lpinned 2


=� �
Pcr pinned Lfixed

Pcr fixed L 2
= �0.5 L�
Total area Pcr pinned
= A = 2 × a = 2 × 2921 = 5842mm 2
Pcr fixed
Since X-X axis of the built-up section =4
Pcr pinned
passes through the C.G of individual
Angle, minimum moment of inertia
occurs about axis XX. Q.8 (d)
Minimum moment of inertia as about X
Pcr P L 2
– X is = � LQ �
Pcr Q
I XX = 2 × I XX of each angle P

= 2 × 6335000 mm 2 Pcr fixed 2L 2


= �L�
I Pcr pinned
∴ rmin = XX
A
Pcr fixed
2 × 6335000 =4
= = 46.6 mm Pcr pinned
5842
Q.9

© Copyright Reserved by Gateflix.in No part of this material should be copied or reproduced without permission
𝑃𝑃𝑐𝑐𝑐𝑐 2 𝐿𝐿 2
= �𝐿𝐿1 �
𝑃𝑃𝑐𝑐𝑐𝑐 1 2

2
𝑃𝑃𝑐𝑐𝑐𝑐 2 2𝐿𝐿
= �1 �
𝑃𝑃𝑐𝑐𝑐𝑐 1 𝐿𝐿
√2

𝑃𝑃𝑐𝑐𝑐𝑐 𝑓𝑓𝑓𝑓𝑓𝑓𝑓𝑓𝑓𝑓
=8
𝑃𝑃𝑐𝑐𝑐𝑐 𝑝𝑝𝑝𝑝𝑝𝑝𝑝𝑝𝑝𝑝𝑝𝑝

Q.10 (a)
Case- I

π2 EImin
Pcr =
leff 2

2a(a)3 a4
Imin = =
12 6

leff 2 = h2

a4
π2 E 1 π2 Ea4
∴ Pcr 1 = P = 6
=6
h2 h2

π2 Ea4
= 6P
h2

Case – II

π2 EImin
Pcr =
leff 2

3a(0.5a)3 a4
Imin = = 32
12

9
leff 2 = (1.5h)2 = 4 h2

1
π2 E a4 1 π2 Ea4
∴ Pcr2 = 32
9 2 = 72
h h2
4

1 P
Pcr2 = 72 × 6P = 12

P
Pcr2 = 12

© Copyright Reserved by Gateflix.in No part of this material should be copied or reproduced without permission
4 BEAMS

Q.1 Group-I contains some elements in Q.3 An unstiffened web I section is


design of a simply supported plate fabricated from a 10mm thick plate
girder and Group-II gives some by fillet welding as shown in the
qualitative locations on the girder. figure. If yield stress of steel is
Match the items of two lists as per 25OMPa, the maximum shear load
good design practice and relevant that section can take is
codal provisions.
Group I
P. flange splice
Q. web splice
R. bearing stiffeners
S. horizontal stiffener
Group II
1. at supports (minimum) a) 750kN b) 350kN
2. away from centre of span c) 337.5 kW d) 300kN
3. away from support [GATE – 2005]
4. in the middle of span
Q.4 The adjoining figure shows a
5. longitudinally somewhere in the
schematic representation of a steel
compression flange
plate girder to be used as a simply
Codes:
supported beam with a concentrated
P Q R S
load. For stiffeners, PQ (running
a) 2 3 1 5
along the beam axis) and RS (running
b) 4 2 1 3
between the top and bottom
c) 3 4 2 1
flanges) which of the following pairs
d) 1 5 2 3
of statements will be TRUE?
[GATE – 2003]
Q.2 A square steel slab base of area 1 m2
is provided for a column made of
two rolled channel sections. The 300
mm x 300 mm column carries an
axial compressive load of 2000 kN.
The line of action of the load passes
through the centroid of the column a) i) RS should be provided under the
section as well as of the slab base. concentrated load only
The permissible bending stress in ii) PQ should be placed in the
the slab base is 185 MPa. The tension side of the flange.
required minimum thickness of the b) i) RS helps to prevent local buckling
slab base is of the web.
a) 110 mm b) 89 mm ii) PQ should be placed in the
c) 63 mm d) 55 mm compression side of the flange.
[GATE –2004] c) i) RS should be provided at supports
ii) PQ should be placed along the
neutral axis

© Copyright Reserved by Gateflix.in No part of this material should be copied or reproduced without permission
d) i) RS should be provided away from 500 cm3 , 650 cm3 and 200MPa,
points of action of concentrated loads. respectively. The design flexural
ii) PQ should be provided on the capacity (expressed in kN-m) of the
compression side of the flange. section is ____.
[GATE – 2011] [GATE – 2016]
Q.5 The semi-compact section of a
laterally unsupported steel beam
has an elastic section modulus,
plastic section modulus and design
bending compressive stress of

ANSWER KEY:
1 2 3 4 5
(a) (d) (d) (b) 100

© Copyright Reserved by Gateflix.in No part of this material should be copied or reproduced without permission
EXPLANATIONS

Q.1 (a)
Flange splice: Flange resists V = 300 ×10 ×100 = 300 ×103 N
moment. Since moment will be = 300 kN
generally maximum at centre, there Q.4 (b)
should not be flange joint at the Vertical stiffeners are provided to
centre. prevent buckling of web.
Web spice: Web resists shear force. Longitudinal stiffener (PQ) is
Maximum shear force occurs at provided to prevent bucking of web
support. Hence avoid the web splice due to compression. Hence it should
at the support. be provided on compression side.
Bearing stiffener: It is to avoid
compression buckling. Hence Q.5 (100)
provided near the compressive As per IS 800, the design bending
flange. strength of laterally unsupported
beam as governed by lateral
Q.2
(d) torsional buckling is:
a= b= 1000 − 300 / 2 = 350mm M d = βb .Zp .f bd
P 2000 ×103 Z
=w = = 2
2N / mm 2 β b = e for semi compact section
Actual area (1000) Zp
Thickness ‘t’ Z
So M d = e .Zp .f bd
Zp
3w  2 b 2 
=t a −  Ze.f bd = 500 ×103 × 200 ×10−6 = 100kN-m
σ bs  4
3× 2  3502 
=t  350 2
− 
185  4 
∴ t = 55mm

Q.3 (d)
V
τva =
d×tw
=
V d.t w . τv.a
τ v.a = 0.4fy = 0.4 × 250 = 100 N/mm 2

© Copyright Reserved by Gateflix.in No part of this material should be copied or reproduced without permission
5 PLATE GIRDERS & INDUSTRIAL ROOTS

Q.1 Which of the following elements of a a) the bending stresses in the


pitched roof industrial steel building flanges
primarily resists lateral load parallel b) the vertical shear force at the
to the ridge? section
a) Bracings b) purloins c) the horizontal shear force
c) Truss d) columns between the flanges and the web
[GATE – 2003] plate
d) the forces causing buckling in
Q.2 In a plate girder, the web plate is the web
connected to the flange plates by [GATE – 2004]
fillet welding. The size of the fillet
welds is designed to safely resist

ANSWER KEY:
1 2
(a) (c)

© Copyright Reserved by Gateflix.in No part of this material should be copied or reproduced without permission
6 PLASTIC ANALYSIS

Q.1 A prismatic beam (as shown below)


has plastic moment capacity of Mp,
then the collapse load P of the beam
is

a)16.5 MP/L b)15.5 MP/L


c)15.0 MP/L d)16.0 MP/L
[GATE-2015]
2M p 4M p
a) b)
L L
Q.4 A propped cantilever of span L
6.M p 8M p
c) d) carries a vertical concentrated load
L L at the mid-span. If the plastic moment
[GATE-2014] capacity of the section is Mp, the
magnitude of the collapse load is
8M P 6M P
Q.2 For formation of collapse a) b)
L L
mechanism in the following figure, 4M P 2M P
the minimum value of Pu is cMP/L. c) d)
Mp and 3Mp denote the plastic L L
moment capacities of beam sections [GATE-2016]
as shown in this figure. The value of
c is _______ Q.5 A fixed-end beam is subjected to a
concentrated load (P) as shown in
the figure. The beam has two
different segments having different
plastic moment capacities (Mp, 2Mp)
as shown.

[GATE-2015]

Q.3 A fixed end beam is subjected to a The minimum value of load (P) at
load, W at 1/3rd span from the left which the beam would collapse
support as shown in the figure. The (ultimate load) is
collapse load of the beam is a) 7.5M p / L b) 5.0M p / L
c) 4.5M p / L d) 2.5M p / L
[GATE-2016]

© Copyright Reserved by Gateflix.in No part of this material should be copied or reproduced without permission
Q.6 The prismatic propped cantilever
beam of span L and plastic moment
capacity Mp is subjected to a concentrated
load at its mid-span. If the collapse load of
Mp
the beam is α , the value of α is__________
L

[GATE-2018]

ANSWER KEY:
1 2 3 4 5 6
(c) 13.33 (c) (b) (a) 6

© Copyright Reserved by Gateflix.in No part of this material should be copied or reproduced without permission
EXPLANATIONS

Q.1 (c) Plastic hinges formed = 3


Degree of static indeterminacy DS =0 (1)
∴ Number of plastic hinges=Ds+1 = 1
From principal of virtual work
L  P L 
-M p .θ - M P .θ + P.  .θ   - .  .θ  =0
2  2 3 

6M P
⇒P=
L
θ
α=
Q.2 (13.33) 2
Mechanism-I θ = 2α
L
3M P . θ + M P ( 2θ ) + MP.θ = Pu × ×θ θ=
V
4 L/3
L L
⇒ 6M p .θ = Pu .  .θ ∆= θ
4 3
Mp α2L
⇒ Pu = 24 ∆=
L 3
θ = 2α
2M P θ + 2M P θ + 2M P α + M P α= W∆
θ
2M P θ + 2M P θ +M P θ + M P
2
L
= W× θ
3
WL 16.5
5.5M P = ⇒ W= MP
Mechanism-II 3 L
1. φ = 3.θ ⇒ φ = 3θ (2)
L
3M P .θ + M P ( θ + φ ) + M P .φ= Pu × φ
4
⇒ 3M P .θ + M P ( θ + 3θ ) + M P .3θ
L
= Pu × 3 θ×
4
L
⇒ 10M P .θ= Pu × 3 θ×
4
40 M P M 2M P θ + M P θ + M P ∆ + M P θ= w × ∆
⇒ . =13.33 P
3 L L L
5 M Pθ = W θ
So, C = 13.33 3

Q.3 (c)

© Copyright Reserved by Gateflix.in No part of this material should be copied or reproduced without permission
Mp 15M p
15 =W P = 7.5Mp / L
L 2L
15M P
Lowest is collapse load Mechanism -II
L

Q.4 (b)

21 41
θ= φ
3 3
L θ = 2φ
− M p θ − M p θ − M p θ + P ×  θ =
0
2
PLθ  2L 
 3M pθ = -2M pθ-2M pθ-2M p f-M p f+P   θ = 0-4
2  3 
6M p 2PL
P= M pθ-3M p f+ θ=0
L 3
 θ  2PL
-4M pθ-3M p   + θ=0
Q.5 (a) 2 3
11 2PL
=
M pθ θ
2 3
33
=P = M p 8.25M p
4
So the minimum value of load = 7.5
Mp/L

Q.6 (6)
Mechanism -I

 2Lθ 
-2M pθ - M pθ - M pθ - M pθ +P  = 0
 3 
2PL
- 5M pθ+ θ=0
3
2PL
=5M p
3

© Copyright Reserved by Gateflix.in No part of this material should be copied or reproduced without permission
𝐿𝐿
−3𝑀𝑀𝑝𝑝 𝜃𝜃 = −𝑊𝑊 𝜃𝜃
2
𝑀𝑀𝑝𝑝
6 = 𝑊𝑊
𝐿𝐿

∴ 𝛼𝛼 = 6

© Copyright Reserved by Gateflix.in No part of this material should be copied or reproduced without permission
ASSIGNMENT QUESTIONS

Q.1 A singly reinforced balanced section l0


d) +b w Or `b’ whichever is lesser
designed by limit state method will  l0 
have   +4
b
1. Lesser depth compared to designed
by working stress method
Q.4 A deep continuous beam has
2. Higher value of steel compared to
designed by working stress method
effective depth of 500 mm and
3. Lower cost compared to designed effective cover of 50 mm. Its
by working stress method maximum effective length is
Which of these statements is/are a) 1375 mm
correct? b) 1100 mm
a) 1 and 2 b) 2 and 3 c) 1275 mm
c) 3 and 1 d) Only 3 d) cannot be determined from the
given data
Q.2 Which one represents correct
deflected profile corresponding to Q.5 If a beam of cross-section 350 mm ×
section 1-1for a two way simply 600 mm is subjected to torsional
supported RCC slab subjected to moment of T = 50 KN-m then
UDL as shown in the figure below? equivalent shear will be
a) 129 kN b) 229 kN
c) 329 kN d) Zero
Q.6 For a continuous RC beam, match
list-1(condition) with list-2
a) b) (Placement of live load) and select
the correct answer using the codes
given below the lists:
c) d) List-1
A. For maximum sagging moment
in the span
Q.3 Which expression represents B. For maximum hogging moment
effective flange width of isolated at a support
reinforced concrete T- beam? C. For maximum hogging moment
Symbols have their usual meaning. in a span
l List- 2
a) 0 +b w +6d f Or `b’ whichever is lesser
6 1. The spans adjoining the span as
l
b) 0 +b w +3Df Or `b’ whichever is lesser well as alternate span
12 2. The same span as well as
0.5l0 alternate span
c) +b w Or `b’ whichever is lesser 3. The adjacent span on both side
 l0 
 +4  of this support as well as spans
b 
alternate to these

© Copyright Reserved by Gateflix.in No part of this material should be copied or reproduced without permission
4. Span next to the adjacent spans of b) 0.40d
the support plus alternate spans c) 0.45 d
A B C d) Dependent on grade of concrete
a) 2 3 1 also
b) 1 4 2
c) 2 4 1 Q.11 As per the provision of IS: 456-2000,
d) 1 3 2 in the limit state method for design
of beams, the limiting value of the
Q.7 For a continuous slab of 3 m × 3.5 m depth of neutral axis in a reinforced
size the minimum overall depth of concrete beam of effective depth `d’
slab to satisfy vertical deflection is given as
limits is a) 0.53 d
a)5 cm b) 7.5 cm b) 0.48 d
c) 10 cm d) 12 cm c) 0.46 d
d) Any of the above depending on
Q.8 Which one of the following the different grade of steel
statements about the percentage of
tensile steel required to produce a Q.12 The span to depth ratio limit is
balanced reinforced concrete specified in IS: 456-2000 for the
section is correct? reinforced concrete beam, in order
The required percentage of steel to ensure that the
a) Reduces as the yield strength of a) Tensile crack width is below a
steel increases limit
b) Remains unchanged irrespective b) Shear failure is avoided
of the yield strength of steel c) Stress in the tension
c) Is the same for a given quality of reinforcement is less than the
steel irrespective of whether allowable value
working stress d) Deflection of the beam is below a
method is followed or ultimate limiting value
load method used
d) Is only function of the modulus Q.13 A floor slab of thickness t is cast
of elasticity of steel monolithically transverse to a
rectangular continuous beam of
Q.9 The effective depth of the singly span L, and width, B. If the distance
reinforced rectangular beam is 30 between two consecutive points of
cm. The section is over reinforced contra flexure is, L 0 , the effective
and the neutral axis is 12 cm below width of compression flange at a
the top. If the maximum stress continuous support is
attained by concrete is 50 kg/cm 2
a)B b) L
and modular ratio is 18, then stress 3
developed in the steel would be c)B+ 12t d) B+ 6t+ l0
a) 1800 kg/cm 2 b) 1600 kg/cm 2 6

c) 1350 kg/cm 2 d) 1300 kg/cm 2


Q.14 The total compressive force at the
time of failure of a concrete beam
Q.10 If permissible stress in steel in
section of a width `b’ without
tension is 140 N/mm 2 , then the considering the partial safety factor
depth of neutral balance section of the material is
using working stress method is
a) 0.36 f ck b X u b) 0.54 f ck b X u
a) 0.35d

© Copyright Reserved by Gateflix.in No part of this material should be copied or reproduced without permission
c) 0.66 f ck b X u d) 0.8 f ck b X u 4  fy 
b) <  +0.002 
 1.15E s 
Q.15 What is the modular ratio to be used
 fy 
in the analysis of RC beam using c) Exactly equal to  +0.002 
working stress method if the grade  1.15E s 
of concrete is M 20? d) < 0.002
a)18.6 b)13.3 Where, F y = Characteristic strength
c)9.9 d)6.5
Q.16 A simply supported RC beam having of steel, & E S = Modulus of elasticity
clear span 5 m and support width of steel
300 mm has the cross-section as Q.20 Consider the following statements. In
shown in the figure below? an under-reinforced concrete beam,
1) Actual depth of neutral axis is
less than the critical depth of
neutral axis
2) Concrete reaches ultimate stress
prior to steel reaching the
ultimate stress
What is the effective span of the 3) Moment of resistance is less than
beam as per IS: 456? that of balanced sections.
a) 5300 mm b) 5400 mm 4) Lever arm of resisting couple is
c) 5200 mm d) 5150 mm less than of balanced section
Which of these statements is/are
Q.17 What is the adoptable maximum correct?
spacing between vertical stirrups in a) 1 and 2 b)1 and 3
a RCC beam of a rectangular cross- c) 2, 3 and 4 d)1 and 4
section having an effective depth of
300 mm Q.21 A doubly reinforced concrete beam
a) 300 mm b) 275 mm ahs effective cover `d’ to the centre
c) 250 mm d) 225 mm
of compression reinforcement. ` xu ’
Q.18 In limit state design method, the is the depth of neutral axis and `d’ is
moment of resistance for a balanced the effective depth to centre of
section using M20 grade concrete tension reinforcement. What is the
and HYSD steel of grade Fe 415 is maximum strain in concrete at the
level of compression reinforcement?
given by M u ,lim = Kbd 2 , What is the
a) 0.0035 (1- d’/d)
value of K? b) 0.0035 (1-d’/ xu )
a)2.98 b)2.76
c) 1.19 d) 0.89 c) 0.002 (1-d’/ xu )
d) 0.002(1- d’/d)
Q.19 The maximum strain in the tension
reinforcement in the section at Q.22 A reinforced concrete beam is
failure when designed for the limit subjected to the following bending
state of collapse should be moments:
 fy  Dead load – 20 kN-m
a) >  +0.002  Live load - 30 kN-m
 1.15E s  Seismic load – 10 kN-m
The design bending moment for
limit state of collapse is

© Copyright Reserved by Gateflix.in No part of this material should be copied or reproduced without permission
a) 60 kN-m b) 75 kN-m a) 0.75 times the area of steel
c) 72 kN-m d) 80 kN-m provided at the mid span in the
same direction
Q.23 The maximum depth of neutral axis b) 0.375 times the area of steel
for a beam width `d’ as the effective provided at the mid span in the
depth, in limit state method of same direction
design for Fe 415 steel is c) 0.375 times the area of steel
a) 0.46d b) 0.48d provided in the shorter span
c) 0.50d d) 0.53d d) nil

Q.24 The main reinforcement of a RC slab Q.29 A reinforced concrete member is


consists of 10mm bars at 10 cm subjected to combined action of
spacing. If it is desired to replace 10 compressive axial force and bending
mm bars by 12 mm bars, then the moment. If ε c is the least
spacing of 12 mm bars should be compressive strain in the member, f
a)12 cm b)14 cm
y , the yield stress of steel and, E S ,
c)14.40 cm d)16 cm
the modulus of elasticity of steel, the
Q.25 In case of 2-way slab, the limiting maximum permissible compressive
deflection of the slab is strain in concrete member will be
a) Primarily a function of the long a) 0.002
span b) 0.002 + f y / (1.15 E S )
b) Primarily a function of the short
span c) 0.0035 – 0.75 ε c
c) Independent of long or short d) 0.0035
spans
d) Dependent on both long and Q.30 In doubly reinforced rectangular
short spans beam, the allowable stress in
compression steel is
Q.26 When shear reinforcement is not a) Equal to the permissible stress in
provided in flat slab, the calculated tension in steel
shear stress at the critical section b) More than permissible stress in
shall not exceed K sτ c , where K s is tension in steel
a) (1 + β C ) b) (0.6 + β C ) c) Less than permissible stress in
tension in steel
c) (0.5 + β C ) d) (1.5 + β C )
d) Not related to the permissible
Q.27 A reinforced concrete slab is 75 mm concrete compression stress
thick. The maximum size of
reinforcement bar that can be used Q.31 The splicing of reinforcement bars
is of in RCC beam can be done at a
a)12 mm diameter section where
b)10 mm diameter a) Bending moment is zero
c)8 mm diameter b) Bending moment is less than the
d)6 mm diameter maximum bending moment
c) Bending moment is less than the
Q.28 In the design of two-way slab one half of the maximum
restrained at all edges, torsional bending moment
reinforcement required is d) Shear force is zero

© Copyright Reserved by Gateflix.in No part of this material should be copied or reproduced without permission
Q.32 The side face reinforcement, if d) 30% of total shear
required, in a T-beam will be reinforcement

a) 0.1% of the web area Q.38 If the nominal shear stress ( τ v ) at a


b) 0.15% of the web area section does not exceed the
c) 0.2% to 0.3% of the web area
permissible shear stress ( τ c )
depending upon the breadth of
the web a) Minimum shear reinforcement is
d) Half the longitudinal reinforcement still provided
b) Shear reinforcement is provided
Q.33 What shall be the maximum area of to resist the nominal shear stress
reinforcement (i) in compression c) No shear reinforcement is
and (ii) in tension to be provided in provided to resist the nominal
an RC beam, respectively, as per IS: shear stress
456? d) Shear reinforcement is provided
a) 0.08% and 2% for the difference of the two
b) 2% and 4%
c) 4% and 2% Q.39 Abeam of rectangular cross-section
d) 4% and 4% (b × d) is subjected to a torque.
What is the maximum torsional
Q.34 In the reinforced concrete slab, the stress induced in the beam (b < d
spacing between main and α is a constant)?
reinforcement should not exceed a) T2 b) T 2
a) Three times its effective depth αb d α bd
b) Four times its effective depth c) T d) T
c) Five times its effective depth α bd bd
d) Six times its effective depth
Q.40 Minimum shear reinforcement in
Q.35 The maximum percentage of beams is provided in the form of
moment redistribution allowed in stirrups
RCC beams is a) To resist extra shear force due to
a) 10% b) 20% live load
c) 30% d) 40% b) To resist the effect of shrinkage
of concrete
Q.36 Ratio of permissible shear stress in c) To resist principal tension
limit state method of design and d) To resist shear cracks at the
working stress method of design is bottom of beam
a) 25 : 16 b) 5 : 4
c) 16 : 25 d) 4 : 5 Q.41 The development length in
compression for a 20 mm diameter
Q.37 The contribution of bent up bars deformed bar of grade Fe-415
towards shear resistance shall not embedded in concrete of
be more than grade M-25, whose design bond
a) 50% of total shear stress is 1.40 N/mm 2 is
reinforcement a) 20 × 0.87 × 415 b) 20 × 0.87 × 415
b) 40% of total shear 4 ×1.40 4 × 1.25 × 1.40
c) 20 × 0.87 × 415 d) 20 × 0.87 × 415
reinforcement
c) 55% of total shear 4 ×1.6 ×1.40 4 × 1.25 ×1.6 ×1.40
reinforcement

© Copyright Reserved by Gateflix.in No part of this material should be copied or reproduced without permission
Q.42 Consider the following statements: i) High grade concrete is used
1. Development length includes ii) Mechanical anchorages are
anchorage value of hooks in tension employed
reinforcement. iii) Smaller diameter bars are used
2. The development length of each iv) Deformed bars are used
bar of bundled bars shall be that for Which of these statements are
the individual correct?
bar, increased by 33% for three bars a) (i), (ii) and (iii)
in contact. b) (i), (ii) and (iv)
3. Deformed bars may be used c) (i), (iii) and (iv)
without end anchorage provided d) (i), (ii), (iii) and (iv)
development length
Requirement is satisfied. Q.46 The effective length of a column in a
Which of these statements is/are reinforced concrete building frame,
correct? as per IS: 456-2000, is independent
a) Only 2 of the
b) both 1 and 3 a) Frame type i.e., braced (no sway)
c) Both 1 and 2 or un-braced (with sway)
d) 1, 2 and 3 b) span of the beam
c) Height of the column
Q.43 An RC structural member d) loads acting on the frame
rectangular in cross section of width
b and depth d is subjected to a Q.47 The reduction coefficient of a
combined action of bending moment reinforced concrete column with an
M and torsional moment. The effective length of 4.8 m and size of
longitudinal reinforcement shall be 250 mm × 300 mm is
designed for a moment M e given by a) 0.80 b) 0.85
c) 0.90 d) 0.95
a) M e = M + T (1 + D / b)
1.7
Q.48) The ratio of the maximum and
b) M e = M + T (1 − D / b)
1.7 minimum percentage of longitudinal
T (1 + D / b) reinforcement that can be provided
c) M e =
1.7 in a column is
a) 6 b) 7.5
d) M e = T (1 − b / D)
1.7 c) 5 d) 8

Q.44 Which one of the following is the Q.49 In the pedestal, the factor by which
correct expression to estimate the the effective length should not
development length of deformed exceed the least lateral dimension is
reinforcing bar as per IS code in a) 2 b) 3
limit state design? c) 4 d) 5
φσ s φσ s
a) b) Q.50 A reinforced concrete beam of 10 m
4.5τ 5τ
bd bd
effective spam and 1 m effective
φσ s φσ depth is supported on 500 mm ×
c) d) s
6.4τ bd
8τ bd
500 mm columns. If the total
uniformly distributed load on the
Q.45 Bond strength can be enhanced beam is 10 kN/m, the design shear
when force for the beam is

© Copyright Reserved by Gateflix.in No part of this material should be copied or reproduced without permission
a) 50 kN b) 47.5 kN 1.In a helical reinforced concrete
c) 37.5 kN d) 43 kN column, the concrete core is
subjected to tri-axial
Q.51 Which of the following are the state of stress.
additional moments considered for 2.Helically reinforced column are
design of slender compression very much suitable for earthquake
member in lieu of deflection in x and resistant structures.
y direction Which of these statements is/are
correct?
Pu l 2 ex Pu l 2 ey a) 1 only b) 2 only
a) And c) Both 1 and 2 d)None of these
2000 D 2000 D
Pu lex Pu ley
b) And Q.56 According to IS: 456-2000, the
2000 D 2000 D column or the strut is the member
2
Pu l ex Pu l 2 ey whose effective length is greater
c) And
2000 D 2000b than
2
Pu l 2 ey a) The least lateral dimension
Pl
d) u ex And b) 2 times the least lateral
200 D 200 D dimension
Where P u is axial load; l ex and l ey c) 3 times the least lateral
are effective length in respective dimension
directions; D is depth of section d) 4 times the least lateral
perpendicular to major axis; b is dimension
width of the member
Q.57 According to IS 456, the slenderness
Q.52 What is the minimum number of ratio for a short column should not
longitudinal bars provided in exceed
reinforced concrete column of a) 12 b) 18
circular cross-section? c) 24 d)None of these
a) 4 b) 5
c) 6 d) 8 Q.58) Consider the following statements:
Bars that extend into a simple
Q.53 An axially loaded column is of 300 support must be able to develop
mm × 300 mm size. Effective length their full strength at a designated
of column is 3 m. What is the point L so that their moment
minimum eccentricity of the axial capacity is more than the bending
load for the column? moment at that point. The clauses of
a) 0 b) 10 mm the code require that ( σ s = 0.85 σ sy )
c) 16 mm d) 20 mm
1 L d ≤ 1.3M 1 + L0
Q.54 The load carrying capacity of a V
column designed by working stress φσ s 1.3M 1
2 ≤ + L0
method is 500 kN. The ultimate 4τ bd
V
collapse load of the column is
3 ∅≤ 4τ bd  1.3M 1 + L0 
a) 500kN b) 662.5kN σs  V 
c) 750kN d) 1100kN Which of these statements is/are
Q.55 Consider the following statements: correct?
a) 1 and 2 b) 2 and 3
c) 1 and 3 d) 1, 2 and 3

© Copyright Reserved by Gateflix.in No part of this material should be copied or reproduced without permission
Q.64 If the foundations of all the columns
Q.59 In a combined footing for two of a structure are designed on the
columns carrying unequal loads, the total live and dead load basis, then
maximum hogging moment occurs at a) There will be no settlement of
a) inside face of the heavier column columns
b) a section equidistant from both b) There will be no differential
the column settlement
c) a section having maximum shear c) The settlement of exterior
force columns will be more than
d) a section heaving zero shear force interior columns
d) The settlement of interior
Q.60 A square column section of size columns will be more than
350mm × 350mm is reinforced with exterior columns
four bars of 25mm diameter and Q.65 In the case of isolated square
four bars of 16mm diameter. Then concrete footing, match the
the transverse steel should be locations at which the stress
a) 5 mm dia at 240 mm c/c resultants are to be checked, where
b) 6 mm dia at 250 mm c/c d is effective depth of footing and
c) 8 mm dia at 250 mm c/c select the correct answer using the
d) 8mm dia at 350 mm c/c codes given below the lists:
Stress Resultant Location
Q.61 A trapezoidal combined footing for A. Bending moment 1.At face of
two axially loaded columns is Column
provided when B. One way shear 2.At d/2
i) Width of the footing near the from face of column
heavier column is restricted. C. Punching shear 3. At d from
ii) Length of the footing is face of column
restricted. Codes:
iii) Projections of the footing beyond A B C
the heavier column are a) 1 2 3
restricted. b) 3 1 2
Select the correct answer using the c) 1 1 3
codes given below: d) 1 3 2
a) 1 and 2 b) 1 and 3
c) 2 and 3 d) 1, 2 and 3 Q.66 IS 1343-1980 limits the minimum
characteristic strength of
Q.62 In RC footing on soil, the thickness prestressed concrete for post-
at the edge should not be less than tensioned work and pretension
a) 10 cm b) 15 cm work as
c) 20 cm d) 25 cm a) 25 MPa, 30 MPa respectively
b) 25 MPa, 35 MPa respectively
Q.63 A strap footing is a special type of c) 30 MPa, 35 MPa respectively
a) Strip footing d) 30 MPa, 40 MPa respectively
b) raft footing Q.67 A simply supported post-tensioned
c) Combined footing prestressed concrete beam of span L
d) spread footing is prestressed by a straight tendon
at a uniform eccentricity `e’ below
the centroidal axis. If he magnitude
of prestressing force is P and

© Copyright Reserved by Gateflix.in No part of this material should be copied or reproduced without permission
flexural rigidity of beam is EI, the Q.70 Match List-1 (post-tensioning
maximum central deflection of the system) with List-2 (type of
beam is anchorage) and select the correct
PeL2 answer using the codes given bellow
a) (Downwards) the lists:
8 EI
List-1
PeL2
b) (Upwards) A Freyssinet
48 EI B Giford-udall
PeL3 C Lee-McCall
c) (Upwards)
8 EI D Magnel Blaton
PeL2 List-2
d) (Upwards) 1. Single bars
8 EI
2. Wires evenly spaced by
Q.68 Concordant cable profile is perforated spacers
a) A cable profile that produces no 3. Horizontal rows of four wired
support reactions due to spaced by metal grills
prestressing 4. Wires spaced by helical wire
b) A cable profile which is parabolic core in annular spacer
in nature Code:
c) A cable profile which produces A B C D
no bending moment at the a) 4 1 2 3
supports of a beam b) 3 2 1 4
d) A cable profile laid corresponding c) 4 2 1 3
to axial stress diagram d) 3 1 2 4

Q.69 Match List-1 (post-tensioning Q.71 An ordinary mild steel bar has been
system) with List-2( type of prestressed to a working stress of
anchorage) and select the correct 200MPa. Young’s modulus of steel is
answer using the codes given bellow 200 GPa. Permanent negative strain
the lists: due to shrinkage and creep is
List-1 0.0008. How much is the effective
A. Freyssinet stress left in steel?
B. Giford-udall a) 184 MPa b) 160 MPa
C. Lee-McCall c) 40 MPa d) 16 MPa
D. Magnel Blaton
List-2 Q.72 What is the allowable upward
1. Flat steel wedges in sandwich deflection in a prestress concrete
plates member under serviceability limit
2. High strength nuts state condition?
3. Split conical wedges a) Span/250 b) Span/300
4. Conical serrated concrete wedges c) Span/350 d) Span/500
Code:
A B C D Q.73 At the time of initial tensioning, the
a) 2 1 4 3 maximum tensile stress
b) 4 3 2 1 immediately behind the anchorage
c) 2 3 4 1 should not exceed which one of the
d) 4 1 2 3 following?
a) 0.50 × ultimate tensile stress
b) 0.60 × ultimate tensile stress

© Copyright Reserved by Gateflix.in No part of this material should be copied or reproduced without permission
c) 0.70 × ultimate tensile stress
d) 0.80 × ultimate tensile stress

Q.74 If a simply supported concrete beam


, prestressed with a force of 2500
kN, is designed by load balancing
concept for an effective span of 10 m a) 2.5 Pg/L b) 3.0 Pg/L
and to carry a total load of 40 kN/m, c) 3.5 Pg/L d) 4.0 Pg/L
the central dip of the cable profile Q.78 If percentage gain in stress due to
should be UDL in tendon is zero, then
a) 100 mm b) 200 mm eccentricity will be
c) 300 mm d) 400 mm 2 2
a) Wl ES b) Wl 1
12 p EC 12 p EC
Q.75 A prestressed concrete beam has a 2
c) Wl EC
2

cross-section with the following d) Wl


12 p ES 12 p
properties:
Where P is prestressing force in
A = 46400 mm 2 , l = 75.8 × 10 7 mm
tendon, W is UDL, E c - modulus of
4
, Y bottom = 244 mm , Y top = 156 mm
elasticity of concrete and E s -
It is subjected to a prestressing force
at an eccentricity `e’ so as to have a modulus of elasticity of steel.
zero stress at the top fibre. The Q.79 A concentrated live load at the
value of `e’ is given by centre of span of prestressed
a) 66.66 mm b) 66.95 mm concrete beam can be counter
c) 104.72 mm d) 133.33 mm balanced by selecting
a) Straight cable profile
Q.76 In prestressed concrete b) Parabolic cable profile
a) Forces of tension and c) Linearly varying profile with
compression change but lever zero eccentricity at centre of span
arm remains unchanged d) concentric cable profile
b) Forces of tension and
compressions remain unchanged Q.80 Deflection of prestressed concrete
but lever arm changes with the beam is excessive in the
moment a) Pre-cracking stage
c) Both forces of tension and b) Plastic stage
compression as well as lever arm c) Post-cracking stage
change d) none of the above
d) both forces of tension and
Q.81 Combination of partial safety factors
compression as well as lever arm
for loads under limit state of
remain unchanged
collapse and limit state of
serviceability will be
Q.77 If `P’ is the prestressing force
a) 1.5 (DL + LL) or 1.5 (DL + WL) or
applied at a maximum eccentricity
1.2 (DL + LL + WL) and DL + 0.8
`g’ at mid-span, figure, to balance the
(LL + WL)
concentrated load `W’, the balancing
b) 1.5(DL+LL) and Dl+0.8(LL + WL)
load will be
c) 1.5 (DL + LL) or 1.5 (DL + WL) or
1.2 (DL + LL + WL) and 1.0 (DL +
LL) or 1.0 (Dl + WL) or 0.8 (LL +
WL)

© Copyright Reserved by Gateflix.in No part of this material should be copied or reproduced without permission
d) 1.2 (DL + LL + WL) and 1.0 (DL + a) M 35 b) M 30
LL) or 1.0 (Dl + WL) or 0.8 (LL + c) M 25 d) M 20
WL)
Q.86 On which one of the following
Q.82 If the depth of neutral axis for a concepts is the basic principle of
singly reinforced balanced structural design based?
rectangular section is represented a) Weak column strong beam
by kd in working stress design, then b) Strong column and weak beam
the value of k for a balanced section, c) Equally strong column and beam
a) Depends on σ st only d) Partial weak column- beam
b) Depends on σ cbc only
c) Depends on both σ st and σ cbc Q.87 Match List-1 with List-2 and select
the correct answer using codes
d) is independent of both σ st and given below the lists:
σ cbc
Q.83 The expression for modular ratio m List-1
280 A. IS-875
= , where σ cbc is the B. IS-1343
3σ cbc
C. IS-1893
permissible compressive stress due D. IS-3370
to bending in concrete in N/mm 2 , List-2
a) Fully takes into account the long- 1. Earthquake resistant design
term effect such as creep 2. Loads
b) Partially takes into account the 3. Liquid structure
long-term effect such as creep 4. Prestressed Concrete
c) Does not take into account the A B C D
long-term effect such as creep a) 3 1 4 2
d) is the same as the modular ratio b) 2 1 4 3
based on the value of modulus of c) 3 4 1 2
elasticity of structural concrete E c d) 2 4 1 3

Q.84 Examine the following statements: Q.88) MatchList-1 with List-2and select
1. Factor of safety for steel should the correct answer using codes
be based on its yield stress. given below the lists:
2. Factor of safety for steel should List-1
be based on its ultimate stress. A. Moment and shear coefficients
3. Factor of safety for concrete B. Fire resistance
should be based on its yield C. Sliding
stress. D. Span to depth ratio of beam
4. Factor of safety for concrete List-2
should be based on its ultimate 1. Durability
stress. 2. Stability
The correct statements are: 3. Analysis of structure
a) 1 and 3 b) 1 and 4 4. Deflection limits
c) 2 and 3 d) 2 and 4 A B C D
a) 4 2 1 3
Q.85 What should be the minimum grade b) 3 2 1 4
of reinforced concrete in and around c) 4 1 2 3
sea coast construction? d) 3 1 2 4

© Copyright Reserved by Gateflix.in No part of this material should be copied or reproduced without permission
Q.94 Design strength of concrete for limit
Q.89 Consider the following statements: state of collapse is
1. Maximum strain in concrete at the a) f ck b) 0.67 f ck /γ
outermost compression fibre is
c) 0.67 f ck d) γ f ck
taken to be 0.0035 in bending.
2. The maximum compressive strain Where,
in concrete in axial compression is f ck = Characteristic compressive
taken as 0.002. strength of concrete
Keeping the provision of IS: 456- γ = Partial factor of safety
2000 on limit state design in mind,
which of the following is true? Q.95 The diagonal tension failure in a
a) Statement 1 is true but 2 is false concrete beam occurs due to
b) Statement 1 is false but 2 is true a) Large shear force and less
c) Both statements 1 and 2 are true bending moment
d) Both statements 1 and 2 are false b) Large bending moment and less
Q.90 To determine the modulus of shear force
rupture, the size of test specimen c) Equal shear force and bending
used is moment
a) 150 × 150 × 500 mm d) None of these
b) 100 × 100 × 700 mm
c) 150 × 150 × 700 mm Q.96 If the cross sectional areas of the
d) 100 × 100 × 500 mm three basic structural elements viz.
beam slab and column are related to
Q.91 For the design of retaining walls, the the amount of steel reinforcement
minimum factor of safety against then which of the following
overturning is taken as statements is correct
a) 2 b) 1.4 1. Percentage steel is usually
c) 1.5 d) 3.0 maximum in a column
Q.92 The centroid of compressive force, 2. Percentage steel is least in a slab
from the extreme compression fibre, a) only 1 b) only 2
in limit state design lies at a distance c) Both 1 and 2 d) neither 1 nor 2
of
a) 0.367 X U b) 0.416 X U Q.97 Consider the following statements in
relation to stress-strain curve for
c) 0.446 X U d) 0.573 X U concrete and steel:
Q.93 Maximum strain at the level of 1. Factor of safety (FOS)is applied
compression steel for a rectangular for whole curve for steel
section having effective cover to 2. Factor of safety (FOS) is applied
compression steel as `d’ and neutral only for a part of steel for which
axis depth from compression face as hook’s law is not applicable.
3. Factor of safety (FOS) is applied
X U is
for whole curve for concrete.
 d   d  4. Factor of safety (FOS) is applied
a)0.0035 1 −  b)0.002 1 − 
 XU   XU  only for a part of concrete for which
hook’s law is not applicable.
 X   X  Which of these statements are
c)0.0035 1 − U  d)0.002 1 − U 
 d   d  correct?
a) 1 and 3 b) 1 and 4
c) 2 and 3 d) 2 and 4

© Copyright Reserved by Gateflix.in No part of this material should be copied or reproduced without permission
Q.98 Compressive strength of concrete is
taken as 0.67 f ck and not f ck because
of
1. Size factor which is constant for
a cube of size greater than 450
mm.
2. Size factor which is constant for
a cube of size greater than 550
mm.
3. End friction zone which acts
throughout the length of cube.
4. End friction zone which acts
throughout the length of
cylinder.
Which of these statements are a) 4.5 kN/m 2 b) 5.0 kN/m 2
correct? c) 5.83 kN/m 2 d) 6.8 kN/m 2
a) 1 and 3 b) 1 and 4
c) 2 and 3 d) 2 and 4 Q.100 If the creep coefficient for concrete
at 7 days is K1 and at 28 days is K 2
Q.99 The rise and tread of staircase are then
150 mm and 250 mm and weight of a) K1 > K 2 b) K1 < K 2
slab on slope is 5 kN/m 2 . The dead
c) K1 = K 2 d) K1 ≤ K 2
weight of horizontal area is

ANSWER KEY:
1 2 3 4 5 6 7 8 9 10 11 12 13 14
(a) (a) (d) (a) (b) (a) (b) (a) (c) (b) (d) (d) (d) (b)
15 16 17 18 19 20 21 22 23 24 25 26 27 28
(b) (a) (d) (b) (a) (b) (b) (b) (b) (c) (b) (c) (c) (a)
29 30 31 32 33 34 35 36 37 38 39 40 41 42
(c) (c) (c) (a) (d) (a) (c) (a) (a) (a) (a) (c) (d) (b)
43 44 45 46 47 48 49 50 51 52 53 54 55 56
(a) (c) (d) (d) (b) (b) (b) (c) (c) (c) (d) (c) (c) (c)
57 58 59 60 61 62 63 64 65 66 67 68 69 70
(a) (d) (d) (c) (c) (b) (c) (c) (d) (d) (d) (a) (b) (c)
71 72 73 74 75 76 77 78 79 80 81 82 83 84
(c) (b) (d) (b) (c) (b) (d) (d) (c) (c) (c) (a) (b) (b)
85 86 87 88 89 90 91 92 93 94 95 96 97 98
(b) (b) (d) (d) (c) (c) (b) (b) (a) (b) (a) (c) (c) (a)
99 100
(c) (a)

© Copyright Reserved by Gateflix.in No part of this material should be copied or reproduced without permission
EXPLANATIONS

For simple span deep beam, l/D ≤2.0


Q. 1 (a) For Continuous span deep beam, l/D
≤.5
Q. 2 (a) Where, l = Effective span taken as
centre to centre distance between
Q. 3 (d) the support of 1.15 time the
See clause 23.1.2 of IS 456 : 2000 clear span, whichever is smaller.
D = Overall depth
Q. 4 (a) D = 30 cm
L xu= 12 cm
<2.5 for continuous beam
D c = 50 kg/ cm 2
So L < 2.5×D C X
⇒ =U
So Maximum effective length t d − XU
= 2.5× (500+50)=1375 mm m
18 × 50
Q. 5 (b)
⇒ t= × (30-12)
12
V= Vu + 1.6
Tu 18 × 50
e ⇒
= t × (30 − 12)
b 12
Here Vu = 0 = 1350kg/ cm 2
50 ×103
Ve =
1.6 × =
229kN Q. 10 (b)
350
Depth of the balanced neutral
Q. 6 (a) section using working stress method
is given by
Q. 7 (b) 280
In case of two way slab, for mild ⇒ X U min = d
280 + 3σSR
steel, (span/Overall Depth) = 40
280
D =(3⨉100/40)=7.5cm = = d 0.4d
280 + 3 + 140
Q. 8 (a)
For a balanced section Q. 11 (d)
0.39f ck * b * X = 0.87f y A st The limiting value of depth of
neutral axis depending on the
0.36f ck * b * X different grades of steel is as
⇒ A st =
0.87f y follows:
As f y increases A st decreases. fy X U min / d
250 0.53
Q. 9 (c) 415 0.48
The limiting value l/D for simply 500 0.46
supported and continuous span
deep beams are as follows: Q. 12 (d)
Basic value of span to effective
depth ratios for spans up to 10 m to
satisfy vertical deflection limits
are for

© Copyright Reserved by Gateflix.in No part of this material should be copied or reproduced without permission
Cantilever beam 7 reinforcement measured along the
Simply supported 20 axis of member shall not exceed
Continuous 26 0.75 d for vertical stirrups and
1d’ for inclined stirrups at 45 ° ,
Q. 13 (d) where 1d’ is the effective depth
For T-beams effective width of of the section under consideration.
compression flange In n case shall the spacing exceed
L 300 mm.
⇒ b f = 0 + B + 6t
6 ∴ Maximum spacing = 0.75 d
For L- beam = 0.75 × 300 = 225 mm
bf = (Lo/12) + bw +3 Df
Q. 18 (b)
Q. 14 (b) Moment of resistance for a balanced
Total compressive force without section is given by
considering the partial safety factor M U ,lim = 0.36 f ck × X U ,lim × (1- 0.42 ×
of material
⇒ 0.36 f ck ∗ b ∗ X U ×1.5 X U ,lim
) × bd 2
= 0.54 f ck ∗ b ∗ X U d
But for FE 415
Q. 15 (b) X U ,lim = 0.48 d
As per IS: 456-2000 modular ratio is
M U ,lim = 0.36 f ck × 0.48 × (1- 0.42 ×
given by
2
280 0.48) × bd
M=
3σ cbc ⇒ M U ,lim = 0.13796 f ck × bd 2
For M20 concrete On comparison, we get
σ cbc =7N/mm 2 K = 0.13796 f ck
280 = 0.13796 × 20 = 2.76
∴ m= = 13.3
3× 7

Q. 16 (a) Q. 19 (a)
As per the clause 22.2 of IS: 456- To ensure ductility the maximum
2000, for simply supported beam strain in tension reinforcement in
and slab, the effective span of a the section at failure shall not be
member that is not built integrally fy
less than + 0.002
with its supports shall be taken as 1.15 ES
clear span plus the effective depth
of slab or beam or centre of Q. 20 (b)
supports, whichever is less. For under- reinforced beam
∴ Effective span = 5 × 1000 + 40
i) U < U ,lim or C
X X X
= 5400 mm
And effective span ∈
ii) sc Steel reaches ultimate
300 300 stress before concrete reaching
= 5 × 1000 + + = 5300 mm
2 2 the ultimate stress
iii) u < u ,lim
M M
Q. 17 (d)
As per clause 26.5.1.5 of IS: 456-
2000, the maximum spacing of shear

© Copyright Reserved by Gateflix.in No part of this material should be copied or reproduced without permission
iv) Lever arm = d-0.42 X U will X U ,max = 0.48d and
be more in the case of under- for f y = 250 N/mm
2

reinforced section
X U ,max = 0.53d
Q. 21 (b)
Q. 24 (c)
The horizontal distance between
parallel main reinforcement bars
shall not be more than three times
he effective depth of solid slab or
300 mm whichever is smaller. The
total reinforcement in the slab
∈sc ∈cu should remain same. By replacing
= 10 mm bars by 12 mm bars, the
Xu − d′ Xu
spacing will increase as
d′
∴ ∈sc = 0.0035(1- )  d2 
2
2
Xu  12 
  × S =   × 10
 d1   10 
1

Q. 22 (b) = 14.4 cm
The various load combination are as
follows: Q. 25 (b)
i) For the dead load and live load The strip of two way slab may be
the ultimate bending moment is checked against shorter span to
given by, effective depth ratios.
Span to effective depth ratio
M u = 1.5 (DL + LL)
= 1.5 × (20+30) = 75 kN-m
ii) For dead load and earthquake
(seismic) load the ultimate
bending moment is given by,
M u = 1.5 (DL + EL) Q. 26 (c)
= 1.5 × (20 + 10) = 45 kN-m k S = 0.5 + 𝛃𝛃 C > 1
iii) For dead load, live load and 𝛃𝛃 C = Shorter side/Longer side
earthquake (seismic) load the
ultimate bending moment is Q. 27 (c)
given by The diameter of the bars shall not
M u = 1.2 (DL + EL + LL) exceed one eighth of the total
= 1.2 (20 + 30 + 10) thickness of the slab.
= 72 kN - m 75
∴ Maximum size =
8
Q. 23 (b) = 9.375 mm
0.0035 So the diameter will be 8 mm.
X U ,max = d
0.87 f y
0.0055 + Q. 28 (a)
Es
For restrained slab, the area of
For f y = 415 N/mm 2 reinforcement in each of the four
And E s = 2 × 10 5 N/mm 2 corner layers shall be three-quarters

© Copyright Reserved by Gateflix.in No part of this material should be copied or reproduced without permission
of the area required for the more than half the bars shall be
maximum mid span moment in the spliced at a section.
slab simply supported on both edges
meeting at that corner. If the corner Q. 32 (a)
contained by edges over only one of For beams exceeding overall depth
which the slab is continuous, of 750 mm side face reinforcement
torsion reinforcement equal to is provided. Such reinforcement
0.375 times the area reinforcement shall be not less than 0.1 percent of
provided at the mid-span in the the web area and shall be
same direction shall be provided. If distributed equally on two faces at a
both edges are continuous, no spacing not exceeding 300 mm or
torsion reinforcement shall be web thickness whichever is less.
provided.
Q. 33 (d)
Q. 29 (c) As per the clause 26.5.1.1 of IS: 456-
Maximum permissible compressive 2000, the maximum area of tension
strain in concrete under axial reinforcement in beams shall
compression is taken as 0.002. The not exceed 0.04 bD. As per clause
maximum compressive strain at the 26.5.1.2 of IS: 456-2000, the
highly compressed extreme fibre in maximum area of compression
concrete subjected to axial reinforcement in beams shall
compression and bending and when not exceed 0.04 bD.
there is no tension on the section
shall be 0.0035 minus 0.75 times the Q. 34 (a)
strain at the least compressed Maximum diameter of reinforcing
extreme fibre. bars in slabs is limited to one-eight
of the total thickness of slab and the
Q. 30 (c) maximum spacing of main bars is
The stress in the compression steel limited to 3d or 300 mm (whichever
depends on the strain, ϵ sc , the level is less).
of the compression steel.The strain
at the level of compression steel Q. 35 (b)
under loading does not reach the As per clause 37.1.1 of IS: 456-2000
yield strain value of steel. the ultimate moment of resistance at
Therefore, allowable stress is less any section should not be less
than the permissible stress in than 70% of the factored moment at
tension in steel. The maximum that section as obtained from the
permissible compressive stress in elastic moment envelope
steel is 130MPa and 190MPa for Fe (considering all loading
250, Fe 415 and Fe 500 grades combinations).
respectively.
Q. 36 (a)
Q. 31 (c) Permissible shear stress,
It is recommended that splices in In limit state method of design,
flexural members should not be τ C = 0.25 f ck
provided at sections where the In working state method of design
bending moment is more than 50% τ C = 0.16 f ck
of the moment of resistance; and not

© Copyright Reserved by Gateflix.in No part of this material should be copied or reproduced without permission
25 into which the section may be
∴ Ratio =
16 divided.
Torsional shear stress for
Q. 37 (a) rectangular section
Total shear strength = Shear  bi 3 di 
resistance of effective concrete area Ti′ = T  
 ∑ bi di 
3
as a function of longitudinal bars +
shear resistance of vertical shear
stirrups + shear resistance of Q. 40 (c)
inclined shear stirrups.Tests have The shear, at which the inclined
shown that inclined bars alone do crack in beam without shear
not provide a satisfactory solution reinforcement is formed first, is
and their contribution is limited to taken as the shear strength of
50% of net shear strength after concrete as the difference between
deducting the contribution of the loads corresponding to the
concrete. The remaining shear first crack and the ultimate failure
resistance is provided by vertical is very less. Formations of such
stirrups. crack occur when the principal
tensile stress reaches the
Q. 38 (a) tensile strength of concrete. At the
If the calculated shear stress ( τ v ) is mid-span of a simply supported
i) Less than allowable shear stress ( beam subjected to uniformly
distributed load, where shear is
τ c ) but more than 0.5 τ c , the
small and bending stress is large,
minimum shear reinforcement in the direction of principal tensile
the form of stirrups shall be stress is flat and is nearly equal to
provided such that the flexural tensile stress. This will
ASV 0.4 cause flexural cracks nearly vertical
≥ to the axis of the beam.
bSV 0.87 f y
These are initiated even, when 0.5 τ c
ii) More than τc , shear
< τ v < τ c .Thus minimum
reinforcement shall be provided in
the form of vertical stirrups or bent reinforcement is needed to prevent
up bars with stirrups or inclined flexural crack due to principal
stirrups to resist tension.
Vus = Vu - τ c bd. Here τ v < τ c ,max
Q. 41 (d)
iii) If τ v > τ c ,max redesign the section. For bars in compression the value of
bond stress is increased by 25%
Q. 39 (a) For deformed bars value of bond
Torsion constant of a rectangular stress is increased by 60%
section of width b and depth d (b <
d) may be expressed as J = b3d Q. 42 (b)
For T, L and I sections torsion In case of bundled bars in contact
constant the development length is increased
1 than that for individual bar by
J = ∑ bi 3 di 1 10% for two bars in contact
3
Where b i and d i are the dimensions 2 20% for three bars in contact
of each of the component rectangles

© Copyright Reserved by Gateflix.in No part of this material should be copied or reproduced without permission
3 33% for four bars in contact iii) Type of frame (sway or non
sway)
Q. 43 (a)
The longitudinal reinforcement shall Q. 47 (b)
be designed to resist an equivalent In the working stress method the
bending moment design of a long column is made by
Me = M + Mt considering reduced stresses
obtained by multiplying the
 1+ d / B 
Where, M t = T   permissible stresses by a coefficient,
 1.7  leff l
 1+ D / b  C r = 1.25 - or 1.25 - eff
∴ Me = M + T   48 D 48 B
 1.7  Whichever is smaller
Equivalent shear 4800
C r = 1.25 - or
48 × 250
V e = V + 1.6 T/b 1.25 - 4800 = 0.85 or 0.92
48 × 300
Q. 44 (c)
As per clause 26.2.1 of IS: 456-2000, Q. 48 (b)
the development length L d is given Maximum reinforcement = 6%
Minimum reinforcement = 0.8%
by
6
φσ s Ratio = = 7.5
0.8
Ld=

Q. 49 (b)
bd

Where,∅ = nominal diameter of bar


Very short columns with effective
σ s = stress in the bars at the section
length less than three least lateral
considered at design load dimensions are called pedestal
τ bd = design bond stress for plain columns.
bars
The value of τ bd should be increased Q. 50 (c)
The shear force should be calculated
by 60% for deformed bars.
at critical section i.e. 1m away from
φσ s the face of the column. Effective
∴ Ld=
 60  span of the beam is to be taken as
4 τ bd + τ bd 
 100  the clear span plus effective depth of
beam or centre to centre spacing of
φσ s supports whichever is less. The
Ld=
6.4τ bd
location for shear calculation will be
Q. 45 (d)
X = 0.25+ 1= 1.25
∴ design shear force
Q. 46 (d)
= 10 ×10 − 10 ×1.25 = 37.5kN
Effective length of a column depends 2
upon:
i)Flexural stiffness (E/L) of beams Q. 51 (c)
joining at a point. 2
ii) Flexural stiffness (E/L) of Pu D  lex  Pu l 2 ex
columns joining a point.
M ax =   =
2000  D  2000 D

© Copyright Reserved by Gateflix.in No part of this material should be copied or reproduced without permission
2 According to the IS (CI. 25.1.2), a
Pu b  ley  Pu l 2 ey
M ay =   = compression member may be
2000  b  2000b classified as a `short column’ if its
slenderness ratios with respect to
Q. 52 (c) the `major principal axis’ (l ex /D x ) as
As per IS: 456- 2000 clause well as the `minor principal axis (l ey
26.5.3.1(a) the minimum number of
/D y ) are both less than 12.
longitudinal bars provided in a
column shall be four in rectangular
columns and six in circular Q. 58 (d)
columns.
Q. 59 (d)
Q. 53 (d) The combined footing resists the
Minimum eccentricity equals to load by bending in two directions.
unsupported length of column/ The footing in the longitudinal
500 plus lateral dimension/30, direction is subjected to a sagging
subjected to a minimum of 20 mm. moment in the cantilever portion
L B 3000 300 and under the columns; and hogging
+ = + = 16
500 30 500 30 moment in the central portion
So eccentricity = 20mm between the columns. Footing in the
transverse direction develops
Q. 54 (c) sagging moments. The maximum
The ultimate collapse load on the hogging moment occurs in the
column is given by central portion where shear force is
W u = W s × 1.5 = 500 × 1.5 zero.
= 750 kN
Q. 60 (c)
The diameter of transverse
Q. 55 (c)
reinforcement shall not be less than
Confinement of concrete by
one fourth of the diameters of the
providing transverse reinforcement
largest longitudinal bar and in
in the form of steel hoops and
no case less than 6 mm. So the
spirals increases the strength and
ductility of concrete. This reduces diameter of the bar 25 = 6.25
4
the tendency for internal cracking mm. Choose 8 mm diameter bar.
and volume increase prior to The pitch of the transverse
failure. reinforcement shall not be more
than the least of the following.
Q. 56 (c) i) The least lateral dimension of the
The code defines the column as a compression member i.e. 350 mm.
compression member, the effective ii) Sixteen times the smallest
length of which exceeds three diameter of the longitudinal
times the least lateral dimension. reinforcement bar to be tied i.e.
The term `pedestal’ is used to 16×6 = 256 mm
describe a vertical compression iii) 300 mm
member whose` effective length’ is So pitch will be 250 mm c/c.
less than three times its least lateral
dimension. Q. 61 (c)
Q. 57 (a)

© Copyright Reserved by Gateflix.in No part of this material should be copied or reproduced without permission
Combined footing supporting two there will be no reactions at the
columns may be used for column on supports due to prestressing action.
property lines or sewer line. In this
case load will be eccentric. Q. 69 (b)
In the freyssinet system anchorage
Q. 62 (b) consists of cylinder of good quality
According to the code (CI. 34.1.2) concrete and is provided with
restricts the minimum thickness at corrugations on the outside. It
the edge of the footing to 150 mm has a central conical hole and is
for footings in general (and to 300 provided with heavy top
mm in the case of pile caps). reinforcement.
In Gifford-Udall system, the wire are
Q. 63 (c) stressed and anchored one by one in
A strap footing consists of a spread a separate cylinder using small
footing of two columns connected wedding grip called Udall grips.
by a strap beam. This type of footing Each grip consists of two split cones.
is useful when the In Lee Mc Call system anchoring of
external column is very near to the bars is done by screwing special
property line so that its footing threaded nuts. The nuts bear
cannot be spread beyond the against a distribution plate provided
property line at the end of the beam.
In the Magnel Blaton system, the
Q. 64 (c) wires are anchored by wedging, two
The ratio of live load to dead load is at a time into sandwich plates.
greater in the case of interior The sandwich plates are provided
column as compared to that in with two wedge shaped grooves on
exterior column. its two faces. The wires are taken
into each groove and tightened.
Q. 65 (d) Then a steel wedge is driven
For footings on piles the critical between the tightened wires to
section for one way shear is d/2 anchor them against the plate.
from the face of the wall.
Q. 70 (c)
Q. 66 (d) In Freyssinet system, high tension
steel wires 5 mm to 8 mm diameter
Q. 67 (d) about 12 in number are arranged
Maximum central deflection, to form a group into a cable with
L2 spiral spring inside.
δ= In the Gifford Udall system the wires
8R
1 Pe are stressed and anchored one by
= one in a separate cylinder using
R EI
small wedging grips called Udall
PeL2 grips.
∴ δ= (Upwards)
8 EI In the Lee- Mc Call system, High
tensile alloy steel bars are used as
Q. 68 (a) the prestressing tends. In the
Concordant cable profile does not Magnel Blaton system cable of
produce secondary moments and rectangular section is provided. The
wires are arranged with four wires

© Copyright Reserved by Gateflix.in No part of this material should be copied or reproduced without permission
per layer. The geometric pattern of P Pe
the wires is maintained in the same =
A Z
form throughout the length of the P Pe
cable by providing grills or spacers ⇒ =
at regular intervals. A I / Ytop
75.8 ×107
Q. 71 (c) ⇒e = = 104.72 mm
156 × 46400
Loss of stress due to shrinkage and Q. 76 (b)
creep
= 200 × 10 3 × 0.0008 Q. 77 (d)
= 160 MPa
Effective stress left = 200 – 160 = Q. 78 (d)
40 MPa Net percentage gain due to load =
2eθ net Es
Q. 72 (b)
As per IS: 1343-1980 (codes of l
practice for prestressed concrete), if 2eθ net Es
finishes are to be applied to the If % gain is zero, then
l
prestressed concrete members, = 0 [e≠0, E s ≠ 0]
the total upward deflection should
not exceed span/300, unless ∴θ net = 0
uniformity of camber between Wl 3 Pel
adjacent units can be insured. ⇒ - =0
24 Ec l 2 Ec l
Q. 73 (d) Wl 2
⇒e =
As per clause 18.5.1 of IS: 1343- 12 P
1980, at the time of initial
tensioning, the maximum tensile Q. 79 (c)
stress immediately behind the
anchorages shall not exceed 80% of Q. 80 (c)
the ultimate tensile strength of the
wire or bar or strand. Q. 81 (c)

Q. 74 (b) Q. 82 (a)
Let the dip of the cable be h. The value of K is given by,
Upward pressure provided by the mc
K=
8Ph t + mc
parabolic cable = For a balanced section,
L2
In order this upward pressure may 280
m=
fully balance the external loading. 3σ cbc
8Ph c = σ cbc , t = σ st
=W
L2 280
× σ cbc
8 × 2500 × h 3σ cbc
⇒ = 40 ∴ K=
2
280
10 σ st + × σ cbc
⇒ h = 0.2 m = 200 mm 3σ cbc

Q. 75 (c)
For zero stress at top fibre

© Copyright Reserved by Gateflix.in No part of this material should be copied or reproduced without permission
280 Q. 87 (d)
= 3
280 Q. 88 (d)
σ st +
3
Q. 89 (c)
Q. 83 (b)
Q. 90 (c)
Q. 84 (a)
The modulus of rupture test
A beam collapses when stress is
employs a 150×150×700 mm plain
equal to its yield strength and stress
concrete beam. The beam is simply
in concrete reaches its ultimate
supported and subjected to third-
strength.
points loading until failure. Assuming
a linear stress distribution across
Q. 85 (b)
Exposure Minimum Minimum grade
the cross section, the theoretical
grade of of reinforced maximum tensile stress reached in
plain concrete the extreme fibre is termed by
concrete applying the flexure formula
i. Mild - M20 M
ii. Moderate M15 M25 f cr =
iii. Severe M20 M30 Z
iv. Very severe M20 M30 Modulus of rupture is given by
v. Extreme M25 M40 f cr = 0.7 f ck
Concrete in sea water or exposed The factor of safety against
directly along the sea coast shall be overturning and sliding should not
at least M20 grade in he case of be less than 1.4i.e.
plain concrete and M30 in case of FOS = Stabilizing force or moment /
reinforced concrete. The use of slag Destabilizing force or moment
or pozzolana cement is In case of retaining walls the
advantageous under such condition. stabilizing force is due to dead load,
thus these stabilizing forces are
Q. 86 (b) factored by a value of 0.9 (IS 456 Cl.
In tall buildings, it is important to 20)
control lateral displacements within ∴FOS = 0.9 Stabilizing force or
the serviceability limit state. A moment / Destabilizing force or
structural system may be classified moment ≥ 1.4
as follows:
i) Building frame system Q. 92 (b)
ii) Moment resisting frame system The stress diagram for concrete
iii) Dual frame system compression is shown below
iv) Tube system
In a moment resistant frame, The
relative stiffness of beam and
columns is very important. A frame
may be designed using weak column
strong beam proportions or strong
column weak beam proportions. A
frame with weak beam-strong
column is more stable and therefore Q. 93 (a)
highly desirable. Maximum strain in concrete under
compression is 0.0035

© Copyright Reserved by Gateflix.in No part of this material should be copied or reproduced without permission
inelastic region because the
modulus of elasticity is independent
of the variations in the yield
strength.

Q. 98 (a)
Using similar triangle Q. 99 (c)
0.0035 ∈sc
= W1 = W′ R +T
2 2

XU ( X U −d ) T
 d 
∈sc = 1 −  0.0035 =5×
(150) 2 + (250) 2
 X u  250
= 5.83 kN/ m 2
Q. 94 (b)
Design strength of concrete is Q.100 (a)
obtained is obtained after applying See clause 6.2.5 of IS 456: 2000
adequate factor of safety (FOS = strain that develops due o constant
1.5 for concrete) to the sustain loading is called creep
characteristic strength of concrete. strain but in the initial age of
Here characteristic strength of concrete, creep strain of concrete is
concrete in the actual structure is higher than later age. However,
taken as 0.67fck and hence the elastic strain remains constant
design strength of concrete is throughout. So creep coefficient =
0.67fck/γ Creep strain/Elastic strain, decrease
∴ Design strength = with time.
0.67fck/γ
= 0.67 fck/1.5 = 0.447fck

Q. 95 (a)

For such a failure to happen


τ>> σ

Q. 96 (c)

Q. 97 (c) 1.In case of concrete, the partial


safety factor γc is applicable at all
stress levels because the stress -
strain curve is directly affected by
changes in the compressive strength
of concrete.
2. In case of steel, the partial safety
factor γs is applicable only for the

© Copyright Reserved by Gateflix.in No part of this material should be copied or reproduced without permission

S-ar putea să vă placă și